Siyavula: Life Sciences Grade 10

317
Siyavula: Life Sciences Grade 10 Collection Editors: Siyavula Megan Beckett

Transcript of Siyavula: Life Sciences Grade 10

Siyavula: Life Sciences Grade 10

Collection Editors:Siyavula

Megan Beckett

Siyavula: Life Sciences Grade 10

Collection Editors:Siyavula

Megan Beckett

Authors:

Jesuit Virtual Learning AcademySiyavula

Megan BeckettTom Caswell

Denver Greene

Colleen HenningDr Zdzislaw (Gustav) Meglicki, Jr

Zamekile SondzabaDaniel Williamson

Online:< http://cnx.org/content/col11410/1.3/ >

C O N N E X I O N S

Rice University, Houston, Texas

This selection and arrangement of content as a collection is copyrighted by Siyavula, Megan Beckett. It is licensed

under the Creative Commons Attribution 3.0 license (http://creativecommons.org/licenses/by/3.0/).

Collection structure revised: April 11, 2012

PDF generated: October 29, 2012

For copyright and attribution information for the modules contained in this collection, see p. 304.

Table of Contents

1 Life at the molecular, cellular and tissue level

1.1 The chemistry of life . . . . . . . . . . . . . . . . . . . . . . . . . . . . . . . . . . . . . . . . . . . . . . . . . . . . . . . . . . . . . . . . . . . . . . . . 11.2 Cells - the basic units of life . . . . . . . . . . . . . . . . . . . . . . . . . . . . . . . . . . . . . . . . . . . . . . . . . . . . . . . . . . . . . . . . . 11.3 Cell division - mitosis . . . . . . . . . . . . . . . . . . . . . . . . . . . . . . . . . . . . . . . . . . . . . . . . . . . . . . . . . . . . . . . . . . . . . . . 11.4 Plant and animal tissues . . . . . . . . . . . . . . . . . . . . . . . . . . . . . . . . . . . . . . . . . . . . . . . . . . . . . . . . . . . . . . . . . . . . 25

2 Life processes in plants and animals

2.1 Support and transport systems in plants . . . . . . . . . . . . . . . . . . . . . . . . . . . . . . . . . . . . . . . . . . . . . . . . . . . . 532.2 Support systems in animals . . . . . . . . . . . . . . . . . . . . . . . . . . . . . . . . . . . . . . . . . . . . . . . . . . . . . . . . . . . . . . . . . 752.3 Transport systems in mammals (humans) . . . . . . . . . . . . . . . . . . . . . . . . . . . . . . . . . . . . . . . . . . . . . . . . . . 114

3 Environmental studies3.1 Biospheres to ecosystems . . . . . . . . . . . . . . . . . . . . . . . . . . . . . . . . . . . . . . . . . . . . . . . . . . . . . . . . . . . . . . . . . . 155

4 Diversity, change and continuity

4.1 Biodiversity and classi�cation . . . . . . . . . . . . . . . . . . . . . . . . . . . . . . . . . . . . . . . . . . . . . . . . . . . . . . . . . . . . . 2234.2 History of life on Earth . . . . . . . . . . . . . . . . . . . . . . . . . . . . . . . . . . . . . . . . . . . . . . . . . . . . . . . . . . . . . . . . . . . . 231Solutions . . . . . . . . . . . . . . . . . . . . . . . . . . . . . . . . . . . . . . . . . . . . . . . . . . . . . . . . . . . . . . . . . . . . . . . . . . . . . . . . . . . . . . . 281

5 Glossary . . . . . . . . . . . . . . . . . . . . . . . . . . . . . . . . . . . . . . . . . . . . . . . . . . . . . . . . . . . . . . . . . . . . . . . . . . . . . . . . . . . . . . . . . . 285Glossary . . . . . . . . . . . . . . . . . . . . . . . . . . . . . . . . . . . . . . . . . . . . . . . . . . . . . . . . . . . . . . . . . . . . . . . . . . . . . . . . . . . . . . . . . . . . 302Index . . . . . . . . . . . . . . . . . . . . . . . . . . . . . . . . . . . . . . . . . . . . . . . . . . . . . . . . . . . . . . . . . . . . . . . . . . . . . . . . . . . . . . . . . . . . . . . 303Attributions . . . . . . . . . . . . . . . . . . . . . . . . . . . . . . . . . . . . . . . . . . . . . . . . . . . . . . . . . . . . . . . . . . . . . . . . . . . . . . . . . . . . . . . .304

iv

Available for free at Connexions <http://cnx.org/content/col11410/1.3>

Chapter 1

Life at the molecular, cellular and tissue

level

1.1 The chemistry of life

1.2 Cells - the basic units of life

1.3 Cell division - mitosis

1.3.1 1.3.1 The Cell Cycle and Mitosis1

Unit 1.3 Cell Division - MitosisThe Cell Cycle and MitosisAnimation on mitosis: http://www.sumanasinc.com/webcontent/animations/content/mitosis.html 2

Khan: Phases of mitosishttp://www.khanacademy.org/video/phases- ofmitosis?playlist=BiologyNeok12: Mitosis games and videos http://www.neok12.com/Cell-Division.htm 3

Cell Cycle:http://www.daviddarling.info/encyclopedia/C/cell_cycle.htmlKhan: Chromosomes, Chromatids, Chromatin, etc: http://www.khanacademy.org/video/chromosomes�

chromatids�chromatin� tc?playlist=Biology

1.3.1.1 Introduction

The cell cycle is the series of events that takes place in a cell 4 leading to its division and duplication(replication). In cells without a nucleus ( prokaryotic 5 cells e.g. bacteria), the cell cycle occurs through aprocess termed binary �ssion 6 . In cells with a nucleus ( eukaryotes 7 ), the cell cycle can be divided intwo brief periods: interphase 8 �during which the cell grows, accumulating nutrients needed for mitosisand duplicating its DNA 9 �and the mitosis 10 (M) phase, just after which the cell splits itself into twodistinct cells, often called "daughter cells". The cell-division cycle is a vital process by which a single-celled

1This content is available online at <http://cnx.org/content/m43135/1.1/>.2http://www.sumanasinc.com/webcontent/animations/content/mitosis.html3http://www.neok12.com/Cell-Division.htm4http://en.wikipedia.org/wiki/Cell_(biology)5http://en.wikipedia.org/wiki/Prokaryotic6http://en.wikipedia.org/wiki/Binary_�ssion7http://en.wikipedia.org/wiki/Eukaryotes8http://en.wikipedia.org/wiki/Interphase9http://en.wikipedia.org/wiki/DNA_replication

10http://en.wikipedia.org/wiki/Mitosis

Available for free at Connexions <http://cnx.org/content/col11410/1.3>

1

2CHAPTER 1. LIFE AT THE MOLECULAR, CELLULAR AND TISSUE

LEVEL

fertilized egg 11 develops into a mature organism, as well as the process by which hair 12 , skin 13 , bloodcells 14 , some internal organs are renewed and wounds are healed

Figure 1.1

Diagram - Cell division.

1.3.1.2 Phases

The cell cycle consists of four distinct phases: G 15 1 16 phase 17 , S phase 18 (synthesis), G 19 220 phase 21 (collectively known as interphase 22 ) and M phase 23 (mitosis). M phase is itself composedof two tightly coupled processes: mitosis, in which the cell's chromosomes 24 are divided between the twodaughter cells, and cytokinesis 25 , in which the cell's cytoplasm 26 divides in half forming two distinctcells.

11http://en.wikipedia.org/wiki/Fertilized_egg12http://en.wikipedia.org/wiki/Hair13http://en.wikipedia.org/wiki/Skin14http://en.wikipedia.org/wiki/Blood_cell15http://en.wikipedia.org/wiki/G1_phase16http://en.wikipedia.org/wiki/G1_phase17http://en.wikipedia.org/wiki/G1_phase18http://en.wikipedia.org/wiki/S_phase19http://en.wikipedia.org/wiki/G2_phase20http://en.wikipedia.org/wiki/G2_phase21http://en.wikipedia.org/wiki/G2_phase22http://en.wikipedia.org/wiki/Interphase23http://en.wikipedia.org/wiki/Mitosis24http://en.wikipedia.org/wiki/Chromosomes25http://en.wikipedia.org/wiki/Cytokinesis26http://en.wikipedia.org/wiki/Cytoplasm

Available for free at Connexions <http://cnx.org/content/col11410/1.3>

3

Figure 1.2

Diagram - Schematic of the cell cycle. outer ring: I = Interphase 27 , M = Mitosis 28 ; inner ring: M= Mitosis 29 , G 1 = Gap 1 30 , G 2 = Gap 2 31 , S = Synthesis 32 ; not in ring: G 0 = Gap0/Resting 33 . [1]

PhaseAbbreviationDescription

quiescent/senescent Gap 0 34 G 0 A resting phase where the cell has left the cycle and has stoppeddividing.

27http://en.wikipedia.org/wiki/Interphase28http://en.wikipedia.org/wiki/Mitosis29http://en.wikipedia.org/wiki/Mitosis30http://en.wikipedia.org/wiki/G1_phase31http://en.wikipedia.org/wiki/G2_phase32http://en.wikipedia.org/wiki/S_phase33http://en.wikipedia.org/wiki/G0_phase

Available for free at Connexions <http://cnx.org/content/col11410/1.3>

4CHAPTER 1. LIFE AT THE MOLECULAR, CELLULAR AND TISSUE

LEVEL

Interphase 35 Gap 1 36 G 1 Cells increase in size in Gap 1. The G 1 checkpoint controlmechanism ensures that everything is ready for DNA 37 synthesis.

Synthesis 38 SDNA replication 39 occurs during this phase.

Gap 2 40 G 2 During the gap between DNA synthesis and mitosis, the cell will continue to grow.The G 2 checkpoint control mechanism ensures that everything is ready to enter the M (mitosis) phaseand divide.

Cell division 41 Mitosis 42 MCell growth stops at this stage and cellular energy is focused on the orderly division into two daughter

cells. A checkpoint in the middle of mitosis ( Metaphase Checkpoint ) ensures that the cell is ready tocomplete cell division.

PhaseAbbreviationDescription

quiescent/senescent Gap 0 34 G 0 A resting phase where the cell has left the cycle and has stoppeddividing.

Interphase 35 Gap 1 36 G 1 Cells increase in size in Gap 1. The G 1 checkpoint controlmechanism ensures that everything is ready for DNA 37 synthesis.

Synthesis 38 SDNA replication 39 occurs during this phase.

Gap 2 40 G 2 During the gap between DNA synthesis and mitosis, the cell will continue to grow.The G 2 checkpoint control mechanism ensures that everything is ready to enter the M (mitosis) phaseand divide.

Available for free at Connexions <http://cnx.org/content/col11410/1.3>

5

Cell division 41 Mitosis 42 MCell growth stops at this stage and cellular energy is focused on the orderly division into two daughter

cells. A checkpoint in the middle of mitosis ( Metaphase Checkpoint ) ensures that the cell is ready tocomplete cell division.

PhaseAbbreviationDescription

quiescent/senescent Gap 0 43 G 0 A resting phase where the cell has left the cycle and has stoppeddividing.

Interphase 44 Gap 1 45 G 1 Cells increase in size in Gap 1. The G 1 checkpoint controlmechanism ensures that everything is ready for DNA 46 synthesis.

Synthesis 47 SDNA replication 48 occurs during this phase.

Gap 2 49 G 2 During the gap between DNA synthesis and mitosis, the cell will continue to grow.The G 2 checkpoint control mechanism ensures that everything is ready to enter the M (mitosis) phaseand divide.

Cell division 50 Mitosis 51 MCell growth stops at this stage and cellular energy is focused on the orderly division into two daughter

cells. A checkpoint in the middle of mitosis ( Metaphase Checkpoint ) ensures that the cell is ready tocomplete cell division.

Available for free at Connexions <http://cnx.org/content/col11410/1.3>

6CHAPTER 1. LIFE AT THE MOLECULAR, CELLULAR AND TISSUE

LEVEL

State Phase Abbreviation Description

quiescent/senescent Gap 0 34 G 0 A resting phase where the cell has left the cycle and has stopped dividing.

Interphase 35 Gap 1 36 G 1 Cells increase in size in Gap 1. The G 1 checkpoint control mechanism ensures that everything is ready for DNA 37 synthesis.

Synthesis 38 S DNA replication 39 occurs during this phase.

Gap 2 40 G 2 During the gap between DNA synthesis and mitosis, the cell will continue to grow. The G 2 checkpoint control mechanism ensures that everything is ready to enter the M (mitosis) phase and divide.

Cell division 41 Mitosis 42 M Cell growth stops at this stage and cellular energy is focused on the orderly division into two daughter cells. A checkpoint in the middle of mitosis ( Metaphase Checkpoint ) ensures that the cell is ready to complete cell division.

Table 1.1

Table � Phases of the cell cycle (the details of the G and S phases are not required but are included togive an overview).

34http://en.wikipedia.org/wiki/G0_phase35http://en.wikipedia.org/wiki/Interphase36http://en.wikipedia.org/wiki/G1_phase37http://en.wikipedia.org/wiki/DNA38http://en.wikipedia.org/wiki/S_phase39http://en.wikipedia.org/wiki/DNA_replication40http://en.wikipedia.org/wiki/G2_phase41http://en.wikipedia.org/wiki/Cell_division42http://en.wikipedia.org/wiki/Mitosis

Available for free at Connexions <http://cnx.org/content/col11410/1.3>

7

1.3.1.3 Stages of Mitosis

Figure 1.3

Diagram � Allium (Onion) cells in the di�erent cycle of mitosis.

1.3.1.3.1 Interphase

The cell spends most of its life in the interphase. During this phase the cell grows to its maximum size andperforms its normal functions. Many scientists do not count interphase as part of mitosis.

1.3.1.3.2 Prophase

The chromatin (a special protein (actually a nucleoprotein) that chromosomes are made of) condenses intochromosomes (human cells have 46 chromosomes � 23 from your father and 23 from your mother). Eachchromosome eventually can be seen to consist of two strands or chromatids joined at a central centromerein an X shape. The nuclear membrane disappears. The centriole splits and starts to move to opposite poles.Spindle threads form between the poles.

Available for free at Connexions <http://cnx.org/content/col11410/1.3>

8CHAPTER 1. LIFE AT THE MOLECULAR, CELLULAR AND TISSUE

LEVEL

1.3.1.3.3 Metaphase

Chromosomes lie on the equator of the cell. Each chromosome is attached to the spindle micro�bers by itscentromere. The chromosomes appear in a straight line across the middle of the cell.

In the other form of cell division, meiosis, homologous chromosomes line up in pairs, side by side.

1.3.1.3.4 Anaphase

The centromere splits. Each chromosome divides into two sister chromatids. Each chromatid is moved toopposite poles of the cell by the shortening of the spindle �bres.

Chromatids (now called daughter chromosomes ) gather at opposite poles of the cell.

1.3.1.3.5 Telophase

A nuclear membrane forms around each of the daughter chromosomes that have gathered at the poles. Thedaughter chromosomes uncoil to form di�use chromatin. The cytoplasm then divides during a process calledcytokinesis . Note �cytokinesis is not a stage of mitosis but the process of the cytoplasm splitting intotwo. There are now two genetically identical daughter cells. They are identical to the parent cell and to eachother. In an animal cell an invagination or infolding will divide the cytoplasm. In a plant cell a crosswall divides the cytoplasm.

Animation � Cell cycle and stages of mitosis �http://highered.mcgraw-hill.com/sites/0072495855/student_view0/chapter2/animatio

n__how_the_cell_cycle_works.html 43

1.3.1.3.6 Summary of mitosis

Two identical daughter cells are formed from the mother cell.Each daughter cell has the same number of chromosomes as the mother cell.Each daughter cell will grow to its maximum size.

1.3.1.4 Biological importance of mitosis

Growth � Living tissue grows by mitosis e.g. bone and skin.Repair - Damaged and worn-out tissues are replaced with new cells by mitosis.Asexual reproduction - Single-celled (unicellular) organisms and bacteria often reproduce asexually by

mitosis. Organisms like amoeba are able to split from a single individual into two and therefore can reproducewithout a mate and sexual reproduction.

1.3.1.5 Chromosomes

In mitosis at the end of prophase the chromosomes appear as X-shaped threads. Each thread is in fact achromatid and they are joined in the centre at a point called the centromere. There are two of each chromo-some and the full set of chromosomes is often shown with the complimentary or homologous chromosomespaired up.

43http://highered.mcgraw-hill.com/sites/0072495855/student_view0/chapter2/animation__how_the_cell_cycle_works.html

Available for free at Connexions <http://cnx.org/content/col11410/1.3>

9

Figure 1.4

1.3.1.6 Activity 1 - Investigating mitosis in allium root tip squash

IntroductionTalking about what chromosomes do during mitosis could be very interesting, but seeing them for yourself

adds an extra dimension.Video - Preparing Microscope Slideshttp://www.youtube.com/watch?v=GHnndVuaync 44

Lesson OrganisationThe allium/onoin roots need to be prepared 1-10 days in advance of the lesson. Some practitioners

report that cutting the root tips around noon makes a di�erence to the mitotic index, so you may wantyour technician to cut and `�x' the tips in ethanoic alcohol rather than ask your students to carry out thisstep. If you have access to a video microscope it is worth capturing some images as this procedure can befrustrating.

Method1. Pour approximately 5 ml. methanol-acetic acid �xative into a small beaker. Place 2-3 mm length

onion root tip into the �xative. Incubate at 60 C for 15 mins. 2. At the end of the �xative incubation period,pour o� the �xative into a waste beaker. Be careful not to pour o� your �xed onion root tip. Now addapproximately 5 ml. 1 M HCl to your �xed onion root tip to partially hydrolize the cells. Incubate at 60C for exactly 10 mins. 3. At the end of the hydrolysis incubation period, pour o� the 1 M HCl into awaste beaker. Be careful not to pour o� your �xed/hydrolyzed onion root tip. Now add approximately 1 ml.

44http://www.youtube.com/watch?v=GHnndVuaync

Available for free at Connexions <http://cnx.org/content/col11410/1.3>

10CHAPTER 1. LIFE AT THE MOLECULAR, CELLULAR AND TISSUE

LEVEL

Feulgen stain to your �xed/hydrolyzed onion root tip in order to stain the chromosomes. Bath your onionroot tip in the Feulgen stain for 20-30 mins. to allow the Feulgen stain to penetrate the chromosomes.4. Tomake a slide of your stained onion root tip, transfer your onion root tip from the beaker to a microscopeslide and add a small drop of 45% acetic acid. Do not allow the onion root tip to dry out duringthe subsequent steps. Add 45% acetic acid if you notice your specimen is drying out. 5.Pulverize your onion root tip into a �ne pulp on the microscope slide by tapping it with a glass rod. Tryto produce as �ne a pulp as possible to prevent large cell clumps which will not be useful for microscopicexamination.6. Now place a microscope cover slip on top of your pulverized onion root tip. Put two layersof paper towel on top of the microscope cover slip and press down hard enough to squash the root but notenough to break the cover slip. This should result in the onion root tip cells from forming a monolayer whichis ideal for microscopic examination.7. Using the scanning objective focus on the onion root tip cells andidentify a cell undergoing mitosis by looking for pink-staining bodies (chromosomes) within the cell.8. Switchto the low-magni�cation10X objective and �ne-focus.9. Switch to the high-magni�cation 40X objective and�ne-focus. At this magni�cation you should be able to identify cells in several stages of mitosis. Identify aspeci�c stage of mitosis and go on to step 10.10. Add a drop of oil to your prepared slide and switch to theoil-immersion 100X objective and �ne-focus. Repeat steps 9 and 10 for all four stages of mitosis.As part of your Lab Exit Quiz you will be asked to: 1) show your prepared onion root tip slideto a lab instructor under the microscope.2) identify several stages of mitosis on your preparedonion root tip slide.

Mitotic Index

Figure 1.5

The duration of each stage of mitosis has been recorded and the data (see table below) could be used tocompare the observed frequencies of the di�erent stages as recorded by students.

1.3.1.7 Activity 2 � Stages of Mitosis

Task � Look at Cells 1 -5. Decide which stage of mitosis each cell is in. For each cell describe the featuresthat make you think it is in this stage.

Available for free at Connexions <http://cnx.org/content/col11410/1.3>

11

Figure 1.6Available for free at Connexions <http://cnx.org/content/col11410/1.3>

12CHAPTER 1. LIFE AT THE MOLECULAR, CELLULAR AND TISSUE

LEVEL

1.3.1.8 Activity 3 - Quiz on Onion Tip Mitosis (with Answers)

1. 1. Why do we study the root tip to �nd mitosis instead of any other part of the onion plant? We studythe root tip because it is growing therefore cells are dividing rapidly. This makes it the best part of the plantto observe various stages of mitosis.

2. Based on you data what can you infer about the relative length of time an onion root-tip cell spendsin each stage of the cell cycle? Most cells are in interphase because most time is spent in this phase.

3. Based on your understanding of the structure of the chromosome, why might it take longer to completeprophase than the other phases of nuclear division? Prophase is the longest phase of mitosis because thechromosomes have to coil up into organized bodies. It takes a long time for the chromatin to coil or condenseinto chromosomes.

4. How do you account for the di�erences between the slides made by di�erent groups? Possible answers:Not all lab groups had the same slide so there can be di�erences among the growth rates of the plants thatwere used to prepare the slide. The groups may have been looking at di�erent areas of the root. Somegroups may not have followed the instructions as carefully as others.

5. If you examined cells in the Zone of Di�erentiation (Zone of Maturation) would you expect to getsimilar results? No Why or why not? These cells are starting to specialize into mature tissues. They areno longer meristematic cells.

6. Why did we use the pie chart to graph the data? The pie chart was used because the data representedthe parts of a whole and it is relatively easy to show proportions of the whole event.

1.3.1.9 Assignment 1 - Animation of Mitosis and Multiple Choice Questions on Mitosis

http://bealbio.wikispaces.com/Genetics 45

Description: Gives a detailed description of the steps involved in mitosis with animated videos andnarrative voice-over. Short multiple choice quiz provided containing the following multiple choice questions:

1Which of thefollowing eventsdo NOT occur inprophase ofmitosis?

Figure 1.7

A)

Figure 1.8

DNA condenses to form chromosomes

continued on next page

45http://bealbio.wikispaces.com/Genetics

Available for free at Connexions <http://cnx.org/content/col11410/1.3>

13

Figure 1.9

B)

Figure 1.10

nuclear membrane breaks down

Figure 1.11

C)

Figure 1.12

nucleolus breaks down

Figure 1.13

D)

Figure 1.14

chromosomes are replicated

Figure 1.15

E)

Figure 1.16

mitotic spindle begins to form

2The mitoticspindle �bersattach tochromosomes viaspecial structurestermed

continued on next page

Available for free at Connexions <http://cnx.org/content/col11410/1.3>

14CHAPTER 1. LIFE AT THE MOLECULAR, CELLULAR AND TISSUE

LEVEL

Figure 1.17

A)

Figure 1.18

centrioles.

Figure 1.19

B)

Figure 1.20

asters.

Figure 1.21

C)

Figure 1.22

centromeres

Figure 1.23

D)

Figure 1.24

centrosomes.

Figure 1.25

E)

Figure 1.26

keratins.

continued on next page

Available for free at Connexions <http://cnx.org/content/col11410/1.3>

15

3Which of thefollowingstatements aboutspindle �bresduring anaphase isTRUE?

Figure 1.27

A)

Figure 1.28

those attached to chromosomes elongate, while those that areunattached shorten

Figure 1.29

B)

Figure 1.30

those attached to chromosomes shorten, while those that areunattached elongate

Figure 1.31

C)

Figure 1.32

both attached and unattached microtubules shorten

Figure 1.33

D)

Figure 1.34

both attached and unattached microtubules elongate

continued on next page

Available for free at Connexions <http://cnx.org/content/col11410/1.3>

16CHAPTER 1. LIFE AT THE MOLECULAR, CELLULAR AND TISSUE

LEVEL

Figure 1.35

E)

Figure 1.36

both attached and unattached microtubules elongate at �rstand then shorten

4Centromeresdivide duringmetaphase.

Figure 1.37

A)

Figure 1.38

True

Figure 1.39

B)

Figure 1.40

False

5Cytokinesis inplant cells occursby means of acleavage furrow.

Figure 1.41

A)

Figure 1.42

True

continued on next page

Available for free at Connexions <http://cnx.org/content/col11410/1.3>

17

Figure 1.43

B)

Figure 1.44

False

Table 1.2

1.3.1.10 Assignment 2 �Observing Mitosis in an Onion Root Tip Activity

Mitosis SlidesIdentify the stage of mitosis for each of the onion root-tip slides below (most stages are

represented more than once).

Identify stage Identify stage

Figure 1.45

__i)_________________________

Figure 1.46

__ii_________________________

continued on next page

Available for free at Connexions <http://cnx.org/content/col11410/1.3>

18CHAPTER 1. LIFE AT THE MOLECULAR, CELLULAR AND TISSUE

LEVEL

Figure 1.47

______iii_____________________

Figure 1.48

__________________iv_________

Figure 1.49

______________v_____________

Figure 1.50

____vi_______________________

continued on next page

Available for free at Connexions <http://cnx.org/content/col11410/1.3>

19

Table 1.3

Practice locating each of the stages of mitosis in the following slides of the onion root tip.Each picture contains at least one cell at each stage of mitosis (and some stages are representedby multiple cells).

Figure 1.51

(vii)Answers

i. Prophase; ii) Anaphase iii) Metaphase iv) Interphase v) Anaphase vi) Metaphase

Answers to (vii)The numbered arrows indicate cells at various stages of mitosis (most of the rest of the

cells are in interphase). The key to the stages is to the right of each �gure. You will notbe asked to distinguish early from late; this is provided to help clarify the more ambiguousstages.

Available for free at Connexions <http://cnx.org/content/col11410/1.3>

20CHAPTER 1. LIFE AT THE MOLECULAR, CELLULAR AND TISSUE

LEVEL

Figure 1.52

Key:

1. late Prophase/early Metaphase2. Metaphase3. Telophase4. early Telophase5. early Telophase6. Metaphase7. Prophase8. Anaphase9. late Anaphase

Table 1.4

1.3.2 1.3.2 Cancer46

Khan video: Cancerhttp://www.khanacademy.org/video/cancer?playlist=BiologyInside Cancer (very good summary)http://www.insidecancer.org/

46This content is available online at <http://cnx.org/content/m43059/1.1/>.

Available for free at Connexions <http://cnx.org/content/col11410/1.3>

21

Figure 1.53

Picture - An invasive ductal carcinoma of the breast (pale area at the center) surrounded by spikes ofwhitish scar tissue in the surrounding yellow fatty tissue.

1.3.2.1 Introduction

Cancer is essentially a disease of mitosis - the normal 'checkpoints' regulating mitosis are ignored or over-ridden by the cancer cell. Cancer begins when a single cell is transformed, or converted from a normal cellto a cancer cell.

Cancer can be caused by agents called carcinogens, such as cigarette smoke, x-rays, nuclear radiation,UV light and some chemicals.

Carcinogens can cause a DNA mutation that occurs in one of several genes that normally function tocontrol growth. E.g. the BRCA 1 gene, the "Breast Cancer Gene" normally functions to supress tumorformation; but if a gene contains mutations such that BRCA1 does not work properly, tumor formation canbegin (Note: mutations in this gene do not mean that a person will develop breast cancer, just that theyhave an increased risk for breast cancer).

Once these crucial Cell Cycle genes start behaving abnormally, cancer cells start to develop wildly byrepeated, uncontrolled mitosis.

Video - Very simply explained visual depiction of cancer. http://www.youtube.com/watch?v=LEpTTolebqo47

47http://www.youtube.com/watch?v=LEpTTolebqo

Available for free at Connexions <http://cnx.org/content/col11410/1.3>

22CHAPTER 1. LIFE AT THE MOLECULAR, CELLULAR AND TISSUE

LEVEL

1.3.2.2 Tumours

The cancer cells grow and divide to form mass of cancer cells called a tumor. As the tumor grows larger, itbegins to release proteins from the cell to attract new blood vessel growth.

1.3.2.2.1 Benign Tumours

Tumour cells remain at original site. They do not spread and usually cause little trouble. They can beremoved surgically or killed by radiation, usually eliminating any further cancer development at that site.

1.3.2.2.2 Malignant Tumours

Malignant tumours are dangerous. The malignant cells grow squashing normal cells and preventing themworking normally. Some tumour cells send out signals that tell the body to produce a new blood vessel atthe tumour site. These cells not only have their own food and oxygen supply, they also have an avenue forescape to a new part of the body - through the new blood vessel and into bloodstream. Cells that breakaway from the tumour begin to spread to surrounding tissues (via the bloodstream or lymph) and startnew tumors = metastasis. Usually surgery is performed to remove the tumour, followed by radiation andchemotherapy.

1.3.2.3 Cancer Treatment

1.3.2.3.1 Radiation therapy

Radiation therapy (also called radiotherapy, X-ray therapy, or irradiation) is the use of ionizing radiation tokill cancer cells and shrink tumors. The e�ects of radiation therapy are localised and con�ned to the regionbeing treated. Radiation therapy injures or destroys cells in the area being treated (the "target tissue")by damaging their genetic material, making it impossible for these cells to continue to grow and divide.Although radiation damages both cancer cells and normal cells, most normal cells can recover from thee�ects of radiation and function properly. The goal of radiation therapy is to damage as many cancer cells aspossible, while limiting harm to nearby healthy tissue. Hence, it is given in many fractions, allowing healthytissue to recover between fractions.

1.3.2.3.2 Chemotherapy

Chemotherapy is the treatment of cancer with drugs ("anticancer drugs") that can destroy cancer cells. Incurrent usage, the term "chemotherapy" usually refers to cytotoxic drugs which a�ect rapidly dividing cellsin general, in contrast with targeted therapy (see below). Chemotherapy drugs interfere with cell divisionin various possible ways, e.g. with the duplication of DNA or the separation of newly formed chromosomes.Chemotherapy has the potential to harm healthy tissue, especially those tissues that have a high replacementrate (e.g. intestinal lining). These cells usually repair themselves after chemotherapy.

Because some drugs work better together than alone, two or more drugs are often given at the same time.This is called "combination chemotherapy"; most chemotherapy regimens are given in a combination.

Available for free at Connexions <http://cnx.org/content/col11410/1.3>

23

Figure 1.54

Patient undergoing chemotherapy.http://www.�ickr.com/photos/franziska/314275598/sizes/o/in/photostream/Animation - Overview- text and diagram, last part is video:http://insidecancer.org/ Select `Hallmarks of Cancer'- `overview'Animation � Cancer: An introduction to what cancer is and how it is the by-product of broken DNA

replication.http://www.khanacademy.org/video/cancer?playlist=Biology

1.3.2.3.3 Activity 4 - Cancer in South Africa

Look at the following tablePercentage of cancer deaths by cause in South Africa 2000

Available for free at Connexions <http://cnx.org/content/col11410/1.3>

24CHAPTER 1. LIFE AT THE MOLECULAR, CELLULAR AND TISSUE

LEVEL

Rank In All People Cause of Death % In All People % In Men % In Women

1 Tracheal/Bronchal/Lung cancer 16.5 21.9 10.9

2 Oesophageal Cancer 13.4 16.7 9.9

3 Cervical Cancer 8.4 17.2

4 Breast Cancer 7.7 0.2 15.6

5 Liver Cancer 6.4 5.4 6.9

6 Colo-Rectal cancer 6.2 5.4 6.9

7 Prostate Cancer 6.1 11.8

8 Stomach Cancer 5.6 6.5 4.7

9 Pancreatic Cancer 3.7 3.7 3.7

10 Leukaemia 3.5 3.8 3.2

Table 1.5

1.Draw a bar graph to show the percentage deaths for each type of cancer for men and women (12 marks)2.Which type of Cancer is the most common in a) men and b) women. (2 marks)3.Why are there no percentages given for a) men with cervical cancer and b) women with prostate cancer?

(2 marks)

Available for free at Connexions <http://cnx.org/content/col11410/1.3>

25

1.3.3 1.3.3 Summary48

1.3.3.1 Summary

1.3.3.1.1 Mitosis and the Cell Cycle

The cell cycle is made of stages including G1, S, G2 and Mitosis.Cell division by mitosis has �ve stages: Interphase (which in theory is the phase between divisions and

not part of mitosis), Prophase, Metaphase, Anaphase and Telophase.Mitosis is important in growth, repair and asexual reproduction.

1.3.3.1.2 Reproduction in simple organisms

Simple organisms such as bacteria and protozoa reproduce asexually using mitosis,

1.3.3.1.3 Cancer

Cancer is due to cells dividing out of control by mitosis.There are many types of cancer.Cancer can be treated by chemotherapy and radiotherapy.Video - Overview: Binary �ssion- Mitosis and cell division- Cancer

http://www.neok12.com/php/watch.php?v=zX45724701445b595e766477&t=Cell- Division 49

1.4 Plant and animal tissues

1.4.1 1.4.1 What is a tissue?50

1.4.1.1

Tissue is a level of organization level between cells 51 and a complete organism. A tissue is a group ofcells, not necessarily identical, but from the same origin, that together carry out a speci�c function. Theseare called tissues because of they are functioning together. Organs 52 are then formed by the functionalgrouping together of multiple tissues.

Tissues- http://www.s-cool.co.uk/a-level/biology/cells-and-organelles/revise-it/tissues 53

1.4.2 1.4.2 Plant Tissues54

1.4.2.1

Video � Intro to plant tissues: http://www.youtube.com/watch?v=4V0VcNCRKTo&feature=related 55

Types of plant tissues: http://users.rcn.com/jkimball.ma.ultranet/BiologyPages/P/PlantTissues.html56

Plant tissues picture: http://www.britannica.com/EBchecked/media/388/Cell-types-and-tissues 57

48This content is available online at <http://cnx.org/content/m43130/1.1/>.49http://www.neok12.com/php/watch.php?v=zX45724701445b595e766477&t=Cell-Division50This content is available online at <http://cnx.org/content/m43172/1.1/>.51http://en.wikipedia.org/wiki/Cell_(biology)52http://en.wikipedia.org/wiki/Organs53http://www.s-cool.co.uk/a-level/biology/cells-and-organelles/revise-it/tissues54This content is available online at <http://cnx.org/content/m43140/1.1/>.55http://www.youtube.com/watch?v=4V0VcNCRKTo&feature=related56http://users.rcn.com/jkimball.ma.ultranet/BiologyPages/P/PlantTissues.html57http://www.britannica.com/EBchecked/media/388/Cell-types-and-tissues

Available for free at Connexions <http://cnx.org/content/col11410/1.3>

26CHAPTER 1. LIFE AT THE MOLECULAR, CELLULAR AND TISSUE

LEVEL

Figure 1.55: Di�erent types of plant tissue

Note - Protective tissue (shown here) is better described as epidermal tissue

1.4.2.1.1 Meristematic tissue

Meristematic tissue consists of cells dividing by mitosis. This leads to an increase in length and thicknessof the plant. The primary growth of a plant occurs only in certain regions, such as in the tips of stems orroots. It is in these regions that meristematic tissue is present. Cells in these tissues are roughly sphericalor polyhedral, to rectangular in shape, and have thin cell walls and no vacuoles or plastids. New cellsproduced by meristem are initially those of meristem 58 itself, but as the new cells grow and mature, theircharacteristics slowly change and they become specialised.

1.4.2.1.2 Parenchyma

Parenchyma forms the majority of stems and roots as well as soft fruits like tomatoes and grapes. It consistsof relatively unspecialised cells with thin cell walls. They are living cells. They are usually loosely packed,so that large spaces between cells (intercellular spaces) are found in this tissue. This tissue provides supportto plants through turgor pressure and also stores food in its cells in the form of sugar and starch. In somesituations, it contains chlorophyll and performs photosynthesis, and then it is called chlorenchyma

1.4.2.1.3 Collenchyma

Collenchyma cells are thin-walled but possess thickening of cellulose 59 and pectin 60 substances at thecorners where number of cells join together. This tissue gives a tensile strength (because of the thickenedcorners) and �exibility due to the thinner areas). The cells are compactly arranged and do not have inter-

58http://en.wikipedia.org/wiki/Meristem59http://en.wikipedia.org/wiki/Cellulose60http://en.wikipedia.org/wiki/Pectin

Available for free at Connexions <http://cnx.org/content/col11410/1.3>

27

cellular spaces. It occurs chie�y in hypodermis 61 of stems and leaves. It is absent in monocots 62 andin roots. Collenchyma tissue acts as a supporting tissue in stems of young plants. It provides mechanicalsupport, elasticity, and tensile strength to the plant body.

Figure 1.56: Cross section of collenchyma cells http://en.wikipedia.org/wiki/File:Plant_cell_type_collenchyma.png63

1.4.2.1.4 Sclerenchyma

This tissue consists of thick-walled, dead cells. These cells have hard and extremely thick secondary wallsdue to uniform distribution of lignin 64 . Lignin deposition is so thick that the cell walls become strong,rigid and impermeable to water. Sclerenchyma cells are closely packed without inter- cellular spaces betweenthem. Thus, they appear as hexagonal net in transverse section 65 . The cells are cemented together withthe help of lamella 66 . The middle lamella 67 is a wall that lies between adjacent cells.

The main function of sclerenchyma tissues is to give support and protection to the plant. For examplesclerenchyma keeps stems upright and protects nuts in their hard shells.

61http://en.wikipedia.org/wiki/Hypodermis62http://en.wikipedia.org/wiki/Monocots63http://en.wikipedia.org/wiki/File:Plant_cell_type_collenchyma.png64http://en.wikipedia.org/wiki/Lignin65http://en.wikipedia.org/wiki/Transverse_section#Planes66http://en.wiktionary.org/wiki/lamella67http://en.wikipedia.org/wiki/Middle_lamella

Available for free at Connexions <http://cnx.org/content/col11410/1.3>

28CHAPTER 1. LIFE AT THE MOLECULAR, CELLULAR AND TISSUE

LEVEL

Figure 1.57: Cross section of sclerenchyma �bers. http://en.wikipedia.org/wiki/File:Plant_cell_type_sclerenchyma_�bers.png68

Note that some of the cells still have their cytoplasm contents and may not have died yet.

1.4.2.1.5 Epidermis

A single-layer of cells that covers plants' leaves 69 , �owers 70 , roots and stems 71 . It forms a boundarybetween the plant 72 and the external environment. The epidermis serves several functions, it protectsagainst water loss, regulates gas exchange, secretes metabolic compounds, and (especially in roots) absorbswater and mineral nutrients.

68http://en.wikipedia.org/wiki/File:Plant_cell_type_sclerenchyma_�bers.png69http://en.wikipedia.org/wiki/Leaf70http://en.wikipedia.org/wiki/Flowers71http://en.wikipedia.org/wiki/Plant_stem72http://en.wikipedia.org/wiki/Plant

Available for free at Connexions <http://cnx.org/content/col11410/1.3>

29

Figure 1.58: Scanning electron microscope image of Nicotiana alata upper leaf surface, showing tricomesand a few stomates. http://en.wikipedia.org/wiki/File:Leaf_epidermis_w_scale.jpg 73

1.4.2.1.6 Xylem

Xylem tissue has the dual function of supporting the plant and transporting water and dissolved mineralsalts. It is made up of vessel elements, tracheids, �bres and parenchyma cells. These cells are joined endto end to form long tubes. Vessels and tracheids are dead at maturity. Tracheids have thick secondary cellwalls and are tapered at the ends. The thick walls provide support. They do not have end openings like thevessels do. The tracheids ends overlap with each other, with pairs of pits present. The pit pairs allow waterto pass horizontally from cell to cell.

1.4.2.1.7 Phloem

Phloem tissue is responsible for translocation which is the transport of soluble organic substances - forexample, sugar. The substances travel along sieve elements but other types of cells are also present, thecompanion cells, parenchyma cells and �bres. Phloem is living tissue. The end walls, unlike vessel membersin xylem, do not have large openings. The end walls, however, are full of small pores where cytoplasmextends from cell to cell. These porous connections are called sieve plates. In spite of the fact that theircytoplasm is actively involved in the conduction of food materials, sieve-tube members do not have nucleiat maturity. The activity of the sieve tubes is controlled by companion cells through plasmadesmata.

73http://en.wikipedia.org/wiki/File:Leaf_epidermis_w_scale.jpg

Available for free at Connexions <http://cnx.org/content/col11410/1.3>

30CHAPTER 1. LIFE AT THE MOLECULAR, CELLULAR AND TISSUE

LEVEL

Figure 1.59: Phloem and xylem http://mrb-science.wikispaces.com/Roots+and+Stems 74

1.4.2.1.8 Investigation 1 - Observing Xylem Tissue

Materials:•2 soft young stems of a plant such as Impatiens, Aster or Lily � with leaves and a �ower•2 containers•Water•Blue / red food colouring•A scalpel•A hand lens•A glass slide•A cover slip•A microscopeMethod:1.Fill the containers with water.2.Put some blue food colouring into one container and red colouring into the other container.3.Cut the bottom end of the �owers' stems diagonally and put one stem into each container.4.Leave overnight (can leave longer)5.Cut the stem again near the cut end and use a hand lens to observe the cut surface.a.Do you notice any of the colouring/stain inside the plant?b.If so, where? Draw a diagram to show what you see.

74http://mrb-science.wikispaces.com/Roots+and+Stems

Available for free at Connexions <http://cnx.org/content/col11410/1.3>

31

6.Use the scalpel to cut a few THIN cross sections of the stem.7.Make a fresh wet mount slide of this section.8.Observe the section under the microscope and draw a labelled diagram of what YOU SEE!

Figure 1.60: Di�erences between monocotyledons and dicotyledons

1.4.3 1.4.3 Animal Tissues75

1.4.3.1 Animal Tissues

There are several main types of animal tissue including : epithelial, muscle, nervous tissue and connectivetissues such as bone, cartilage, blood and loose connective tissue (sometimes called areolar tissue).

Video - Animal tissues introduction: http://www.youtube.com/watch?v=I2bSWCyKOz0&feature=related76

Tissues of human body: http://www.mhhe.com/biosci/ap/histology_mh/tismodov.html 77

75This content is available online at <http://cnx.org/content/m43137/1.1/>.76http://www.youtube.com/watch?v=I2bSWCyKOz0&feature=related77http://www.mhhe.com/biosci/ap/histology_mh/tismodov.html

Available for free at Connexions <http://cnx.org/content/col11410/1.3>

32CHAPTER 1. LIFE AT THE MOLECULAR, CELLULAR AND TISSUE

LEVEL

Figure 1.61: Mammalian tissues

Available for free at Connexions <http://cnx.org/content/col11410/1.3>

33

1.4.3.1.1 Epithelial tissue

Figure 1.62: Four types of epithelial tissue (Ciliated epithelium is another specialized type of epithelialtissue.) http://www.hartnell.edu/tutorials/biology/tissues.html 78

Epithelial tissues are formed by cells that cover surfaces such as the surface of the skin, and line tubes andcavities, such as the digestive organs, blood vessels, kidney tubules and airways. The cells comprising anepithelial layer are linked via semi-permeable junctions; hence, this tissue provides a barrier between theexternal environment and the organ it covers. In addition to this protective function, epithelial tissue mayalso be specialized to function in secretion 79 and absorption 80 . Epithelial tissue helps to protect organismsfrom microorganisms, injury, and �uid loss. All epithelial tissues are free surfaces attached to the underlyinglayers by a basement membrane.

78http://www.hartnell.edu/tutorials/biology/tissues.html79http://en.wikipedia.org/wiki/Secretion80http://en.wikipedia.org/wiki/Digestion

Available for free at Connexions <http://cnx.org/content/col11410/1.3>

34CHAPTER 1. LIFE AT THE MOLECULAR, CELLULAR AND TISSUE

LEVEL

1.4.3.1.2 Muscle tissue

Figure 1.63: 3 types of muscle http://www.hartnell.edu/tutorials/biology/tissues.html 81

Muscle cells form the active contracting tissue of the body known as muscle tissue. Muscle tissue functionsto produce force and cause motion within internal organs. Muscle tissue is separated into three distinctcategories: smooth muscle 82 , which is found in the inner linings of organs; skeletal muscle 83 , which isfound attached to bone providing movement; and cardiac muscle 84 which is found in the heart 85 , allowingit to contract and pump blood throughout an organism.

1.4.3.1.2.1 Smooth muscle

Smooth muscle is unstriated (not striped) in appearance. The contraction of smooth muscle can be relativelyslow and often happens automatically without our conscious control. Some call it involuntary muscle. Itrarely becomes fatigued. Smooth muscle is found in the gut where it squeezes food along the intestines byperistalsis. It is found in the walls of our blood vessels where it can make the vessel wider or narrowerallowing more or less blood to �ow.

1.4.3.1.2.2 Skeletal muscle

Skeletal muscle or voluntary muscle appears striped or striated when seen through a microscope. This isdue to banding from the pattern of actin and myosin protein �laments in the muscle. Skeletal muscle isattached to the bones of the skeleton. Skeletal muscle fatigues or tires quickly. It also contracts quickly andis controlled by the conscious part of our brain. The biceps is made of skeletal muscle and when the musclecontracts it shortens bringing the lower arm upwards. Skeletal muscles are often arranged in antagonisticpairs where if one muscle moves a limb one way the other muscle will move it back.

1.4.3.1.2.3 Cardiac Muscle

Cardiac muscle appears striated like skeletal muscle except in cardiac muscle there are cross bridges or crossconnections linking muscle �bres together. Cardiac muscle is myogenic, in other words it generates its ownimpulse to contract from within itself. It is found only in the heart and it contracts rhythmically at a speeddictated by the brain without ever su�ering fatigue.

81http://www.hartnell.edu/tutorials/biology/tissues.html82http://en.wikipedia.org/wiki/Smooth_muscle83http://en.wikipedia.org/wiki/Skeletal_muscle84http://en.wikipedia.org/wiki/Cardiac_muscle85http://en.wikipedia.org/wiki/Heart

Available for free at Connexions <http://cnx.org/content/col11410/1.3>

35

1.4.3.1.3 Nerve tissue

Figure 1.64

Cells making up the central nervous system 86 and peripheral nervous system 87 are classi�ed as neuraltissue. In the central nervous system 88 , neural tissue forms the brain 89 and spinal cord 90 and, in theperipheral nervous system 91 forms the cranial nerves 92 and spinal nerves 93 , including the sensory andmotor neurons 94 . The function of nerve tissue is to transmit electrical messages around the body.

86http://en.wikipedia.org/wiki/Central_nervous_system87http://en.wikipedia.org/wiki/Peripheral_nervous_system88http://en.wikipedia.org/wiki/Central_nervous_system89http://en.wikipedia.org/wiki/Brain90http://en.wikipedia.org/wiki/Spinal_cord91http://en.wikipedia.org/wiki/Peripheral_nervous_system92http://en.wikipedia.org/wiki/Cranial_nerves93http://en.wikipedia.org/wiki/Spinal_nerves94http://en.wikipedia.org/wiki/Motor_neurons

Available for free at Connexions <http://cnx.org/content/col11410/1.3>

36CHAPTER 1. LIFE AT THE MOLECULAR, CELLULAR AND TISSUE

LEVEL

1.4.3.1.4 Three Types of Nerve Cells

1.4.3.1.4.1 Sensory neurones

Figure 1.65

Sensory nerve cells (or sensory neurones) carry impulses (electrical signals) from a receptor to the CNS(central nervous system). The cell body is located o� to one side of the axon as it enters a vertebra (one ofthe boney discs of the spine).

1.4.3.1.4.2 Motor neurones

1.4.3.1.4.3

Motor neurones carry impulses out from the CNS to e�ectors, instructing them to do something. In mostcases the e�ector is a muscle being told to contract but other e�ectors could be glands or colour cells(chromatophores). The cell body is at the head of a motor neurone in the vertebra and its tail or dendritesare attached to the muscle or other e�ector.

1.4.3.1.4.4 Relay or connector neurones

Relay neurones are very short especially compared to the other two types. They connect a sensory neuronewith a motor neurone across the grey matter region in the CNS inside the spine. The impulse travels fromthe cell body at the head end along the short axon to the dendrites.

Available for free at Connexions <http://cnx.org/content/col11410/1.3>

37

1.4.3.1.5 Connective tissue

Figure 1.66: Di�erent types of connective tissue http://www.hartnell.edu/tutorials/biology/tissues.html95

Connective tissues are made up of separate cells which are �oating in a matrix. Connective tissues are �broustissues. They are made up of cells separated by non-living material, which is called matrix 96 . Connectivetissues give shape to organs and holds them in place. Both bone and blood are examples of connective tissue.As the name implies, connective tissue serves a "connecting" function. It supports and binds other tissues.

95http://www.hartnell.edu/tutorials/biology/tissues.html96http://en.wikipedia.org/wiki/Extracellular_matrix

Available for free at Connexions <http://cnx.org/content/col11410/1.3>

38CHAPTER 1. LIFE AT THE MOLECULAR, CELLULAR AND TISSUE

LEVEL

1.4.3.1.5.1 Cartilage

Cartilage is a �exible connective tissue 97 found in many areas of the bodies of humans and other animals,including the joints between bones 98 , the rib cage 99 , the ear 100 , the nose 101 , the elbow, the knee,the ankle, the bronchial tubes 102 and the intervertebral discs 103 . It is not as hard and rigid as bone104 but is sti�er and less �exible than muscle 105 .

Figure 1.67

97http://en.wikipedia.org/wiki/Connective_tissue98http://en.wikipedia.org/wiki/Bone99http://en.wikipedia.org/wiki/Rib_cage

100http://en.wikipedia.org/wiki/Ear101http://en.wikipedia.org/wiki/Nose102http://en.wikipedia.org/wiki/Bronchial_tubes103http://en.wikipedia.org/wiki/Intervertebral_disc104http://en.wikipedia.org/wiki/Bone105http://en.wikipedia.org/wiki/Muscle

Available for free at Connexions <http://cnx.org/content/col11410/1.3>

39

Cartilage is composed of specialized cells called chondroblasts 106 that produce a large amount ofextracellular matrix 107 composed of collagen 108 �bres, abundant ground substance 109 and elastin 110

�bers. Chondroblasts that get caught in the matrix are called chondrocytes 111 . They lie in spaces, calledlacunae, with up to eight chondrocytes per lacuna. Cartilage is classi�ed in three types, elastic cartilage112 , hyaline cartilage 113 and �brocartilage 114 , which di�er in the relative amounts of these three maincomponents.

1.4.3.1.6 Blood

Blood is a tissue because it is made up of several types of cell functioning together. In fact blood is composedof red blood cells, white blood cells and platelets �oating in a yellow liquid called plasma.

Figure 1.68

106http://en.wikipedia.org/wiki/Chondroblast107http://en.wikipedia.org/wiki/Extracellular_matrix108http://en.wikipedia.org/wiki/Collagen109http://en.wikipedia.org/wiki/Ground_substance110http://en.wikipedia.org/wiki/Elastin111http://en.wikipedia.org/wiki/Chondrocyte112http://en.wikipedia.org/wiki/Elastic_cartilage113http://en.wikipedia.org/wiki/Hyaline_cartilage114http://en.wikipedia.org/wiki/Fibrocartilage

Available for free at Connexions <http://cnx.org/content/col11410/1.3>

40CHAPTER 1. LIFE AT THE MOLECULAR, CELLULAR AND TISSUE

LEVEL

1.4.3.1.6.1 Red blood cells

Red blood cells are shaped like biconcave discs (round discs with a dimple in the top and another in thebottom). Their job is to carry oxygen round the body form the lungs to the tissues. They contain a redpigment (coloured chemical) called haemoglobin which is brilliant at carrying oxgen.

1.4.3.1.6.2 White blood cells

White blood cells are slightly larger than red ones. They are more irregular in shape and their job is to �ghtpathogens (disease-causing organisms like some bacteria and viruses). Some engulf 9swallow up pathogens),some produce chemicals called antibodies to kill the pathogens while a third type release antitoxins, chemicalswhich neutralize the poisons made by the pathogens. When looked at under a microscope white blood cellsmay appear purple because a dye is used to stain them.

1.4.3.1.6.3 Platelets

Platelets are fragments of cells �oating in the plasma. They are important in clotting and stick togetherwhere a blood vessel is damaged to close the wound.

1.4.3.1.6.4 Plasma

Plasma is a yellow liquid in which the solid cells are suspended or �oating. It carries many importantchemicals around the body including the waste carbon dioxide from respiration, hormones, urea, and glucoseand also transports heat.

1.4.3.2 Structure and function

Table to show the relationship between structure and function in 4 animal tissues.

Tissue Example Structure Function Relationship

Epithelial tissue Lining of the airsacs in the lungs

Thin, sometimesone cell thick,often moist

Di�usion of gasesacross lining

Gases di�use bestacross thin, moistsurafces

Connective tissue Bone Composed of min-erals and organicmatter

Strength and �exi-bility

Strength is pro-vided by minerals,�exibility by or-ganic matter

Muscle tissue Heart muscle Contains �bres Pumping by con-traction

Fibres ratchet pasteach other to con-tract the muscle

Nerve tissue Optic nerve Long thin, insu-lated �bres

Carrying electricalmessages from eyeto brain

Long and insu-lated so electricalmessage is carriedquickly

Table 1.6

Available for free at Connexions <http://cnx.org/content/col11410/1.3>

41

1.4.3.3 Assignment 1 � Mammal tissues

MAMMALIAN TISSUES

Exercise 1.4.3.1Question 11.1 Answer the following questions based on the drawings below.

Figure 1.69

1.1 Provide labels for 1, 2, 3 & 4.(4)1.2 Which tissue (A or B) is found in the rib cage?(1)1.3 Which tissue (A or B) is found in the lining of the blood vessels?(1)Question 2The following diagram shows a blood smear.W

Available for free at Connexions <http://cnx.org/content/col11410/1.3>

42CHAPTER 1. LIFE AT THE MOLECULAR, CELLULAR AND TISSUE

LEVEL

Figure 1.70

rite down the number of the part that . . .2.1 contains haemoglobin2.2 is needed for the clotting of blood2.3 are phagocytic (engulf germs)2.4 transport nutrients2.5 transport oxygen2.6 distributes heat in the body[6]Question 3A scientist knows that as you move to higher altitudes (height above sea level), the air has less

oxygen. She therefore wanted to investigate whether altitude in�uences the number of red bloodcorpuscles in people's blood.

3.1 Write a hypothesis for the scientist's investigation. (2)3.2 Identify the:dependent variable (the variable to be tested)independent variable. (2)3.3 The scientist drew the following graph after she had �nished her investigation. Study it and

answer the questions based on it.

Available for free at Connexions <http://cnx.org/content/col11410/1.3>

43

Figure 1.71

3.4 What is the relationship between the number of red blood corpuscles and altitude? (2)3.5 Explain the reason for this relationship. (1)3.6 The Sharks Rugby Team (a rugby team from Durban) often get tired in the last twenty

minutes when the play the Blue Bulls (a team from Pretoria) in Gauteng.Explain why, in terms of the information from the graph. (3)3.7 Suggest how the Sharks could overcome this problem as described in the above question.

(2)3.8 The scientist lost the table from which she drew the graph above, Draw the table of �gures

and values to represent the graph above. (6)3.6 Do you think that the scientist took just one person's blood sample at altitude? Explain

your answer. (2)

[20]Assignment 2 - Practical and theory Test: Plant Tissues and PhotosynthesisTime: 90 Minutes (+10 minutes reading time)Marks: 70Instructions:Section A: practical work and answers should be given on the question paperSection B: theory and answers should be given on your own lined paper.

Exercise 1.4.3.2Section AAim: Testing for the presence of starchQuestion 1ApparatusYou are provided with the following equipment:a spotting tilemarking pensolutions A (starch solution) and B (sugar solution)

Available for free at Connexions <http://cnx.org/content/col11410/1.3>

44CHAPTER 1. LIFE AT THE MOLECULAR, CELLULAR AND TISSUE

LEVEL

leaf discs marked C and DIodine reagentDroppersForcepsThe leaf discs have been prepared for testing for the presence of starch .Describe the procedure ( only ) followed in this preparation. Outline 5 steps.(5)MethodsClearly mark your spotting tile A-D using the marker provided.Using the droppers (in the beakers) place a few drops of the solution A in well A and a few

drops of solution B in well B and the leaf discs (using the forceps) in C and in D in each ofthe marked areas.

Call your teacher to observe you apply iodine solution to each area A-D. Wait afew moments and then make observations. (5)

Draw up a table in the space below to �ll in your observations and conclusions .(10)Clean up your area by returning all apparatus to the trolley and wiping o� the

spotting tile with the cloth provided.Question 2Water plants are exposed to light for one minute at a time at di�erent temperatures. The

amount of oxygen given o� per minute at di�erent temperatures is shown graphically in Figure 1.Figure 1: Graph showing the oxygen given o� by a plant placed in light at di�erent temperatures

Figure 1.72

2.1 Which one of the following statements is the best summary of the results? Highlight yourchoice.(2)

i. Oxygen is formed in plants exposed to light.ii. A rise in temperature increases the production of oxygen.iii. The oxygen production is highest between 20ºC and 30ºC.iv. Temperature a�ects the rate of oxygen production.v. A rise in temperature increases the oxygen production to a certain maximum.

2.2 In the experiment what is the:Independent variable? (1)Dependent variable? (1)2.3 Design an experiment to show how you obtain the results shown in the graph. You need

only outline the method.(10)Section BQuestion 3Choose TWO of any of the following questions, number your choices very clearly:3.1 The strengthening substances, cellulose and lignin are found in various plant tissues. Name

any four of the plant tissue types and explain the speci�c role in each tissue mentioned.(8)

Available for free at Connexions <http://cnx.org/content/col11410/1.3>

45

3.2 Compare the structure of a guard cell and an ordinary epidermal cell.(8)3.3 Parenchyma is termed �packing tissue/storage/gaseous exchange tissue�. Discuss four fea-

tures of this tissue that enable it to ful�ll this role.(8)3.4 Soluble substances need to be transported around the plant body. Highlight 4 features of

phloem tissue that allow for e�cient transport.(8)[16]Question 4: Mini-Essay�Photosynthesis and food production�It is very important to understand the factors that a�ect the rate of photosynthesis when growing

crops that provide us with food. The greater the rate of photosynthesis of a plant, the greater itsrate of productivity. The productivity of a plant refers to how much extra dry mass it gains over aperiod of time. This extra dry mass is in the form of carbohydrates, proteins or fats that the plantmakes from the products of photosynthesis. A plant crop that is very productive will gain moredry mass over a period of time, and will therefore provide us with more food to eat. Farmers willalso get paid more if they produce a greater quantity of food. Plants, such as tomatoes, grown ingreenhouses (or growing tunnels) may grow better than those grown outside.

You need to write a motivation to a farmer where you convince him that the cost of a greenhouse(plant growing tunnel) will be recouped /earned with the greater pro�t that he will earn from hiscrops.

In your answer you need to consider:At least 4 structural features of the greenhouse that will enable the plant to grow better,while at the same time considering the factors necessary for a plant to photosynthesis most

e�ectively.[20]

Marking Rubric:

Assessment Crite-rion

Content 0-4Little referenceto green house fea-tures, minimal fac-tors mentioned

5-9Some green-house featuresmentioned, a fewfactors mentioned

10-12Allgreenhouse features mentioned,and at least 4 factors necessary for pho-tosynthesis mentioned

Skills 0-2Very little accu-racy, much irrele-vant data

3-4Some confu-sion, some linkage,some irrelevance

5Coherent, linked concepts, logical path-way correct

Presentation 1 no paragraphs,intro or conclu-sion; poor expres-sion

2Paragraphed butpoor intro andconclusion

3Good intro and conclusion, para-graphed; good biological expression

Table 1.7

1.4.4 1.4.4 Applications of indigenous knowledge and biotechnology115

1.4.4.1 Traditional and modern biotechnology

Traditional Biotechnology based on indigenous methods of fermentations is becoming a source of increasinginterest in the scienti�c community. Below is an example of the kind of interest being shown in traditionalmethods of biotechnology.

115This content is available online at <http://cnx.org/content/m43060/1.1/>.

Available for free at Connexions <http://cnx.org/content/col11410/1.3>

46CHAPTER 1. LIFE AT THE MOLECULAR, CELLULAR AND TISSUE

LEVEL

Figure 1.73

The above is the abstract from a scienti�c paper. The abstract provides a summary of the contents ofthe paper at the start of the published paper.

1.4.4.2 Medical biotechnology

In medicine modern biotechnology �nds promising applications in such areas as: drug production, phar-macogenomics (how a person's genes a�ects their response to drugs), gene therapy and genetic testing (orgenetic screening): techniques in molecular biology detect genetic diseases. To test the developing fetus forDown syndrome, Amniocentesis and chorionic villus sampling can be used.[2]

1.4.4.3 Cloning

Cloning is the process of arti�cially producing genetically identical individuals that occurs in nature whenorganisms such as bacteria, insects or plants reproduce asexually. Cloning in biotechnology refers to processesused to create copies of DNA fragments (molecular cloning), cells (cell cloning), or organisms.

Available for free at Connexions <http://cnx.org/content/col11410/1.3>

47

1.4.4.3.1 Dolly the sheep

Figure 1.74

Dolly was born 5 July 1996 to three mothers (one provided the egg, another the DNA and a third carriedthe cloned embryo to term). She was created using the technique of somatic cell nuclear transfer, where thecell nucleus from an adult cell is transferred into an unfertilised oocyte (developing egg cell) that has had itsnucleus removed. The hybrid cell is then stimulated to divide by an electric shock, and when it develops intoa blastocyst it is implanted in a surrogate mother. Dolly was the �rst clone produced from a cell taken froman adult mammal. The production of Dolly showed that genes in the nucleus of such a mature di�erentiatedsomatic cell are still capable of reverting back to an embryonic totipotent state, creating a cell that can thengo on to develop into any part of an animal. Dolly's existence was announced to the public on 22 February1997.

1.4.4.3.1.1 Activities and Links on Biotechnology

•http://www.pbs.org/wgbh/harvest/engineer/transgen.htmlTakes students on a step-by-step process showing them how to genetically engineer a plant and bacterium•http://www.biotechnologyonline.gov.au/biotechnologyonline/topitems/resour ces.htmlAustralian government resource for educators and learners on various aspects of biotechnology.

Available for free at Connexions <http://cnx.org/content/col11410/1.3>

48CHAPTER 1. LIFE AT THE MOLECULAR, CELLULAR AND TISSUE

LEVEL

•http://www.scienti�camerican.com/biotechnologyAll the latest articles on developments in biotechnology written in accessible language.•http://seedmagazine.com/content/tag/biotechnology/Discusses latest issues in biotechnology and their political, economic and cultural implications.

1.4.5 1.4.5 Organs116

1.4.5.1 Organs

1.4.5.1.1 Organs and tissues

An organ is a collection of tissues 117 joined in structural unit to serve a common function. Functionallyrelated organs often cooperate to form whole organ systems . Organs exist in all higher biological organisms,they are not restricted to animals, but can also be identi�ed in plants, e.g. the leaf is an organ in a plant asis the root, stem, �owers and fruits.

Figure 1.75

116This content is available online at <http://cnx.org/content/m43162/1.1/>.117http://en.wikipedia.org/wiki/Tissue_(biology)

Available for free at Connexions <http://cnx.org/content/col11410/1.3>

49

Cabbage by Christian Guthierhttp://www.fotopedia.com/items/�ickr-2671495796 118

1.4.5.1.2 Leaf Structure

The leaf is the main photosynthesising organ of a plant. The Leaf is an organ because it contains a numberof tissues including epidermis, xylem, phloem and parenchyma working together to make food for the plantin photosynthesis.

Figure 1.76: Diagram of a section through a leaf

1. Upper epidermis2. Palisade3. Air space4. Mesophyll5. Xylem6. Phloem7. Stoma

1.4.5.1.2.1 Leaf Adaptations

The upper epidermis is transparent to allow light to pass through to the chlorophyll-containing cells deeperin the leaf.

The palisade cells are tightly packed just underneath the upper epidermis. They are full of chloroplastsfor photosynthesis.

Intercellular spaces allow for di�usion and gaseous exchange.Spongy mesophyll contains chloroplasts for photosynthesis and air spaces to aid di�usion of gases.Xylem transports water to the cells, as water is a requirement of photosynthesis.Phloem transports the products of photosynthesis (starch etc.) away from the leaves.Stomata (controlled by guard cells) allow carbon dioxide to enter and oxygen to leave by di�usion for

photosynthesis.Video � (free, to be embedded) Plant Structure: http://www.neok12.com/php/watch.php?v=zX64544265547901594f5567&t=Plants

119

118http://www.fotopedia.com/items/�ickr-2671495796119http://www.neok12.com/php/watch.php?v=zX64544265547901594f5567&t=Plants

Available for free at Connexions <http://cnx.org/content/col11410/1.3>

50CHAPTER 1. LIFE AT THE MOLECULAR, CELLULAR AND TISSUE

LEVEL

Video � (free, to be embedded) Photosynthesis http://www.neok12.com/php/watch.php?v=zX4b7a504d047d5b6e660173&t=Plants120

1.4.5.1.3 Assignment 3 - Structure of a dicot leaf

A Label parts 1-7(7)B Write down the function and one adaptation of each part. (14)

Figure 1.77

120http://www.neok12.com/php/watch.php?v=zX4b7a504d047d5b6e660173&t=Plants

Available for free at Connexions <http://cnx.org/content/col11410/1.3>

51

1.4.6 1.4.6 Summary121

1.4.6.1 Summary

A tissue is a group of cells grouped together and working together.An organ is group of tissues grouped together and working together.Plant tissues include xylem, phloem, collenchyma, sclerenchyma, epidermis and meristematic tissue.Animal tissues include epithelial tissue, connective tissue, muscle tissue and nerve tissue.Plant and animal tissues are used in traditional technology, medical technology and cloning processes.The leaf is an example of a plant organ and its structure is adapted to its role in photosynthesis.

121This content is available online at <http://cnx.org/content/m43064/1.1/>.

Available for free at Connexions <http://cnx.org/content/col11410/1.3>

52CHAPTER 1. LIFE AT THE MOLECULAR, CELLULAR AND TISSUE

LEVEL

Available for free at Connexions <http://cnx.org/content/col11410/1.3>

Chapter 2

Life processes in plants and animals

2.1 Support and transport systems in plants

2.1.1 2.1.1 - Anatomy of dicotyledenous plants1

2.1.1.1 Anatomy of Dicotyledonous Plants

Plant structure:http://www.emc.maricopa.edu/faculty/farabee/biobk/biobookplantanat.htmlPlants are made up of roots, stems, leaves and �owers. The function of the root is to hold the plant

�rmly in the ground as well as to absorb water from the soil. The function of the stem is to transport thefood made by the leaf to the rest of the plant as well as to hold the plant upright. The main function of theleaves is to photosynthesize (make food).

1This content is available online at <http://cnx.org/content/m43142/1.1/>.

Available for free at Connexions <http://cnx.org/content/col11410/1.3>

53

54 CHAPTER 2. LIFE PROCESSES IN PLANTS AND ANIMALS

Figure 2.1

2.1.1.1.1 Di�erences between monocotyledonous and dicotyledonous plants

Traditionally the �owering plants (angiosperms) are divided into two groups, monocotyledons (monocots)and dicotyledons (dicots). Monocots are the grass and grass-like �owering plants (e.g. maize), while dicotsinclude the rest of the �owering plants (e.g. bean). The embryos of monocots have only a single (mono-)cotyledon (the �rst leaf) while the embryos of dicots have paired (di-) cotyledons. Other di�erences betweenmonocots and dicots are shown in the table below. Monocots have long narrow leaves with parallel veinswhile dicots have broad leaves with net-like veins. In monocots the �ower parts are in multiples of three whilein dicots they are in multiples of four or �ve. In monocots the vascular bundles of the stem are scatteredwhile in dicots there is a ring of vascular bundles. Monocots grains have one furrow or pore while dicotgrains have three furrows or pores. Monocots have adventitious roots while in dicots the roots develop froma radicle.

Available for free at Connexions <http://cnx.org/content/col11410/1.3>

55

Figure 2.2

2.1.1.1.2 Plant structure

Most plants are stationary which means that they cannot move from place to place. Some plants growreally tall in order to obtain sunlight. They need to stand tall and erect and therefore need to supportthemselves. They have tissues present in almost all parts of their body e. g. roots, stems, branches, leaves.These supporting tissues keep the stem �rm and other parts such as leaves in a favourable position forphotosynthesis to occur as e�ciently as possible.

Refer to Unit 1.4 for functions of the di�erent tissues found in roots, stems and leaves.

2.1.1.1.3 Dicotyledonous Root

External structure of the dicot root

Available for free at Connexions <http://cnx.org/content/col11410/1.3>

56 CHAPTER 2. LIFE PROCESSES IN PLANTS AND ANIMALS

Figure 2.3

This diagram shows the external structure of a dicot root.

• Root cap protects the tip of the root and it is slimy to facilitate movement through the soil as the rootgrows.

• Above the root cap is the meristematic region where cells divide continuously by mitosis to producenew cells.

• Cells enlarge in size in the region of elongation. This results in the root growing in length.• Thousands of tiny root hairs are found in the root hair region. The function of this region is to absorb

water and dissolved mineral salts from the soil.• The root grows thicker and may produce lateral roots in the mature region.

Available for free at Connexions <http://cnx.org/content/col11410/1.3>

57

2.1.1.1.3.1 Internal structure of the dicot root

Figure 2.4

• No waterproof cuticle in the root as this would hinder the absorption of water.• The epidermis is a single layer of cells on the outside that protects the inner tissues. Some epidermal

cells are specialized to form root hair cells. These absorb water and dissolved mineral salts.• The cortex consists of parenchyma cells. These cells are large to store water and food. They also

facilitate the movement of water from the root hair cells on the outside to the xylem on the inside.• The endodermis is lined with Casparian strips, distinctive bands made of a water-impermeable, waxy

substance called suberin, that prevents water and minerals from passively seeping between the cellsand thus forces water to enter through the cell membranes of the endodermal cells in order to enterthe stele (vascular cylinder).

• The stele consists of the:

· Pericycle (responsible for forming lateral roots)· Xylem (responsible for transporting water and mineral salts to the stem)· Phloem (responsible for transporting food from the leaves to the roots)

Available for free at Connexions <http://cnx.org/content/col11410/1.3>

58 CHAPTER 2. LIFE PROCESSES IN PLANTS AND ANIMALS

2.1.1.1.3.2 Movement of water through the dicotyledonous root

Figure 2.5

This diagram shows the movement of water through the root

• Water is found in the spaces between the soil particles. Water enters through the cell wall and cellmembrane of the root hair cell by osmosis. Water �lls the vacuole of the root hair cell.

• Water can now move across the parenchyma cells of the cortex in two ways:• Most of the water passes along the cell walls of the parenchyma cells by di�usion. Movement of

water and solutes between the intercellular spaces without crossing the plasma membrane is known asapoplastic movement.

• Some of the water passes from the vacuole of one parenchyma cell to the vacuole of the next cell byosmosis. Movement of water and solutes through the cells is known as symplastic movement.

• The water must pass through the endodermis to enter the xylem.• Once water is in the xylem of the root, it will pass up the xylem of the stem.

Transpiration and movement of water: http://www.phschool.com/science/biology_place/l abbench/lab9/xylem.html

This website shows a diagram of how water moves up through the plant.http://www.neok12.com/Plants.htmThis video shows plant transport and provides some interactive quiz games.

2.1.1.1.3.3 Investigation: Water uptake by roots

Aim : To measure the uptake of water by rootsApparatus

• Plastic 2 litre Coke bottle• water• soil• scissors• measuring scale• tree or plant cuttings

Available for free at Connexions <http://cnx.org/content/col11410/1.3>

59

• ruler

Method

1. Remove the label from the Coke bottle. Cut the top of the bottle o� 20cm from the bottom of thebottle. Poke holes in the bottom of the bottle for drainage. Hold the plant cutting in the containerwhile you �ll the container with soil. Leave one or two leaf buds about 5cm above the soil.

2. Weigh the container to get the total weight of the bottle, soil and plant.3. Water the plant with enough water so that it starts to run out of the bottom of the bottle through the

holes.4. Weigh the container again after the water has stopped running out and subtract the total weight in

step 2 from the weight in step 4 to get the weight of the water. 1 litre of water is equal to 1 kilogramof water, therefore you can work out exactly how much water is in the container.

5. Set the containers by the window where they will receive enough sunlight. Wait for the leaves to startgrowing (1-3 weeks).

6. After the leaves are growing, weigh the containers every 1-3 days for 3 weeks. Subtract the new weightfrom the weight calculated in step 4. The new number is the amount of water that the plant is using.Water the plant as necessary (when the soil becomes dry), but remember to reweigh the containerwhen you add more water so that you can still tell how much water the plant is taking up.

7. Draw a graph of the amount of water the plant is using. The X-axis should show the number of dayssince the beginning of the experiment and the Y-axis should show the amount of water that the planthas used.

Question on investigation � Surely as the plant grows it forms new leaves etc which have mass? How do wesubtract the mass of the new leaves to prevent

this making the mass of the entire container inaccurate in terms of how much mass isjust water usage? - I also wondered this but obtained the investigation elsewhere so don' t know. I

thought perhaps the plant does not put on that much mass compared to the mass of the water so it becomesinsigni�cant

2.1.1.1.4 Dicotyledonous stem

• Leaves develop from the nodes.• The sections of stem between the nodes are called internodes.• An axillary bud is often found at the node. These forms lateral branches.• A terminal bud is found at the tip of the stem and allows the stem to increase in length.

Available for free at Connexions <http://cnx.org/content/col11410/1.3>

60 CHAPTER 2. LIFE PROCESSES IN PLANTS AND ANIMALS

Figure 2.6

Available for free at Connexions <http://cnx.org/content/col11410/1.3>

61

2.1.1.1.4.1 Internal structure of the dicotyledonous stem

Figure 2.7

This diagram of a cross section shows the internal structure of a young dicot stem

• A waterproof cuticle is found on the outside of the epidermis to prevent water loss.• The epidermis consists of a single layer of cells to protect the underlying tissue.• The cortex is made up of parenchyma cells that stores water and food.• The vascular bundles are arranged in a ring in the medulla and are surrounded by non-living scle-

renchyma cells for strengthening and support.• Each vascular bundle contains the following:

· Cambium (contains meristematic cells that divide to widen the stem)· Phloem (transports food from leaves to the roots)· Xylem (transports water from the roots to the stem)

http://bcs.whfreeman.com/thelifewire/content/ch p36/36020.htmlThis is a link to an online tutorial about phloem, xylem and pressure �ow.

2.1.1.1.4.2 Movement of water up the stem

• Water moves up the xylem from the roots to the leaves.• Adaptations of xylem for transporting water:

· Long, elongated tubes joined end-to-end without any cross-walls, forming good conducting tubes.· The cell walls are thickened with lignin for support (annual or spiral thickening) so that they donot collapse due to the upward pull of water

· Pitted vessels and tracheids allow for lateral movement of water into neighbouring xylem vessels.· Cells are dead, so there is no obstruction to water transport

Available for free at Connexions <http://cnx.org/content/col11410/1.3>

62 CHAPTER 2. LIFE PROCESSES IN PLANTS AND ANIMALS

Figure 2.8

Diagram of xylemThree forces are responsible for the movement of water up the xylem � capillarity, root pressure and

transpiration suction force.

• Capillarity involves forces of cohesion (forces of attraction between water molecules) and adhesion(forces of attraction between water molecules and the sides of the xylem vessels). Because the xylem'slumen (opening) is so tiny, water will move up by capillary. However, this force is weak and its role inmoving water up the stem is small.

• Root pressure is a force that pushes water up the xylem. As water enters the root by osmosis, it pushesthe water that is already in the xylem of the stem upwards.

• Transpiration suction force is a very important force that pulls water up the xylem of the stem. Aswater evaporates from the stomata of the leaves during transpiration, it creates a sucking force thatwill pull the water up the xylem.

2.1.1.1.4.3 Investigation: plant tissue anatomy (root and stem)

Aim: To examine the structure of the root and stem

Available for free at Connexions <http://cnx.org/content/col11410/1.3>

63

Apparatus

• Scalpel or knife• Celery stalk (stem)• Carrot (root)• Glass slide• Iodine solution (Stain) or water• Cover slip• Dissecting needle or tweezers• Paper and pencil

Method1. Cut a very thin slice (cross section) from the middle of the celery stem or the carrot root.2. Place this section on a glass slide.3. Cover the specimen with iodine solution in order to stain it. This makes it more visible under the

microscope. The specimen can also be placed on a drop of water if iodine is not available.4. Cover the specimen by carefully lowering the cover slip onto it with a dissecting needle or tweezers.

Take care not to trap any air bubbles.This link gives information about making a wet mount microscope slide and shows an instructional video.http://www.microbehunter.com/2010/08/13/making-a-wet-mount-microscope-slide/5. Call your teacher.6. Switch on the microscope making sure the lowest objective is in position (the 4x objective).7. Place your slide on the stage.8. Focus the image under the 4x objective (lowest objective) and view the structure of the celery stem.

Switch to the 10x objective to look a little more closely. To see amazing details of the structure of planttissue, use the 40x objective and the slide, carefully observing all of the parts and di�erent cells.

9. Once you are able to see cells,10. Call your teacher.11. Make a biological drawing of your specimen as viewed under the microscope. Take note of the

magni�cation and draw a scale bar. Label your diagram according to the tissues you have learnt about.Variation: Be creative and try using your favourite vegetables! Which vegetables are roots, stems and

leaves?To prepare a slide:

Available for free at Connexions <http://cnx.org/content/col11410/1.3>

64 CHAPTER 2. LIFE PROCESSES IN PLANTS AND ANIMALS

Figure 2.9

Place the sample in the centre of the slide. Add a drop of iodine or water on top of the sample. Placethe cover slip next to the droplet as shown in the diagram.

Lower the coverslip into place with tweezers. As you lower the coverslip downwards, the drop will spreadoutward and suspend the sample between the slide and the coverslip.

(Diagrams from http://www.ehow.com/how_5164819_prepare-wet-mount-slide.html )

2.1.1.1.4.4 Investigation: water uptake by stem

Aim: To examine the uptake of water by the stemApparatus:

• Water• Food colouring dye (available at supermarket)• White �ower on a stem, e.g. Impatients, carnation or chrysanthemum• Scissors• Two jars, cups or measuring cylinders• Plastic tray• Sticky tape

Method:Before starting this experiment, try to guess how the dye might move up the stem into the �ower.1. Fill one jar with plain water, and one with water containing several drops of food colouring dye.2. Take the �ower and carefully cut the stem lengthwise, either part way up the stem or right up to the

base of the �ower (try both � the results will be di�erent!)

Available for free at Connexions <http://cnx.org/content/col11410/1.3>

65

3. Put one half of the stem into the jar containing plain water and one half of the stem into the jarcontaining food colouring dye. To make it easier to insert the stalks without breaking them, it helps towedge paper underneath the jars so that you can tilt them towards each other. Tape the jars or cylindersdown onto a tray so that they do not fall over.

4. Observe the �owers after a few hours and the next day, and note where the dye ends up in the�owerhead. You can leave the �owers up to a week but be sure to make sure that they have enough water.

Variation: Instead of using one cylinder with water and one with food dye, use two di�erent colourfood dyes (e.g. blue and red). At �rst the �ower will show two separate colours, but as time goes by thewhole �ower will show both dyes. This is because water can move sideways between xylem vessels throughopenings along their length. The ability of water to move sideways between vessels is useful for when airbecomes trapped in a vessel, causing a blockage. If you cut the stem right up to the base of the �ower, thiswill limit movement between the xylem vessels.

Variation: Try using celery stalks with leaves. Cut open the celery stalk (cross-section) and you willsee that the little holes inside are coloured � these are the vessels.

An example of this experiment with photographs can be foundat: http://www.practicalbiology.org/areas/intermediate/cells-to-systems/transport-in-plants/investigating-transport-systems-in-a-�owering-plant,70,EXP.html

2.1.1.1.5 Secondary growth

2.1.1.1.5.1 Meristematic Tissue

• Meristematic tissue consists of small cells that are unspecialized. These cells divide by mitosis to formnew cells that can di�erentiate (undergo changes in their structure) and can become specialized tissue(e.g. xylem, phloem, epidermal cells)

• Primary meristematic tissue is found in the tips of roots, stems and buds. When it divides new cellsare produced which causes the plant to grow longer. This is referred to as primary growth.

• Secondary meristematic tissue originates from permanent tissue, usually parenchyma tissue whichdivides by mitosis. Cambium is secondary meristematic tissue that is found in roots and stems. Whenthese cells divide by mitosis it results in the plant becoming wider. This is called secondary growth.

• Every growing season the stem of a plant increases in width � this is known as secondary thickening.• Towards the end of the �rst year of growth, the parenchyma cells between the vascular bundles become

meristematic and link up with the cambium tissue to form a cambium ring.• The cells in the cambium ring start dividing to form secondary phloem (on the outside) and secondary

xylem (on the inside).• Each year another ring of secondary phloem and secondary xylem is formed, making the stem grow

wider.• It is not possible to see the layers of secondary phloem but the secondary xylem are visible. These

form rings called annular rings which can be used to work out the age of a plant.• As new rings are formed each year, the older rings are pushed inward and the xylem vessels collapse

due to the pressure. The wood in the centre becomes denser and harder than the wood at the surfaceand is called heartwood.

• The youngest annual rings found on the outside serve its function of transporting water. This wood isnot as dense and is called sapwood.

• The light-coloured rings are called spring wood. They are formed during spring and summer when thegrowing conditions are favourable. The rings are therefore relatively thick and light in colour as thexylem cell walls are thin.

• The dark-coloured rings are called autumn wood. They are formed during autumn and winter whenthe growing conditions are unfavourable. The rings are therefore relatively thin and dark in colour asthe xylem cell walls are thick.

http://www.emc.maricopa.edu/faculty/farabee/biobk/biobookplantanat.htmlThis website provides information on plant structure and support.

Available for free at Connexions <http://cnx.org/content/col11410/1.3>

66 CHAPTER 2. LIFE PROCESSES IN PLANTS AND ANIMALS

Figure 2.10

This diagram shows the process of secondary thickening in stems

Available for free at Connexions <http://cnx.org/content/col11410/1.3>

67

Figure 2.11

This diagrams shows the annual rings of a tree trunk

2.1.1.1.5.2 Investigation - Tree rings and climate change

Every year a tree forms a new layer of xylem around the trunk. This forms tree rings, which are visible ascircles in a cross section of a tree that has been cut down. Each tree ring, or wood layer, consists of twocolours of wood; light wood that grows in spring and summer and dark wood that grows in autumn andwinter. Tree rings can be counted to give you a rough estimation of the age of a tree. Occasionally a treewill form many rings in one year or miss forming rings in a year. The width of tree rings is greater in yearswhere good growing conditions occur. In years with droughts or low temperatures, the trees will producesmaller rings. Therefore, by looking at the tree rings you can get an idea of the weather a�ecting a tree ina particular year. Scientists can use this information to help determine the weather patterns of the past aswell as events such as forest �res, earthquakes and volcanic eruptions. The study of past events using thegrowth rings of trees is known as dendrochronology (�dendros� = tree, �chronos� = time).

Aim: to observe annual tree rings to assess age and climatic conditions1. Find a cut or fallen tree, and count the tree rings, starting with the innermost ring. Measure the

width of each ring using a ruler, or make a note of whether a ring is narrow or wide. Make a note of anyscars caused by events such as �res or pests.

2. Draw a bar graph showing the width of your tree rings for every year of the tree's life.

Available for free at Connexions <http://cnx.org/content/col11410/1.3>

68 CHAPTER 2. LIFE PROCESSES IN PLANTS AND ANIMALS

3. How old is your tree? What can you say about the climatic conditions throughout the life of yourtree?

http://www.classzone.com/books/earth_science/terc/content/investigations/es2905/es2905page01.cfm

This is a link to an online tutorial about counting tree rings.http://www.arborday.org/kids/carly/lifeofatree/This is a link to a great cartoon video about the di�erent tissue layers in trees (xylem, phloem, etc) and

the formation of tree rings.http://www.worsleyschool.net/science/�les/tree/rings.htmlThis is a link to a good website about tissue layers in trees.

2.1.1.1.6 Economic importance of plant support tissues

Plant support tissue supplies with two important resources namely wood and �bre . Xylem is a sourceof wood and the sclerenchyma is a source of �bre.

Of course in order to obtain wood and �bre we need to cut down numerous trees. This is called defor-estation . Deforestation has escalated in the recent years due to the growing need for wood.

ActivityCollect data showing the area covered by forests in the years 1990 and 2010. Find this data for the

following countries: South Africa, Europe, Asia, North and Central America and South America.Divide the class into two teams. One group will argue the need for us to cut down trees and the other

will be responsible for convincing us that deforestation must be reduced dramatically.

2.1.1.1.7 INDIGENOUS KNOWLEDGE

Making paper, �ax, cotton, sisal. Traditional use of �brous plants by san bushman using sansevaria to makerope etc. thatching. Can we please get help with this?

2.1.1.1.8 Dicotyledonous leaf

2.1.1.1.8.1 Internal structure of the dicotyledonous leaf

Refer to chapter 1 to remind yourselves of the internal structure of a dicotyledonous leaf.

Available for free at Connexions <http://cnx.org/content/col11410/1.3>

69

Figure 2.12

This diagram shows the movement of water through a dicot leaf.

2.1.2 2.1.2 Transpiration2

2.1.2.1 Transpiration

Transpiration is the evaporation of water from the leaves of plants. Water is lost from the leaf throughspecial pores called stomata. Stomata are found on both surfaces of the leaf but there are usually more onthe ventral (lower surface ) of the leaf. This is to reduce the amount of transpiration that will occur becausethe top of the leaf is exposed to more sunlight than the bottom.

http://education.uoit.ca/lordec/ID_LORDEC/transpiration_pull/ 3

This interactive website explains transpiration pull. Plants use the process of transpiration pull to movewater from the soil up into the leaves.

Heat from the environment causes the water in the sub-stomatal air spaces to evaporate out of thestomata. This process is called transpiration.

Transpiration is therefore de�ned as the loss of water vapour from the leaves of a plant.Transpiration only occurs during the day when the stomata are open. At night the stomata are closed.

2This content is available online at <http://cnx.org/content/m43163/1.1/>.3http://education.uoit.ca/lordec/ID_LORDEC/transpiration_pull/

Available for free at Connexions <http://cnx.org/content/col11410/1.3>

70 CHAPTER 2. LIFE PROCESSES IN PLANTS AND ANIMALS

Excess water di�uses into the sub-stomatal air spaces to replace that which has been lost from the leavesWater di�uses from the xylem of the leaf into surrounding mesophyll cells.Water circulates to supply plants with their water requirements.Water is pulled from the xylem of the stem to the xylem of the leaves. The xylem is found in the veins

of the leaf.

2.1.2.1.1 Rate of transpiration

This increases in conditions of . . .

• High light intensity (bright sunlight)• Increased temperatures (hot weather)• Wind• Low humidity (dry conditions)

Light Plants transpire more rapidly in the light than in the dark. This is largely because light stimulatesthe opening of the stomata. Light also speeds up transpiration by warming the leaf.

Temperature Plants transpire more rapidly at higher temperatures because water evaporates morerapidly as the temperature rises due to the increased kinetic energy of the water molecules. At 30 ◦C, a leafmay transpire three times as fast as it does at 20 ◦C.

Wind When there is no breeze, the air surrounding a leaf becomes increasingly humid thus reducingthe rate of transpiration. When a breeze is present, the humid air is carried away and replaced by drier air.So a steep di�usion gradient is maintained.

Humidity The rate of di�usion of any substance increases as the di�erence in concentration of thesubstances in the two regions increases. When the surrounding air is dry, di�usion of water out of the leafgoes on more rapidly.

Soil water A plant cannot continue to transpire rapidly if its water loss is not made up by replacementfrom the soil. When absorption of water by the roots fails to keep up with the rate of transpiration, lossof turgor occurs, and the stomata close. This immediately reduces the rate of transpiration (as well as ofphotosynthesis). If the loss of turgor extends to the rest of the leaf and stem, the plant wilts.

The volume of water lost in transpiration can be very high. It has been estimated that over the growingseason, one acre of corn (maize) plants may transpire 1.5 million litres of water. As liquid water, this wouldcover the �eld with a lake 38 cm deep. An acre of forest probably does even better.

This diagram shows a potometer which is used to measure the rate of transpiration. As the leafy twigtranspires, the air bubble moves to the left. The quicker the air bubble moves the faster the leafy twig istranspiring.

Available for free at Connexions <http://cnx.org/content/col11410/1.3>

71

Figure 2.13

Suggestion: Can we have agraph relating totranspiration rate over time?Also add an arrow to showmovement of the bubbleTOWARDS the plant forgreater clarity. [clark]The diagram below shows a summary of the movement of water from the roots to the leaf.

Figure 2.14

2.1.2.1.2 Why do plants need water?

Plants need water to maintain turgor pressure. This helps provide support for the plant as when a cellabsorbs water the cell membrane pushes against the cell wall. The cell is now turgid. If there isn't enoughwater in the plant the membrane moves away from the cell wall and the cell is now �accid. This is when a

Available for free at Connexions <http://cnx.org/content/col11410/1.3>

72 CHAPTER 2. LIFE PROCESSES IN PLANTS AND ANIMALS

plant begins to wilt and it will eventually die. Wilting is the loss of rigidity of the non-woody parts of plantsand occurs when the turgor pressure falls towards zero. Lower water availability can results from droughtconditions, high salinity, saturated soil conditions, low temperatures or bacterial or fungal infections thata�ect the vascular system of the plant,

When the environment is extremely humid (moist) the rate of transpiration is very low. In some plants,the leaves secrete water onto the surface of the leaves through specialised pores called hydathodes. Hy-dathodes are the open ends of xylem vessels at the edges of the leaves in certain species of plants such asstrawberries and some grasses.

2.1.2.1.2.1 Investigation � the e�ect of environmental conditions on transpiration rate (usinga simple potometer)

A potometer measures the rate of transpiration by measuring the movement of water into a plant. Thefollowing experiment uses a simple hand made photometer.

Aim: To assess the e�ect of di�erent environmental conditions (e.g. temperature) on transpirationrate.

Apparatus

• a drinking straw• a soft green leafy shoot• Vaseline• Marking pen• Play dough / putti• Plastic bag• Elastic band• Ruler

MethodPerform the following steps under water

1. Cut the stem of the leafy shoot under water .2. Test to make sure the stem of the leafy twig will �t snug tightly into the top of the straw.3. Remove the leafy shoot from the straw and set aside.4. Fill the straw with water. Place your �nger over one end of the straw to stop the water from running

out.5. Put the leafy shoot into the open end and seal it with play dough while removing it from water (KEEP

FINGER ON THE STRAW!)

Perform the following steps above water

1. Seal with Vaseline. Make sure it is air tight and water tight � if not, all the water will run out whenyou take your �nger o� the straw.

2. Mark the water level on the straw.3. Place your photometer under one of the following conditions for one hour:

• as is, in a warm, sunny place (no wind)• as is, in a warm, windy place• with a plastic bag tied around the leaf, in a warm, sunny place.• A shady place

1. After an hour: use the marking pen to mark the change in water level on the straw.2. Measure the distance the water moves.

Available for free at Connexions <http://cnx.org/content/col11410/1.3>

73

Results

• Draw a table and record the class' results.• Plot a bar graph to compare the distances the water moved in the di�erent straws.

Discussion

• Why is it important to cut the stem under water?• What does the water movement in the straw indicate?• Which four external environmental factors are you investigating?• Under which condition is water loss from the leaf the greatest?

Conclusion

• What can you conclude from this investigation?• Give two ways in which you can improve your experimental results.

More information about potometer experiments can be found on the following websites:http://www.practicalbiology.org/areas/advanced/exchange-of-materials /transpiration-in-

plants/measuring-rate-of-water-uptake-by-a-plant-shoot- using-a-potometer,62,EXP.html 4

http://www.practicalbiology.org/areas/advanced/exchange-of-materials /transpiration-in-plants/ 5

2.1.3 2.1.3 - Movement of manufactured food6

2.1.3.1 Movement of manufactured food

Plants use carbon dioxide and water to manufacture glucose and oxygen is the waste product. Sunlight andenzymes are necessary for photosynthesis to occur. Once the food is manufactured in the leaves it needs tobe distributed to the entire plant so that the glucose can be used by each cell for respiration (manufactureenergy).

Sunlight and enzymeswater + carbon dioxide ↔ glucose (carbohydrates) + oxygenThe glucose is manufactured mainly in the palisade cells and then passes into the phloem. Transport

of food material from leaves to other parts of the plant is called translocation. This food may be stored inroots, stems or fruit.

Read more: Anatomy of Plants - Biology Encyclopedia - cells, body,function, system, di�erent, organs, hormone, structure, types, membranehttp://www.biologyreference.com/A-Ar/Anatomy-of-Plants.html#ixzz1an9JO8yK

Phloem tissue is made up of two di�erent types of cells which are sieve tubes and companion cells. Sievetubes are the main conducting cells. These cells look like a sieve and phloem sap moves from cell to cellthough the phloem walls. Unlike cells of the xylem, sieve tubes are alive at functional maturity, but donot have nuclei. Companion cells have nuclei and are closely associated with sieve tubes. Companion cellscarry out all the cellular functions of the sieve tubes. The cytoplasm of sieve tubes and companion cellsis connected through numerous channels called plasmodesmata. These cytoplasmic connections allow thecompanion cells to regulate the content and activity of the sieve tube cytoplasm. The companion cells alsohelp load the sieve tube with sugar and the other metabolic products that they transport throughout theplant.

4http://www.practicalbiology.org/areas/advanced/exchange-of-materials/transpiration-in-plants/measuring-rate-of-water-uptake-by-a-plant-shoot-using-a-potometer,62,EXP.html

5http://www.practicalbiology.org/areas/advanced/exchange-of-materials/transpiration-in-plants/6This content is available online at <http://cnx.org/content/m43091/1.1/>.

Available for free at Connexions <http://cnx.org/content/col11410/1.3>

74 CHAPTER 2. LIFE PROCESSES IN PLANTS AND ANIMALS

Figure 2.15

Aphids feeding on phloem sap

Available for free at Connexions <http://cnx.org/content/col11410/1.3>

75

2.1.4 2.1.4 Summary7

2.1.4.1 Summary

Plants are made up of roots, stems, leaves and �owers. The various types of tissues found within plantswork together to ful�l functions such as support and transport of water and nutrients. In this chapter youhave learnt about the structure of roots and stems, and found out how they function. Roots consist ofa protective epidermis, a cortex to store food and water, and an endodermis surrounding an inner stele,which is composed of vascular tissue important in transport. Xylem is responsible for transporting waterand minerals up from the roots to the stem, while phloem is responsible for transporting food (which theplant has made by photosynthesis) from the leaves to the roots. Similarly, a stem consists of a waterproofcuticle, a protective epidermis, a cortex for storage, and contains vascular bundles arranged in a ring (indicotyledonous plants) or scattered throughout the stem (in monocotyledonous plants). The most importantforce that causes the movement of water through the xylem, from the roots up to the stem and leaves, istranspiration. As water evaporates from the stomata of the leaves during transpiration, it creates a suctionforce that pulls the water up through the roots. Hot, bright, dry and windy conditions can all lead to anincrease in the rate of transpiration. If plants lose too much water and do not replace it, this causes a loss ofturgor pressure, and the plant will wilt and eventually die. Water is also required for photosynthesis, whichis the process whereby plants use water, carbon dioxide and the energy from sunlight to make glucose, andrelease oxygen as a by-product. Glucose is manufactured mainly in the palisade cells of the leaves, fromwhere it passes into the phloem and is transported to the rest of the plant. This food may be stored in theroots, stems or fruit.

2.2 Support systems in animals

2.2.1 2.2.1 Skeletons8

2.2.1.1 Skeletons

Skeletons in animals:http://www.cli�snotes.com/WileyCDA/Cli�sReviewTopic/Skeletons-in- Animals.topicArticleId-

8741,articleId-8716.html 9

Skeleton � interactive animationhttp://www.medtropolis.com/VBody.asp 10

2.2.1.1.1 Overview

• The word support means to hold upright or to give strength• Aquatic organisms get support from water. Plants and animals living on land require stronger support

systems as air gives their bodies little support against the force of gravity.• Animals have skeletons that support their bodies and enable movement to occur.• In animals movement is brought about by the contraction or relaxation of muscles. In order to bring

about movement muscles need a medium (�uid) or solid structure against which the force of contractioncan be applied. A skeletal system provides the resistance needed to cause movement.

• Animals are able to move from one point to another to look for food, shelter and mates.• The simplest invertebrates have specialised cells and tissues to assist them to move to and from stimuli.• Skeletons also have a protective function in that they cushion vital organs thereby preventing or limiting

damage.

7This content is available online at <http://cnx.org/content/m43074/1.1/>.8This content is available online at <http://cnx.org/content/m43098/1.1/>.9http://www.cli�snotes.com/WileyCDA/Cli�sReviewTopic/Skeletons-in-Animals.topicArticleId-8741,articleId-8716.html

10http://www.medtropolis.com/VBody.asp

Available for free at Connexions <http://cnx.org/content/col11410/1.3>

76 CHAPTER 2. LIFE PROCESSES IN PLANTS AND ANIMALS

2.2.1.1.2 Hydrostatic skeleton

Figure 2.16

• It consists of a �uid-�lled cavity enclosed by the muscles of the body wall• The �uid presses against the muscles, that contract against the pressure of the �uid• So, a combination of the pressure of the �uid and the contracting muscles, can alter the shape of the

animal and allows for movement• If the body is segmented the pressure of the �uid is localised in a few segments at a time.• Occurs in �atworms, round worms, earthworms star�sh, slugs etc.• Note that star�sh and other Echinoderms have an outer skeleton of calcareous ossicles or spicules for

protection . This outer skeleton encloses a water vascular system with tube feet that are moved by�uid pressure changes i.e serves as a hydrostatic skeleton which controls movement.

2.2.1.1.2.1 Advantages:

• allow the animal to move in a more �exible manner• �uid cavity stimulates circulation in the animal• allows for change of shape e.g. earthworm

Available for free at Connexions <http://cnx.org/content/col11410/1.3>

77

Figure 2.17

2.2.1.1.2.2 Disadvantages:

• dehydration will a�ect the skeleton directly and the ability of the animal to move because of the lossof shape

• does NOT provide protection for the internal organs• does not allow for rapid movement.

2.2.1.1.3 Exoskeleton

Figure 2.18

• This forms the outer covering of the animal• The skeleton is made of a substance called chitin, secreted by the epidermis• It is con�ned to insects, spiders, scorpions, crabs etc all of which belong to the Arthropod group

(jointed legged animals• The abdomen is soft and attached to the thorax• The exoskeleton acts as a hard outer covering to animals and is made up of a series of plates or tubes.

Available for free at Connexions <http://cnx.org/content/col11410/1.3>

78 CHAPTER 2. LIFE PROCESSES IN PLANTS AND ANIMALS

• Muscles are attached to the inside of the exoskeleton which provides the resistance needed for muscleaction.

2.2.1.1.3.1 Advantages:

• forms the point of attachment of internal muscles needed for locomotion and �ight• supports and protects the delicate inner parts of the animal• prevents desiccation (drying out) on land• has a low density and is therefore lightweight, to allow for �ight• mouthparts can be modi�ed for biting, sucking, piercing• grasping etc.

Figure 2.19

2.2.1.1.3.2 Disadvantages:

• �nal body size is limited because as the body size increases, the surface area to volume ratio decreases.The larger the animal, the heavier the exoskeleton, making movement more di�cult.

• growth is restricted, so periodic moulting is required if the animal is to grow• very vulnerable when it is in the moulting process, as it cannot move until the exoskeleton is dry and

has hardened

2.2.1.1.4 Endoskeleton

• This skeleton is found inside the body and can consist of bone (vertebrates) or cartilage (sharks).

Available for free at Connexions <http://cnx.org/content/col11410/1.3>

79

Figure 2.20

2.2.1.1.4.1 Advantages:

• Endoskeletons consist of living tissue - so it is able to grow steadily within the animal enabling someto reach a large size.

• It can support the weight of a large animal• the skeleton is jointed which allows for �exible movement and support• muscles attach directly to the skeletal bones to allow for movement and support• vital organs are protected by bone cavities like the ribcage and the pelvic girdle• lends itself to adaptation to the environment Vertebrates move in di�eret ways ( e.g. running, jumping,

swimming, �ying.)

2.2.1.1.4.2 Disadvantages:

• Lack of mineral elements like calcium and phosphates will cause brittle bones and a�ect movementand support

• Lack of vitamin D in the diet results in a condition/disease caused rickets. A disease characterised bybowed legs.

2.2.1.2 Developmental progression and why skeletons are needed for terrestrial life

2.2.1.2.1 Problems that animals moving from water to land had to overcome

• Loss of body support provided by water

Water is about 1000 times denser than air and is a much more resistant medium to move through, its highdensity buoys up the body providing support.. One of the major problems by animals moving from waterto land was the increase in the force of gravity. In order to counter this, animals needed to develop stronglimbs and to adapt the skeleton to support their body weight on land.

• Locomotion

Available for free at Connexions <http://cnx.org/content/col11410/1.3>

80 CHAPTER 2. LIFE PROCESSES IN PLANTS AND ANIMALS

Moving e�ectively on land is essential particularly if one needs to avoid predator, catch prey or adapt to aparticular habitat.

• Desiccation

Exposure to air results in evaporation of water from breathing organs and body surface. If this problem isnot overcome animals will be con�ned to damp habitats

Constancy of temperatureNatural bodies of water do not show much temperature �uctuation, whereas on land, the range and �uc-

tuation in temperatures can be large. This can cause huge problems for animals which have no temperaturecontrol mechanisms.

• Gaseous exchange

Gills depend on water �ow or movement through water to operate. To survive on land a completely newmechanism of breathing needed to evolve.

• Opportunities for breeding on land

The provision of safe shelter for the protection of vulnerable eggs and young is far easier on land than in waterhabitats.. The main problem here was to �nd methods of reproduction which did not require fertilization inwater

• Variety of habitats

On land there are a tremendous variety of habitats e.g. tropical, coniferous and temperate forests, grasslands,deserts, mountains, oceanic islands and polar regions. Aquatic habitats, despite being far less diverse containthe greatest number and variety of living organisms on earth. Each habitat will have di�erent requirementswith regard to access to food� shelter, protection from enemies etc.

2.2.1.2.2 The exoskeleton in terrestrial arthropods

Arthropods moved onto the land long before the vertebrates emerged and a major factor in their successwas the exoskeleton.

• Highly protective without sacri�cing mobility• Provides attachment for muscles which control movement of appendages• In insects and spiders it is impregnated with waxes forming a waterproof barrier which prevents drying

out.

• Most terrestrial Arthropods have a tracheal system of breathing whereby air is forced in and outof a system of tubes by means of contraction and relaxation of muscles attached to the abdominalexoskeleton.

• Most insects have one or two pairs of wings which are formed from outgrowths in the thoracic regionof the exoskeleton

• The jointed appendages have sensory hairs and can be modi�ed and adapted for sensory functions(antennae), food handling(mouthparts), swift and e�cient walking legs and swimming appendages.

The exoskeleton because of its restriction on the size of terrestrial Arthropods was a major factor in futureprominence of the vertebrates.

2.2.1.2.3 Developmental progression in the vertebrate skeleton and associated organs.

The two major requirements for survival on land are the development of a suitable support system and anair breathing mechanism.

Available for free at Connexions <http://cnx.org/content/col11410/1.3>

81

2.2.1.2.3.1 Fish

• Bony �sh and cartilaginous �sh (sharks, rays etc) , apart from the skull have a skeleton consisting ofa vertebral column to which bands of muscle are attached on either side.

• When swimming a wave of contraction is produced on one side followed by another wave on the oppositeside.

• These alternate waves of contraction gives rise to S-shaped side to side movement of thebody which together with the sweeping movement of the tail drive the �sh forward. The �ns keepstability and play a role in steering.. This method of locomotion depends on the resistance of thesurrounding water and would not work on land.

• A prehistoric group of �sh had lobed �ns . These �sh lived in swamps or ponds which often driedup.

• It is thought that they used their �ns to drag themselves from a dried up pond to a wet one. Fossilforms seem to indicate that these �ns contained bones .

• They also had primitive �lungs� which enabled them to inhale small amounts of air.• Three existing �sh which have similar features are the coelacanth from deep seas, the mudskipper

which occurs in mangrove swamps and uses its front lobed �ns to climb trees, and the lung�sh whichlive in stagnant water surfacing to gulp air. None of these developed into Amphibia which arose fromone or more of the extinct forms.

2.2.1.2.3.2 Amphibia

• Amphibia: have partially adapted to land conditions.• Firstly they have developed lung breathing in adults while tadpoles use gills.• Secondly they have developed limbs for locomotion on land..• One group salamanders and newts (not found in Africa) resemble small lizards. Their legs are mostly

at right angles to the body and are small . Like �sh their bodies bend from side to side whenthey walk.

Frogs have adapted their skeletons to land conditions in the following ways:

• skull is much lighter and more �attened to allow for mobility on land.• the �exible vertebral column of �sh has been shortened and acts as a r igid frame for transmitting

force to the body .• There has been an extreme shortening of the body (9 vertebrae and there is no tail), as this would be

a hindrance when leaping and landing• there is a pectoral girdle which serves as support for the forelimbs which are used mainly to

absorb weight during landing after a jump.• the pelvic girdle is elongated and bear the hind limbs . When at rest the thigh, calf and foot

of a frog are each about the same length. As the frog jumps each part of the leg straightens in turnwhich results in great leaping power The webbing between the toes increase the thrust in the air aswell as when swimming in water.

• the frog's leap is not only used to move from one point to another but is also a very e�ective way ofescaping from a predator.

Despite these skeletal developments in adapting to land, they are still dependent on water inthat:

• They have a naked skin and are restricted to moist areas as they have no protection againstdesiccation.

• They have not developed an egg that is suited to terrestrial conditions and they thus have to laytheir eggs in water in order for the tadpole to mature into a frog.

Available for free at Connexions <http://cnx.org/content/col11410/1.3>

82 CHAPTER 2. LIFE PROCESSES IN PLANTS AND ANIMALS

2.2.1.2.3.3 Reptiles

The two major advantages that reptiles have over amphibians with regard to adaptations to terrestrial lifeare

• The development of a tough dry scaly skin which o�er protection against desiccation and physicalinjury.

• The development of a shelled egg containing food and protective membranes which allow embryonicdevelopment to take place on land.

With regard to the skeleton

• All reptiles except those that are limbless ( i.e. snakes) have better body support than theamphibians and more e�ciently designed limbs for travel on land .

• Most reptiles including crocodiles, most lizards and tortoises do not have an e�cient form of locomotionas the limbs are spread out at the sides of the body and the walk is a slow and waddling one. This isexhausting as a great deal of muscular e�ort is needed to keep the body o� the grounds. Such reptilescannot move rapidly and cannot support a large size.

• Some reptiles overcame this problem by becoming bipedal i.e only using the hind limbs for locomo-tion. Others gradually over time shifted their limbs to under the body. Many dinosaurs followed thisroute and were able to support a greater weight which is why some dinosaurs became huge.

• The development of ribs with a sternum(breastbone) has allowed for larger lungs and a moree�cient method of inhaling and exhaling air by the inward and outward movement of the ribs.

2.2.1.2.3.4 Birds

Birds di�er from the other vertebrates in that as a group they are able to �y and are designed for�ight. and many of their skeletal adaptations are linked to �ight . The main feature which identifya bird are its feathers and they are the only modern animals to have them

• Feathers are modi�ed reptilian scales and there are two types

· Contour feathers which are attached to the wings in such a way that they overlap to producea broad �at surface bene�cial for �ight..

· Down feathers provide excellent insulation against body heat loss. This is important as birds,like mammals are endothermic i.e. they generate heat in order to maintain a constantbody temperature .

• The paired forelimbs are usually adapted for �ying ( an exception is in penguins where theyare used as paddles for swimming).

• The bones are hollow and light due to the presence of air cavities .• Jaws are covered by horny beaks which have varied shapes according to the nature of the food source• The sternum/breastbone is enlarged and has a keel -like extension which provides attachment

for the strong muscles used for �ying.. Flightless birds such as ostriches do not have a keel.• The feet are also adapted to the bird's mode of life e.g. talons for birds of prey, webbed feet for

waterfowl. Perching feet for songbirds.

2.2.1.2.3.5 Mammals

• Body is covered by hair but mammals di�er in the amount, distribution and type of hair.• Apart from two species, mammals do not lay eggs, instead they are retained in a uterus and are

nourished by an organ called the placenta• After birth the young are nourished by milk from mammary glands .

Available for free at Connexions <http://cnx.org/content/col11410/1.3>

83

• Have 2 sets of teeth (milk teeth replaced by permanent teeth)• Have 4 di�erent types of teeth namely;

· Incisors : mainly for biting, snipping or gnawing � prominent in rodents and grazing animals.Elephant tusks are incisors

· Canines : have long cone shaped crowns (the part of the tooth that s above the gums) and arewell developed in carnivores. They have sharp edges for tearing and piercing.

· Premolars : have compressed crowns and one or two cusps (ridges) used for shearing andslicing

· Molars : with large bodies and variable cusp arrangement, are used for crushing and mastication.

• There are four limbs (reduced or absent in some) adapted for many forms of locomotion.• In most four legged mammals the leg bones are held directly underneath the body . In this

position they act as props or struts and it is the bones rather than the muscles that take most of thestrain of the body's weight. For this reason the animal is able to support the body clear of the groundfor long periods of time without tiring.

• Like birds they are endothermic and can maintain a constant body temperature.• Have a muscular partition or diaphragm between the thorax (chest region) and abdomen to make

breathing more e�ective.• They have a highly developed cerebrum (the brain's most complex region).

2.2.2 2.2.2 Human Skeleton11

Human Skeleton

• Humans have an internal skeleton made of bone, cartilage and connective tissue.

Figure 2.21: Human skeleton from the back

11This content is available online at <http://cnx.org/content/m43146/1.1/>.

Available for free at Connexions <http://cnx.org/content/col11410/1.3>

84 CHAPTER 2. LIFE PROCESSES IN PLANTS AND ANIMALS

Figure 2.22: Human skeleton from the front

2.2.2.1 Axial skeleton

This part of the skeleton consists of the skull comprising the cranium, facial bones, foramen magnum,palate and jaws, vertebral column, rib cage and breastbone(sternum)

Axial skeleton animationhttp://www.wisc-online.com/objects/ViewObject.aspx?ID=AP12904 12

2.2.2.1.1 The Skull: consists of the cranium and facial bones

• The cranium consists of eight �at bones joined together by immovable joints called sutures. Thecranium surrounds and protects the brain.

• There is a large opening at the base of the skull called the foramen magnum through which thespinal cord passes

• On either side of the foramen magnum is a projection or condyle which articulate with the �rstvertebra (called the atlas) to give the nodding movement of the head

• There are 15 facial bones. These are irregular bones that include cheek nasal , temple and upperand lower jaw bones. The only movable bone is the lower jaw.

• The upper and lower jaws bear the sockets for the 32 permanent teeth.• The number, type and arrangement of the teeth in an animal is indicated by a dental formula The

human dental formula is: 2.1.2.3

2.1.2.3

12http://www.wisc-online.com/objects/ViewObject.aspx?ID=AP12904

Available for free at Connexions <http://cnx.org/content/col11410/1.3>

85

This formula represents the numbers of each type of tooth in half of the upper jaw and half of the lowerjaw. This formula tells us that in both the upper and lower halves there are 2 incisors, 1 canine, 2 premolarsand 3 molars. Therefore in the whole jaw there are 8 incisors, 4 canines 8 premolars and 12 molars, a totalof 32 teeth in all.

2.2.2.1.2 THE VERTEBRAL COLUMN : CONSISTS OF 33 VERTEBRAE

The vertebral columns is divided into �ve regions

• A cervical (neck) region consisting of 7 vertebrae. The �rst of these called the atlas supportsthe skull and the joint with the skull allows for the nodding movement of the head.

The 2 nd vertebra called the axis has a projection oo which the atlas pivots to give the side to sidemovement of the head.

• A thoracic region (chest) of 12 vertebrae each of which bears a pair of ribs• A lumbar region (lower back) the largest vertebrae as they carry the weight of the body• A sacral region consisting of 5 fused vertebrae forming a bone called the sacrum that forms part

of the pelvic girdle which provides for the attachment of muscles and the legs.• A coccyx made up of 4 fused bones. These bones form the tail in those mammals that have tails.• The vertebrae join up to each other in such a way that there is a continuous spinal canal which

runs from the base of the skull to the pelvic girdle. This canal contains the spinal cord.• Between the vertebrae are discs of �brocartilage which prevent friction between vertebrae and act

as shock absorbers during walking, running and jumping.• Spinal nerves are able to enter and leave the spinal cord through gaps between adjacent vertebrae.• Strong ligaments and muscles around the spine stabilise the vertebrae and help to control

movement.

CURVES OF THE SPINEWhen viewed from the side the vertebral column can be seen to have four curves, with the cervical

and lumbar regions curving forwards while the thoracic and sacral regions curve backwards.In a newborn baby the entire vertebral column curves backwards probably because of the con�nes of the

uterus.Initially a baby cannot support the weight of its head.. When after about 3 months it is able to support

its head, the cervical forward curve is complete .The lumbar forward curve is complete when the baby is able to stand on its own and ready to learn

to walk.These curves of the vertebral column provide some of the resilience and spring so essential in balance

and movement. Abnormal curves may be due to poor posture, congenital disease or bone disease.FUNCTIONS OF VERTEBRAL COLUMN

• Supports the skull• Surrounds and protects the spinal cord.• Provides attachment for ribs, girdles, and back muscles• Separate vertebrae and S-shaped curvature provide �exibility allowing humans to bend backwards,

forwards and sideways.• Cartilage discs act as shock absorbers

RIB CAGEThe rib cage consists of 12 thoracic vertebrae, 12 pairs of ribs and the sternum or breastbone

• All 12 pair of ribs are attached to the thoracic vertebrae• The �rst 7 pairs are attached to the sternum by cartilage. These are the true ribs

Available for free at Connexions <http://cnx.org/content/col11410/1.3>

86 CHAPTER 2. LIFE PROCESSES IN PLANTS AND ANIMALS

• The next 3 pairs are each attached .to the rib above by means of cartilage. These are the false ribs.• The last 2 pairs are not attached at all to in the front and are called �oating ribs.• Between each pair of ribs are external and internal intercostal muscles• The sternum is a �at dagger shaped bone at the front of the rib cage.

Functions of rib cage

• Protection of the heart and lungs• With the help of the diaphragm and the intercostals muscles they increase. And decrease the volume

of the thoracic cavity thereb allowing inhalation and exhalation to take place.

2.2.2.2 APPENDICULAR SKELETON

This part consists of the pectoral girdle with arms and pelvic girdle with legs

Figure 2.23

Appendicular skeleton animationhttp://www.wisc-nline.com/objects/ViewObject.aspx?ID=AP13404 13

13http://www.wisc-nline.com/objects/ViewObject.aspx?ID=AP13404

Available for free at Connexions <http://cnx.org/content/col11410/1.3>

87

2.2.2.2.1 THE PECTORAL GIRDLE AND ARMS

The pectoral girdle consists of 2 clavicles (collar bones) and 2 scapulae (shoulder blades). Each clavicle isattached to the sternum in the front and the scapulae at the sides and they help to support they help tosupport the shoulders. The clavicle is the most frequently broken bone in the body as it often takes the fullimpact of falls on outstretched arms or of blows to the shoulder. Each scapula has a socket into which theupper arm �ts

Each upper arm has a single bone called the humerus which �ts into the Glenoid cavity on thescapula to form a ball and socket joint. This cavity is very shallow which allows the arms to move inalmost any direction for forms an elbow joint with the forearm

The forearm consists of two bones namely the ulna in line with the little �nger and the radius whichlies above the thumb. The joint at the elbow is a hinge joint.

The wrist consists of 8 small carpal bones arranged in two rows of four.The palm of the hand consists of 5 metacarpal bones. Their rounded ends at the base of the �ngers

form the knucklesThere are 14 �nger bones or phalanges in each hand, two in each thumb and three in each of the

�ngersUnlike the pelvic girdle the pectoral girdle is not fused to the spine but is connected by muscles to the

back of the thorax so its role as a supporting structure is much less than the pelvic girdle.. Instead it givesthe shoulders greater freedom of movement which in turn allows greater mobility of the arms.. Any limit tomovement is provided by the clavicle.-

Functions of pectoral girdle

• Forms a strong support for the attachments of the arms.• Provides large area of bone for the attachment of muscles.• Forms ball and socket joints with the arms which allows freedom of movement

2.2.2.2.2 PELVIC GIRDLE AND THE LEGS

The pelvic girdle consists of hip bones joined in the front by the pubic symphesis and attached to the sacrumat the back.

Each hip bone consists of three fused bones namely the:

1. ilium, which forms the upper �ared portion2. ischium, which is the lowest and strongest part and3. pubis, which forms the anterior part.

Portions of all three bones contribute to the formation of the acetabulum, the deep socket that holds thehead of the femur (thigh bone) to form the hip joint.

The female pelvic girdle is wider and lighter than the male as an adaptation to pregnancy and childbirth.The femur is the largest and strongest bone in the body. The upper end forms a ball and socket joint

with the hip bone while the lower end articulates with the tibia to form the hinge joint of the knee.The patella or kneecap is a �at triangular bone which is embedded in the tendon of the thigh muscle

and attached by ligament to the tibia.There are two bones in the lower leg: the tibia or shin bone which is the larger of the two and

supports most of the mass. The upper end articulates with the femur while the lower end articulates withone of the tarsal bones to form the ankle joint. The �bula (calf bone) is smaller than than the tibia andserves mainly for the attachment of muscles

The structure of the foot is similar to that of the hand. However the foot supports the weight of thebody, so it is stronger and less mobile than the hand.. There are seven tarsals or ankle bones only oneof which articulates with the tibia. The largest of these is the heel bone(calcaneum) to which the calfmuscle is attached and which presses �rmly on the ground when one stands, walks or runs.

Available for free at Connexions <http://cnx.org/content/col11410/1.3>

88 CHAPTER 2. LIFE PROCESSES IN PLANTS AND ANIMALS

There are 5 metatarsal bones the which form the ball and arch of the foot.The 14 phalanges of the toes are the counterparts of those in the �ngers, with the big toe having two

phalanges and the other 4 having 3 phalanges each. .

2.2.2.3 Functions of Skeleton:

1. Movement � allows body to move because muscles attach to the bones to give them leverage2. Protection � protects vital organs ( skull=brain, ribcage=heart and lungs and pelvic

bones=digestive tract and reproductive organs)3. Support � provides shape and support to body4. Storage of minerals - bones store minerals such as calcium and phosphate ions5. Hearing - bones in the middle ear, called the hammer, anvil and stirrup, amplify sound waves and

assist in the hearing process6. Red blood cells production - long bones and �at bones contain red bone marrow to produce red

blood cells

2.2.2.4 Structure of a long bone

2.2.2.4.1

2.2.2.4.2 PARTS OF A LONG BONE

Epiphysis: The head of each end of a long bone covered with hyaline cartilage and consisting largelyof spongy bone .

Diaphysis: Cylindrical shaft of a long bone composed of hard compact bone on the outside

Available for free at Connexions <http://cnx.org/content/col11410/1.3>

89

Periosteum: The membrane of dense �brous connective tissue which surrounds the outside surfaceof the shaft of a long bone. It has blood vessels which enables it to nourish the bone and repair injuries.It also provides a surface for the attachment of muscles, by means of tendons and ligaments.

Marrow cavity: This is �lled with yellow marrow which consists largely of fat.Endosteum: The delicate connective tissue layer lining the inside surface of compact bone.Cancellous/spongy bone : Found in the epiphysis of long bones and contain red marrow.Trabeculae: The struts in the network of irregular bony plates in the epiphysis of bones which transfer

stresses from the epiphysis to the diaphysis which has a much thicker layer of compact bone and resists stressbetter.

Red bone marrow: Found in the spaces between the trabeculae in spongy bone. This is where thered blood cells are made at the rate of 2 -3 million per second. White blood cell types are also producedhere.

2.2.2.4.2.1

TYPES OF BONES

• Long bones have a central shaft and two heads, one at each end. An example is the femur ,which is the largest bone in the body.

• Flat bones have two layers of compact bone covering a layer of spongy bone on the inside, forexample the shoulder blades .

• Irregular bones and short bones have a thin layer of compact bone covering spongy bone on theinside, for example vertebrae of the spine and the small bones in the hands and feet.

2.2.2.4.3 PRACTICAL INVESTIGATION

Experiment A:Aim: To investigate the role of the inorganic and organic components of boneApparatus: Exp A.: 2 small chicken bones2 test tubesDilute hydrochloric acidTowelMethod:

1. Label 2 test tubes with your initials and A and B. Put a bone in each tube.

2.Cover Bone A with water and Bone B with dilute hydrochloric acid. Leave for a few days. The acid willdissolve out the mineral component of the bone leaving behind the organic part

3.Take out Bone A and dry it.4.Use tweezers to take Bone B out of the acid. Rinse it under the tap and dry it.5.Compare the two bones and not down how they appear and whether they are soft or hard, �exible or

brittleExperiment BApparatus: 1 small chicken bonePipe clay triangle or wire gauze on a tripod standBunsen burner or Methylated spirits burnerMethod:1. Place the chicken bone (Bone C) on a pipe triangle or wire gauze on a tripod stand2.Roast the bone strongly for 10 minutes. Roasting will burn o� the organic component of bone ( mainly

the protein collagen) leaving behind the mineral part3.Allow the bone to cool down completely before you touch it.4.Describe the appearance of Bone C stating whether it is soft or hard, �exible or brittle

Available for free at Connexions <http://cnx.org/content/col11410/1.3>

90 CHAPTER 2. LIFE PROCESSES IN PLANTS AND ANIMALS

Questions1.What are the main inorganic components of bone?2.What changes have occurred in Bone A?3.What properties have been removed from the bone with the loss of it inorganic components?4.Which de�ciency disease can give similar results on bones in children?5.What is the role of Bone B in this experiment?6.What protein makes up the main organic component of bone?7. What changes took place in Bone C during the roasting process?8. What properties have been removed from Bone C with the loss of its organic component?

2.2.3 2.2.3 Associated Tissues14

2.2.3.1 Associated Tissues

2.2.3.1.1 BONES

• Provide the framework and internal core structure for the attachment of muscles• Bone is a living rigid tissue which forms the support structures for the rest of the body. The process

of bone formation is called ossi�cation.• The matrix of bone contains a dense arrangement of collagen �bres together with mineral salts of

calcium, magnesium and phosphates.• The calcium salts give bone its hardness and rigidity while collagen �bres give bones its �exibility and

strength.

2.2.3.1.1.1 FUNCTIONS OF BONE

• To serve as a �rm support framework for the whole body.• To protect such delicate structures as the brain and spinal cord• To serve as levers, working with attached muscles to produce movement.• To serve as a storehouse for calcium salts , which may be reabsorbed into the blood if there is

not enough calcium in the diet,• To produce blood cells in the red marrow.

14This content is available online at <http://cnx.org/content/m43144/1.1/>.

Available for free at Connexions <http://cnx.org/content/col11410/1.3>

91

Figure 2.24

2.2.3.1.1.2 MICROSCOPIC STRUCTURE OF A LONG BONE

• Numerous hollow tunnels called Haversian canals occur within the matrix of bone tissue and runparallel with the length of the bone. Under the microscope they appear as black circles against alighter background.

• Each Haversian canal is surrounded by concentric rings of compact bone called lamellae• Each of these layers contains a ring of �uid-�lled cavities called lacunae. Each of these lacuna will

contain a number of bone cells called osteocytes.• The lacunae are linked to each other and to the Haversian canal by a system of very tiny interconnecting

canals called canaliculi. Strands of cytoplasm extend through these canals which supply theosteocytes with oxygen and nutrients and remove waste products

• The Haversian canals, lacunae, osteocytes and canaliculi together form a unit called a HaversionSystem and a number of these systems make up compact bone.

• Apart from osteocytes which are embedded in the lacunae of bone there are two other types of bonecells

Osteoblasts : Bone forming cells. These cells allow the bone to change and remodel its shape as theorganism grows and responds to stresses. If a bone is broken or if strengthening is needed, bone cells laydown new tissue and repair damaged tissue

Osteoclasts: Special bone cells for destroying and reabsorbing bone tissue.

2.2.3.1.2 CARTILAGE

2.2.3.1.2.1 Main features

• cartilage is a tough semi-transparent �exible tissue• it is enclosed by a �brous capsule called the perichondrium• consists of living cells called chondrocytes which secrete a rubbery protein matrix called chondrin

Available for free at Connexions <http://cnx.org/content/col11410/1.3>

92 CHAPTER 2. LIFE PROCESSES IN PLANTS AND ANIMALS

• chondrocytes occur in small �uid-�lled spaces called lacunae which are scattered throughout thematrix.

• There are no blood vessels or nerves in the matrix.

2.2.3.1.2.2 Cartilage and Bone

Infant and young children do not have bones like those of adults. Their bones are made mostly of cartilage,a �rm elastic �brous material.

As the individual grows and matures, the cartilage is gradually replaced by bone cells which depositcrystals of calcium carbonate and calcium phosphate.

This process called ossi�cation greatly increases the strength of the bone.Bones usually continue to grow through adolescence. During this time a layer of cartilage still exists

between the head and shaft at either end of the bone. The growth of the bone does not interfere with theway joints �t together. Eventually once all the cartilage has become ossi�ed bone growth will stop

2.2.3.1.2.3 TYPES OF CARTILAGE

Hyaline Cartilage:Appearance: glass-like, bluish-white in colour, few �bres presentLocation:

• at the ends of bones as articular cartilage• where the ribs are joined to the sternum• forms rings in the trachea• larynx and tip of nose• as temporary cartilage in bones.

Functions:

• reduces friction at the joints.• allows a degree of movement during breathing• keeps the trachea open.• Forms permanent structures• Allows for bones to increase in length.

FibrocartilageAppearance: has numerous white collagen �bres in the matrix.Location:

• as cartilaginous discs between the vertabrae• in the rim of sockets of ball and socket joints• between the pubic bones

Functions:

• act as shock absorbers• make the cavity deeper without hampering movement• allows for limited movement

Elastic cartilageAppearance: has a network of yellow elastic �bres in the matrix.Location:

• in the pinna of the ear

Available for free at Connexions <http://cnx.org/content/col11410/1.3>

93

• in the epiglottis

Functions:

• maintains the shape of the ear but also allows for changes in shape.• strengthens the epiglottis which prevents food from entering the trachea.

2.2.3.1.3 Ligaments

• Ligaments consist of white collagen �bres and a network of yellow elastic �bres.• The collagen �bres are less orderly and more randomly arranged than in tendons and ligaments have

varying amounts of elastic �bres.• Ligaments join bone to bone and they also control the degree of movement allowed between the two

bones. This is achieved by the amount of elasticity in a ligament i.e. a ligament will only stretchenough to allow a particular movement to happen.

• The more elastic �bres in the ligament the greater the articulation between two bones. Thus theattachment of ligaments between bones keep the bones of a joint in position.

• By restricting bone movement ligaments will prevent any dislocation during normal actions.

2.2.3.1.4 Tendons

Attach muscles to bones and facilitate the various positions of the body related to movement and balance.

• Tendons consist of non elastic collagen �bres only.• These are densely packed, arranged in parallel bundles and are extremely strong, less �exible and more

resistant to stress• The �bres give tendons a white shiny appearance.• There is a minimal amount of matrix present.

2.2.4 2.2.4 Joints15

Jointshttp://www.bbc.co.uk/science/humanbody/body/fact�les/joints/ball_and_socket_joi nt.shtml 16

A joint is formed when two or more bones come into contact

2.2.4.1 Types of Joints

Joints are divided into 3 groups according to their degree of movement.

1. Immovable joints such as the bones of the skull (known as sutures) which are fused.2. Partly movable joints have cartilage between them which allows for a small degree of movement

e.g. between the vertebrae also called cartilaginous joints .3. Synovial joints are freely movable and are divided into 4 groups

• Hinge joints - e.g elbow and knee joints which allow movement in one plane only• Ball and Sock et joints - e.g. shoulder and hip joints allows free movement in almost all directions.• Pivot joint - between atlas and axis vertebrae � allows for turning movement of head• Gliding joints - between ankle bones and wrist bones � allows for rotational movements of hands and

feet

15This content is available online at <http://cnx.org/content/m43161/1.1/>.16http://www.bbc.co.uk/science/humanbody/body/fact�les/joints/ball_and_socket_joint.shtml

Available for free at Connexions <http://cnx.org/content/col11410/1.3>

94 CHAPTER 2. LIFE PROCESSES IN PLANTS AND ANIMALS

2.2.4.2 Structure of synovial joint

• The joint is completely enclosed in a bag-like joint capsule forming a synovial cavity.• The joint capsule is lined by a synovial membrane which secretes synovial �uid �lling the entire

cavity thereby reducing friction• The ends of the bones are covered in hya l ine articular cartilage• In addition to the joint capsule, other ligaments are present which attach bones to each other

Figure 2.25

SYNOVIAL JOINT

2.2.5 2.2.5 Human Locomotion17

De�nition 2.1: Locomotion

= Movement or the ability to move from one place to another.

De�nition 2.2: Human locomotion

= the ability you have to move from one place to another ( walking from your house toa friend's)

Harvard Outreach: Leg mechanics of playing basketball:http://outreach.mcb.harvard.edu/animations_S09.htm 18

17This content is available online at <http://cnx.org/content/m43153/1.1/>.18http://outreach.mcb.harvard.edu/animations_S09.htm

Available for free at Connexions <http://cnx.org/content/col11410/1.3>

95

2.2.5.1 What is used during locomotion?

2.2.5.1.1 1) Bones - body's supporting structure

• provide the framework• provide internal core structure for the attachment of muscles.• Protection of human organs• Keeps body shape

2.2.5.1.2 2) Joints - place in your body where two bones are connected

Three types of joints in your body:

2.2.5.1.2.1 i) Fibrous joints

• join bones where no movement is allowed• An example will be the bones of your cranium (the skull).

2.2.5.1.2.1.1 ii) Cartilaginous joints

• allows slight, restricted movement• for example the discs between the vertebrae of the spine

2.2.5.1.2.1.2 iii) Synovial joints

• Allow free movement in one or more directions to the joints of the pelvic and pectoral girdles.• These joints facilitate movements like standing, sitting, walking and running.

2.2.5.1.2.2 ii) Cartilaginous joints

• allows slight, restricted movement• for example the discs between the vertebrae of the spine

2.2.5.1.2.2.1 iii) Synovial joints

• Allow free movement in one or more directions to the joints of the pelvic and pectoral girdles.• These joints facilitate movements like standing, sitting, walking and running.

2.2.5.1.2.3 iii) Synovial joints

• Allow free movement in one or more directions to the joints of the pelvic and pectoral girdles.• These joints facilitate movements like standing, sitting, walking and running.

2.2.5.1.3 3) Ligaments � connect bone and bone.

• Hold bone in place so that they work in a coordinated manner.

Available for free at Connexions <http://cnx.org/content/col11410/1.3>

96 CHAPTER 2. LIFE PROCESSES IN PLANTS AND ANIMALS

2.2.5.1.4 4) Tendons - connect muscles to bone.

• Attachment to the skeletal muscles move your bones• Facilitate the various positions of the body related to movement and balance.

2.2.5.1.5 5) Antagonistic muscles

• Antagonistic = `opposite'• Antagonistic movement of muscles

• at least two sets of muscles• one set contracts and the other relaxes

• Contraction = stimulated muscle � becomes shorter and thicker• Relaxation = muscle relaxes

2.2.5.1.5.1 Example: Biceps and triceps

• The biceps is an example of a �exor muscle (muscle whose contraction shortens a body part)• Whereas the triceps is an example of an extensor muscle (muscle whose contraction extends or

stretches a body part).• Note that voluntary muscles are normally connected to at least two bones.

In the case of the biceps the two bones involved are the scapula and the humerus

• When the biceps muscle contracts only one of the bones moves ( in this case the radius). The point ofattachment to the movable bone is called the point of insertion and the biceps is attached to thispoint by a single tendon. So when the biceps contracts the forearm is lifted or bent, decreasing theangle between the forearm and humerus. and �exing your arm, Thus the biceps is a �exor muscle

• The biceps muscle gets its name from having two tendons attached to the scapula. The resistance. ofthese two tendons prevents the contractile force of the biceps from moving the scapula and thereforethere is no movement of the bone..

• The point of attachment of a muscle to the immovable bone is called the point of origin.

Available for free at Connexions <http://cnx.org/content/col11410/1.3>

97

Figure 2.26

Figure 2.2.1: Illustration of the triceps (extensor muscle) and biceps muscles (�exor mus-cle). Found in http://commons.wikimedia.org/wiki/File:Anatomy_and_physiology_of_animals_Antagonistic_muscles,_�exion%26tension.jpg 19

Straightening of the forearm

• When the arm is bent the biceps cannot contract as it is already in a contracted state as muscles canonly cause movement by pulling as they contract not by pushing when they relax.

• Therefore the straightening of the arm is brought about by the contraction of the triceps musclewhich is an extensor muscle as it increases the angle between forearm and humerus

• The triceps has three points of origin, two on the humerus and one on the scapula, and a single pointof insertion on the ulna.

19http://commons.wikimedia.org/wiki/File:Anatomy_and_physiology_of_animals_Antagonistic_muscles,_�exion%26tension.jpg

Available for free at Connexions <http://cnx.org/content/col11410/1.3>

98 CHAPTER 2. LIFE PROCESSES IN PLANTS AND ANIMALS

2.2.5.1.5.1.1 title

Videoillustrating the mechanics of the antagonism within the biceps and triceps.

http://www.youtube.com/watch?v=T-ozRNVhGVg&feature=related 20

Antagonistic muscles:http://www.botany.uwc.ac.za/Sci_Ed/grade10/manphys/skel_mus.htm 21

2.2.6 2.2.6 Muscles22

2.2.6.1 Muscles

De�nition 2.3: De�nition:= Muscle is a contractile 23 tissue 24 type of animals

Khan video: Anatomy of a muscle cellhttp://www.khanacademy.org/video/anatomy-of-a-muscle-cell?playlist=Biology 25

Khan video: Myosin and actinhttp://www.khanacademy.org/video/myosin-and-actin?playlist=Biology 26

Khan video: Role of sarcoplasmic reticulum in muscle cellhttp://www.khanacademy.org/video/role-of-the-sarcoplasmic-reticulum-in-muscle-

cells?playlist=Biology 27

20http://www.youtube.com/watch?v=T-ozRNVhGVg&feature=related21http://www.botany.uwc.ac.za/Sci_Ed/grade10/manphys/skel_mus.htm22This content is available online at <http://cnx.org/content/m43159/1.1/>.23http://en.wikipedia.org/wiki/Muscle_contraction24http://en.wikipedia.org/wiki/Tissue_%28biology%2925http://www.khanacademy.org/video/anatomy-of-a-muscle-cell?playlist=Biology26http://www.khanacademy.org/video/myosin-and-actin?playlist=Biology27http://www.khanacademy.org/video/role-of-the-sarcoplasmic-reticulum-in-muscle-cells?playlist=Biology

Available for free at Connexions <http://cnx.org/content/col11410/1.3>

99

2.2.6.1.1 Three types of muscle

2.2.6.1.1.1 Smooth/ involuntary

• not by will- spontaneous• unconscious routine tasks of the body:

· Food moving down the digestive system· keeping the eyes in focus· adjusting the diameter of blood vessels

2.2.6.1.1.1.1 Structure:

• spindle shaped cells with nucleus

Figure 2.27: Illustrates the structure of a smooth muscle

2.2.6.1.1.1.2 Functions:

Found in the walls of:

• blood vessels

Available for free at Connexions <http://cnx.org/content/col11410/1.3>

100 CHAPTER 2. LIFE PROCESSES IN PLANTS AND ANIMALS

• Uterus• bladder• Intestines

2.2.6.1.1.2 Cardiac muscle

• Responsible for your heart beat (muscle only found in the heart)• Only found in the walls of the heart

2.2.6.1.1.2.1 Structure

• branched and contains intercalated disks• Carry message in each cell for heart contraction

Figure 2.28: Illustrates the structure of the cardiac muscle

Available for free at Connexions <http://cnx.org/content/col11410/1.3>

101

2.2.6.1.1.3 Voluntary/skeletal

controlled by will

• Running• Walking• Skipping

2.2.6.1.1.3.1 Structure:

• The basic units of a muscle are called myo�brils .• These myo�brils make up the muscle �bre (large muscle cells).• Numerous muscle �bres (cells) are found in bundles .• These bundles are surrounded by perimisium

· This is called fasciculus

• Numerous fasciculi are surrounded by epimysium• This forms a muscle

Figure 2.29: Indicates the di�ering structural components of the voluntary muscle.

Available for free at Connexions <http://cnx.org/content/col11410/1.3>

102 CHAPTER 2. LIFE PROCESSES IN PLANTS AND ANIMALS

2.2.6.1.2 Muscle contraction

• Myo�brils are responsible for the muscle contraction.• Each myo�bril consists of units called sarcomeres (there are many sarcomeres in each myo�bril )• Sarcomeres consist of thin actin �laments and thick myosin �laments.• When muscle �bres contract these �laments slide across each other.• The actin �laments shorten, but the length of the myosin �laments do not change.• This causes the sarcomeres shorten,

· leading to the whole muscle to shorten

• ATP (energy) is a substance in the muscle �bre that provides energy for the contracting actin �lament.

IMAGE!!!Details on wish listVideo: Summary of the workings of the musclehttp://www.khanacademy.org/video/anatomy-of-a-muscle-cell?playlist=Biology 28

Muscle Exercise:Column B

A) Attached to skeleton by tendons 1) Cardiac muscle

B) Seen in bundles 2) Blood vesels

C) They make up muscle �bers 3) Muscles

D) Spindle shaped structure 4) movement

E) Causes the pumping action of the heart. 5) muscle �bres

F) smooth muscles are found here 6) Fasciculus

G) specialized tissue 7) myo�brils

H) contraction and relaxation 8) voluntary muscles

I) bundles surrounded by perimysium 9) epimysium

J) Numerous fasciculi are surrounded by 10) Involuntary muscle

Column B

A) Attached to skeleton by tendons 1) Cardiac muscle

B) Seen in bundles 2) Blood vesels

C) They make up muscle �bers 3) Muscles

D) Spindle shaped structure 4) movement

E) Causes the pumping action of the heart. 5) muscle �bres

28http://www.khanacademy.org/video/anatomy-of-a-muscle-cell?playlist=Biology

Available for free at Connexions <http://cnx.org/content/col11410/1.3>

103

F) smooth muscles are found here 6) Fasciculus

G) specialized tissue 7) myo�brils

H) contraction and relaxation 8) voluntary muscles

I) bundles surrounded by perimysium 9) epimysium

J) Numerous fasciculi are surrounded by 10) Involuntary muscle

Column B

A) Attached to skeleton by tendons 1) Cardiac muscle

B) Seen in bundles 2) Blood vesels

C) They make up muscle �bers 3) Muscles

D) Spindle shaped structure 4) movement

E) Causes the pumping action of the heart. 5) muscle �bres

F) smooth muscles are found here 6) Fasciculus

G) specialized tissue 7) myo�brils

H) contraction and relaxation 8) voluntary muscles

I) bundles surrounded by perimysium 9) epimysium

J) Numerous fasciculi are surrounded by 10) Involuntary muscle

Column B

A) Attached to skeleton by tendons 1) Cardiac muscle

B) Seen in bundles 2) Blood vesels

C) They make up muscle �bers 3) Muscles

D) Spindle shaped structure 4) movement

E) Causes the pumping action of the heart. 5) muscle �bres

F) smooth muscles are found here 6) Fasciculus

G) specialized tissue 7) myo�brils

Available for free at Connexions <http://cnx.org/content/col11410/1.3>

104 CHAPTER 2. LIFE PROCESSES IN PLANTS AND ANIMALS

H) contraction and relaxation 8) voluntary muscles

I) bundles surrounded by perimysium 9) epimysium

J) Numerous fasciculi are surrounded by 10) Involuntary muscle

Column B

A) Attached to skeleton by tendons 1) Cardiac muscle

B) Seen in bundles 2) Blood vesels

C) They make up muscle �bers 3) Muscles

D) Spindle shaped structure 4) movement

E) Causes the pumping action of the heart. 5) muscle �bres

F) smooth muscles are found here 6) Fasciculus

G) specialized tissue 7) myo�brils

H) contraction and relaxation 8) voluntary muscles

I) bundles surrounded by perimysium 9) epimysium

J) Numerous fasciculi are surrounded by 10) Involuntary muscle

Column A Column B

A) Attached to skeleton by tendons 1) Cardiac muscle

B) Seen in bundles 2) Blood vesels

C) They make up muscle �bers 3) Muscles

D) Spindle shaped structure 4) movement

E) Causes the pumping action of the heart. 5) muscle �bres

F) smooth muscles are found here 6) Fasciculus

G) specialized tissue 7) myo�brils

continued on next page

Available for free at Connexions <http://cnx.org/content/col11410/1.3>

105

H) contraction and relaxation 8) voluntary muscles

I) bundles surrounded by perimysium 9) epimysium

J) Numerous fasciculi are surrounded by 10) Involuntary muscle

Table 2.1

Choose the correct answer for column A from column B (only one correct answer per question)

2.2.6.1.2.1 Activity: Classifying Muscle Types

• Use the following story to classify the di�erent muscle types. Use a coloured pen or highlighter tounderline or highlight the actions in the story which require the di�erent types of muscles

• Suggested colours to use: Pink = Cardiac Muslces; Blue =Voluntary ; Yellow = Involuntary• Then draw each structure in the space provided

Story:BEEP BEEP BEEP!!!6 a.m on a Monday morning Tsholo's alarm goes o�.She jumps out of bed and walks to the toilet to relieve her bladder. Tsholo is very excited for the day

and skips back to her room to get dressed and pack her school bag for the new week. In the kitchen momhas prepared Tsholo's favourite porridge �Mielie Meal *. Tsholo eats het porridge with great pleasure. Afterbreakfast, she brushes het teeth and skips to the car where she waits for mom to unlock the doors.

At school Tsholo runs to her friends in total excitement to tell them about her visit to her grandmother.While chatting she sees Tom - the boy she likes a lot! He looks her way and Tsholo's starts blushing. Herheart rate increases and her palms become sweaty.

The bell rings.Tsholo and her friends walk to class, giggling and talking. Her heart rate slowly returns back to normal

. The week has begun. . .Draw and label the three di�erent muscle typesCardiac:Voluntary:Involuntary:

2.2.6.1.3 Interesting facts � Skeleton

• A baby is born with more bones (360 bones) than an adult (average 206 bones). Bones making up theskull and the spine fuse together as the body grows making it less.

• The femur/thigh bone is the largest in your body. The femur is approximately one quarter of a person'soverall height.

• Strengthen your skeleton by drinking milk and eating leafy greens (such as brussels sprouts and kale).They contain calcium which keeps bones healthy and strong.

• A broken bone produces many new cells to rebuild the bone. These cells cover both ends of the brokenpart of the bone and close up the break.

• Your bones are alive! In your body bones have their own nerves and blood vessels.• Your bone is 50% water and 50% solid material• You have 14 bones are in your face.• There are 8 bones in each of your wrists• You have 23 bones in each foot ( this includes the ankle)• Your skull is made up of fused bones which acts like a hard protective helmet for your brain.

Available for free at Connexions <http://cnx.org/content/col11410/1.3>

106 CHAPTER 2. LIFE PROCESSES IN PLANTS AND ANIMALS

2.2.7 2.2.7 Diseases29

2.2.7.1 Diseases

Bones like any other part of your body require proper care and can get diseased in older people and mal-nourished children. Common bone problems include Rickets, Osteoporosis and arthritis.

2.2.7.2 Rickets

• Childhood disease• Delay in mineral (calcium phosphate) deposition to harden bones � soft bones• causing characteristic bowed legs• Caused by Vitamin D de�ciency in the diet or when the body does not receive enough sunlight as the

body requires sunlight to make Vitamin D!

An X-ray picture of an adult who su�ered from rickets as a child (http://depts.washington.edu/bonebio/ASBMRed/diseases/rickets/rickets.html 30 )

2.2.7.3 Osteoporosis

• Greek for �bones with holes�.• Osteo �bone Porosis � Holes/Passages.• Common in older people.• When there is a shortage of calcium in the body or when a bone is inactive e.g leg in plaster or

imobilised, calcium is withdrawn from the leg for use in other parts of the body.. This together witha decrease in bone protein, will result in an increased breakdown of bone tissue without an increase indeposit of new bone by osteoblasts

and the development of holes in the bone ( hence the name of the disease

• Decrease in bone density which makes it weak and prone to fractures• The most typical fractures are of the spine, wrist and hip.

2.2.7.4 Arthritis:

• The most common form of arthritis a�ects the bone and is known as Osteoarthritis.• Commonly known as �wear-and-tear� arthritis.• Degradation of the soft cartilage in joints between bones causes the end of the bones to rub against

each other which is painful!

Visualizing osteoarthritis: http://www.youtube.com/watch?v=a1d8qK4BEx0&feature=related 31

Further reading on the web: http://depts.washington.edu/bonebio/ASBMRed/diseases.html 32

Types of arthritis: http://www.vimovo.com/types-of-arthritis.aspx 33

29This content is available online at <http://cnx.org/content/m43104/1.1/>.30http://depts.washington.edu/bonebio/ASBMRed/diseases/rickets/rickets.html31http://www.youtube.com/watch?v=a1d8qK4BEx0&feature=related32http://depts.washington.edu/bonebio/ASBMRed/diseases.html33http://www.vimovo.com/types-of-arthritis.aspx

Available for free at Connexions <http://cnx.org/content/col11410/1.3>

107

2.2.8 2.2.8 Summary34

2.2.8.1 Summary

• There are 3 types of skeletons:

1. Hydrostatic skeleton2. Endoskeleton3. Exoskeleton

• When animals moved from water to land, there was a need for the development of strong limbs and askeleton to provide support to the bodies, which before water had provided

• Humans have an endoskeleton consisting of:

1. Axial skeleton (cranium, facial bones, foramen magnum, palate and jaws, vertebral column, rib cageand breastbone(sternum)

2. Appendicular skeleton (pectoral girdle with arms and pelvic girdle with legs)

• Functions of the human skeleton are:

1. Movement2. Protection3. Support4. Storage of minerals5. Hearing

• There tissues associated with the human skeleton are bone, cartilage, tendons and ligaments

• Joints

• A joint is formed when two or more bones come into contact

• There are three types of joints

1. Immovable joints2. Partly movable joints3. Synovial joints (Hinge joints, ball and socket joints, pivot joints, gliding joints)

• Human locomotion requires the use and coordination of bones, joints, ligaments, tendons and antago-nistic muscles

• Muscles

- There are three types:1) Smooth/involuntary2) Skeletal/voluntary3) Cardiac muscle

• Myo�brils are responsible for muscle contraction

• There are many diseases that a�ect the skeleton, such as rickets, osteoporosis and arthritis

34This content is available online at <http://cnx.org/content/m43078/1.1/>.

Available for free at Connexions <http://cnx.org/content/col11410/1.3>

108 CHAPTER 2. LIFE PROCESSES IN PLANTS AND ANIMALS

2.2.9 2.2.9 Exercises35

Figure 2.30

Figure 2.31

2.2.9.1 Exercises

2.2.9.1.1 Question 1

1. Name three types of skeletons and provide one advantage and one disadvantage of each. (9)2. State where the Haversian canal is located and state its function. (3)3. Name four functions of bone tissue. (4)4. Tabulate two di�erences between tendons and ligaments. (5)

2.2.9.1.2 Question 2

1. Supply the biological term for each of these bones:

a. thigh boneb. knee capc. shin boned. ankle bonese. heel bonef. upper arm boneg. wrist bonesh. breast bone (8)

2. Name FOUR functions of the human skeleton. (4)3. State the number of:

a. bones in the human vertebral column. (1)b. pairs of true ribs (1)c. lumbar vertebrae (1)

35This content is available online at <http://cnx.org/content/m43169/1.1/>.

Available for free at Connexions <http://cnx.org/content/col11410/1.3>

109

2.2.9.1.3 Question 3

Study the following diagrams showing the main bones of the pectoral girdle and the human arm (forelimb)and answer the questions that follow:

Figure 2.32

1. Identify bone X. (1)2. Parts of some of these bones meet at certain joints. By using the letters3. (A � H) only, state which parts of the bones form the shoulder joint. (2)4. Name the type of synovial joint that is located at the following parts of the body:

a. At the elbow (1)b. Where the lower limb joins the pelvis (1)c. In the wrist (1)

2.2.9.1.4 Question 4

The diagram below shows the legs of an athlete while he is waiting for a race to start. The letters A to Fshow some of the muscles as well as joints that will be used during the race.

Available for free at Connexions <http://cnx.org/content/col11410/1.3>

110 CHAPTER 2. LIFE PROCESSES IN PLANTS AND ANIMALS

Figure 2.33

1. When the Starter's gun is �red, the athlete's right leg will straighten, pushing the athlete upwards andforwards. Which of the letters (A to F) indicate muscles that will:

a. Relax (2)b. Contract (2)

2. The leg shown in the diagram has di�erent types of joints. Which of the following letters (A to F)indicates:

a. A hinge joint (1)b. A ball and socket joint (1)

2.2.9.1.5 Question 5

Skeleton and Movement � True or False?

1. The skeleton's role is to provide support, protection and capacity for movement.2. The skeleton is divided into the axial and appendicular skeleton.3. The axial skeleton consists of the pectoral and pelvic girdles and their attached limbs.4. Carpals are found in the ankles and tarsals in the wrists.5. The biceps muscle raises the arm while the triceps lowers it in an antagonistic pair.6. Synovial liquid lubricates joints and keeps them friction free.7. Bone joints in the cranium are examples of �brous joints.

Available for free at Connexions <http://cnx.org/content/col11410/1.3>

111

8. The neck contains 7 lumbar vertebrae.9. Tendons join muscles to bone and are elastic while ligaments join bone to bone and are non-elastic.10. Bone is composed of �exible minerals such as Calcium and Phosphate with rigid �bres of Collagen.11. Osteocyte is another word for bone cell. (11)

2.2.9.1.6 Question 6

Compare the biceps and triceps muscles with respect to:

1. Point of origin (4)2. Point of insertion (2)3. Function (2)

2.2.9.1.7 ANSWERS

2.2.9.1.7.1 Question 1

1)

SKELETON ADVANTAGE DISADVANTAGE

Hydrostatic - Allows animal to be very �exi-ble � moves easily; not restrictedin terms of possible movements.-Give support without addingmuch weight.- Allows rapid dif-fusion of gases through the bodywall, so a transport system is of-ten unnecessary, e.g. jelly�sh.

- Not very strong � easily dam-aged or lost if the enclosed cav-ity around it is pierced.- Gener-ally not suitable to terrestrial an-imals and o�ers not protectionsagainst dehydration.- Limits thesize of the animal � large animalswould not be feasible.

Exoskeleton - Very strong and provides goodprotection against damage.- Canbe present on great variety ofcolours to provide protection viacamou�age.- O�er good protec-tion against dehydration.

- Heavy, so it prevents the an-imal getting very large. (smallanimals are easy prey)- Necessi-tates moulting, making the ani-mal very vulnerable.- Movementis only possible at thinner joints,but these are more vulnerablethan thick areas.

continued on next page

Available for free at Connexions <http://cnx.org/content/col11410/1.3>

112 CHAPTER 2. LIFE PROCESSES IN PLANTS AND ANIMALS

Endoskeleton - Bone is very hard, so gives ex-cellent protection of vital organse.g. brain.- Bone marrow insidebones forms blood cells.- Allowsanimal to become bigger � largeanimals have fewer enemies

- Broken bones take a long timeto heal and are painful.- Bones in-side the body o�er no protectionto some soft tissues, e.g. intes-tine.

Table 2.2

(One mark per skeleton, one for an advantage and disadvantage each) [9]2) Haversian canal is in the centre of a Haversian systemΠ in compact bone. It contains a nerve to carry

impulsesΠ, blood vessels to transport gasesΠ, food and wastes and a lymph duct to drain tissue �uid. (onefor where it is, + 2 for any two functions of parts in it [3]

3) Any four of the following:

• Is hard to support the body part and protect vital organs• Forms a store of calcium and phosphorus in the body• Can undergo mitosis to repair damage, e.g. breaks• Grows to make the body bigger as we age• Provides �rm attachment place for muscles• Protects bone marrow that produces blood cells• Allows fast di�usion to and from osteoblasts via liquid-�lled canaliculi [4]

4)

TENDONS LIGAMENTS

Attach muscles to bonesContain more collagencompared to elastin, so they are very inelasticFibresin tendons are all along the long axis for strength

Attach bones to other bones or to �brocartilage-Contain less collagen compared to elastin, so theyhave slight elasticityThe �bres in ligaments are wo-ven together, not arranged longitudinally

Table 2.3

(One for the table and one each for two di�erences between them) [5]

2.2.9.1.7.2 Question 2

1)

1. femur2. patella3. tibia4. tarsals5. calcanum6. humerus7. carpals8. sternum [8]

2)

• Protection of vital organs, e.g. brain, heart• Attachment place for muscles and resistance for muscle contraction

Available for free at Connexions <http://cnx.org/content/col11410/1.3>

113

• Gives shape to the body, e.g. facial features• Supports body parts and keeps us upright• Bones form levers for locomotion• Bone marrow forms blood cells• Middle ears bones are vital for hearing [4]

3)

1. 332. 73. 5 [3]

Scapula [1]Bone end B joins to part Fhinge jointball and socket jointgliding joint [3]

2.2.9.1.7.3 Question 4

a) B and F [2]b) C and E [2]2a) Db) A [2]

2.2.9.1.7.4 Question 5

1. True2. True3. False (should be appendicular skeleton)4. False (other way round � carpals are in the wrist, tarsals in the ankle)5. False (the biceps and triceps respectively raise and lower the LOWER arm or forearm, not whole arm)6. True (not entirely `friction-free', but close)7. True8. False (neck has cervical vertebrae)9. False (joining functions are correct, but tendons are inelastic and ligaments are more elastic)10. False (Ca and P are in�exible minerals and collagen is a �exible protein)11. True (but bone cells can also be called osteoblasts) [11]

2.2.9.1.7.5 Question 3

1. Biceps and triceps BOTH originate at the shoulder (remember the origin is the part that DOES NOTMOVE when the muscle contracts). Biceps has one tendon at shoulder, triceps has two (4)

2. Biceps is inserted on the radius and triceps on the ulna (remember the insertion is the part thatMOVES when the muscle contracts) (2)

3. The biceps contracts to bend the arm at the elbow, the triceps contracts to straighten the arm at theelbow. (2)

Available for free at Connexions <http://cnx.org/content/col11410/1.3>

114 CHAPTER 2. LIFE PROCESSES IN PLANTS AND ANIMALS

2.3 Transport systems in mammals (humans)

2.3.1 2.3.1 Blood circulatory system36

2.3.1.1 Useful links

Cardiovascular system: http://www.biologyinmotion.com/cardio/index.html 37

Khan: Red blood cells http://www.khanacademy.org/video/red-blood-cells?playlist=Biology 38

Khan: Circulatory system and the heart http://www.khanacademy.org/video/circulatory-system-and-the-heart?playlist=Biology 39

YouTube video: Circulatory system http://www.youtube.com/watch?v=D3ZDJgFDdk0 40

Blood �ow animation http://health.howstu�works.com/human-body/systems/circulatory/adam-200078.htm41

Blood Flow animation http://www.sumanasinc.com/webcontent/animations/content/human_heart.html42

The heart and circulation (interactive activity): http://www.kett6.net/adulteducation/heartanimations.html43

Circulation animation: http://www.bbc.co.uk/schools/gcsebitesize/pe/appliedanatomy/0_anatomy_circulatorysys_rev1.shtml 44

2.3.1.2 Overview

All living cells require nutrients and oxygen to survive. Cells produce metabolic waste, which must be re-moved and excreted. The circulatory system is responsible from providing nutrients and removing metabolicwaste.

Unicellular organisms have a simple system to allow for this and it is by di�usion where substancesmove from a high concentration to a low concentration.

Most invertebrates like a grasshopper have an open circulatory system , where blood(haemolymph) bathes the body organs.

By comparison, mammals have a closed circulatory system since blood is contained within bloodvessels.

2.3.1.3 Pulmonary and Systemic circulatory systems

Open circulatory systemblood is pumped into a heamocoel (an open space or cavity) that surrounds to organs. Muscle movement

also helps to pump then blood. Blood di�uses back the heart. Blood movement is sluggish. There is nodi�erence between the blood and the interstitial �uid. Interstitial �uid is the �uid that surrounds the cells.Blood is not contained within capillaries.

Closed circulatory systemblood is pumped from the heart through arteries and returns to the heart via veins. Blood never leaves

the vascular system (arteries, veins and capillaries). Nutrients, water and metabolic waste di�uses out ofthe vascular system and into the interstitial �uid. Interstitial �uid and blood are separated, by the vascularsystem. Interstitial �uid returns to circulation through the lymphatic system.

36This content is available online at <http://cnx.org/content/m43150/1.1/>.37http://www.biologyinmotion.com/cardio/index.html38http://www.khanacademy.org/video/red-blood-cells?playlist=Biology39http://www.khanacademy.org/video/circulatory-system-and-the-heart?playlist=Biology40http://www.youtube.com/watch?v=D3ZDJgFDdk041http://health.howstu�works.com/human-body/systems/circulatory/adam-200078.htm42http://www.sumanasinc.com/webcontent/animations/content/human_heart.html43http://www.kett6.net/adulteducation/heartanimations.html44http://www.bbc.co.uk/schools/gcsebitesize/pe/appliedanatomy/0_anatomy_circulatorysys_rev1.shtml

Available for free at Connexions <http://cnx.org/content/col11410/1.3>

115

2.3.1.3.1 The Human Circulatory System

All mammals have a closed blood circulatory system - blood always �ows inside blood vessels.A double circulatory system = blood passes through the heart twice:

1. Pulmonary circulation: the blood is pumped from the heart to the lungs to oxygenate the bloodand then back to the heart.

2. Systemic circulation (to all the systems): the blood is pumped from the heart to all parts ofthe body and back to the heart again.

3. Coronary circulation : is a circulatory system that supplies the heart muscle with the blood itrequired in order to function.

Very simple simulation of blood �ow through the systemic and pulmonary circulatory systems. The illus-tration shows each of these circulatory systems to be separate loops leaving from one side of the heart andreturning to the other.

http 45 :// 46 www 47 . 48 biologyinmotion 49 . 50 com 51 / 52 cardio 53 / 54 index 55 .56 html 57

Figure: Simpli�ed Diagrammatic sketch of the entire circulatory system. Blood �ows to every inch ofthe body, even to the tips of the �ngers and toes. Lungs provide oxygen to the blood. The digestive systemsupplies nutrients. The kidneys �lter the blood.

45http://www.biologyinmotion.com/cardio/index.html46http://www.biologyinmotion.com/cardio/index.html47http://www.biologyinmotion.com/cardio/index.html48http://www.biologyinmotion.com/cardio/index.html49http://www.biologyinmotion.com/cardio/index.html50http://www.biologyinmotion.com/cardio/index.html51http://www.biologyinmotion.com/cardio/index.html52http://www.biologyinmotion.com/cardio/index.html53http://www.biologyinmotion.com/cardio/index.html54http://www.biologyinmotion.com/cardio/index.html55http://www.biologyinmotion.com/cardio/index.html56http://www.biologyinmotion.com/cardio/index.html57http://www.biologyinmotion.com/cardio/index.html

Available for free at Connexions <http://cnx.org/content/col11410/1.3>

116 CHAPTER 2. LIFE PROCESSES IN PLANTS AND ANIMALS

2.3.1.4 The Heart and Associated Blood Vessels

Figure 2.34

Available for free at Connexions <http://cnx.org/content/col11410/1.3>

117

Figure 1 : Heart overlayed on a body so show the location of the heart within the chest.http://en.wikipedia.org/wiki/File:Surface_anatomy_of_the_heart.png58

• The heart is situated in your thorax just behind your breastbone and is about the size of your �st.• It is a large muscle that pumps through repeated rhythmic contractions and therefore requires lots of

nutrients and oxygen.• On the surface of the heart are coronary arteries that are arteries that branch o� the aorta and

supply the heart with oxygen and nutrients.• The heart is made up of 4 chambers and divided by a septum into a right and left half.• The right half of the heart pumps deoxygenated blood up into the pulmonary artery, towards the

lungs (pulmonary circulation), where it is oxygenated.• Oxygenated blood returns from the lungs via the pulmonary veins and enters the left side of the heart.• The left side of the heart then pumps oxygenated blood up through the aorta, and into the general

circulation (systemic circulation) and the oxygen is consumed by the body.• Deoxygenated blood returns to the right side of the heart via the inferior vena cava which drains blood

from below the heart and superior vena cava, which brings blood from the head and arms.• The human circulatory system is a double circulatory system, because blood travels to the heart twice

during circulation, once before going to the lungs and once before circulating throughout the body.• Blood only �ows in one direction, through the circulatory system.• All vessels that �ow A way from the heart are called A rteries.• All blood vessels entering the heart are called V eins.• The terms artery and vein are not determined by what the vessel transports (oxygenated blood or

deoxygenated) but by whether the vessel �ows to or from the heart.

58http://en.wikipedia.org/wiki/File:Surface_anatomy_of_the_heart.png

Available for free at Connexions <http://cnx.org/content/col11410/1.3>

118 CHAPTER 2. LIFE PROCESSES IN PLANTS AND ANIMALS

Figure 2.35

Figure 2 : General structure of the heart and associated blood vessels(http://en.wikipedia.org/wiki/File:Anatomy_Heart_English_Tiesworks.jpg)

2.3.1.5 internal structure of the heart

• The heart is made up of 4 chambers. There are 2 atria at the top of the heart which receivesblood and 2 ventricles at the bottom of the heart which pumps blood out of the heart.

• The septum divides the left and right side of the heart.• The valves of the heart ensure that blood only �ows one way through the heart.

The tricuspid valve is found between the right atrium and the right ventricle.The mitral valve is found between the left atrium and the left ventricle.Strong tendinous chords attached to valves prevent them from turning inside out when they close.The semi-lunar valves are located at the bottom of the aorta and pulmonary artery.

Available for free at Connexions <http://cnx.org/content/col11410/1.3>

119

Figure 2.36: Internal structure of the heart

Interesting facts : Humans, birds, and mammals have a four-chambered heart. Fish have a two-chambered heart, one atrium and one ventricle. Amphibians have a three- chambered heart with two atriaand one ventricle. The advantage of a four chambered heart is that there is no mixture of the oxygenatedand deoxygenated blood.

Available for free at Connexions <http://cnx.org/content/col11410/1.3>

120 CHAPTER 2. LIFE PROCESSES IN PLANTS AND ANIMALS

Figure 2.37

Figure 2.38

continued on next page

Available for free at Connexions <http://cnx.org/content/col11410/1.3>

121

Figure 2.39

continued on next page

Available for free at Connexions <http://cnx.org/content/col11410/1.3>

122 CHAPTER 2. LIFE PROCESSES IN PLANTS AND ANIMALS

Figure 2.40

continued on next page

Available for free at Connexions <http://cnx.org/content/col11410/1.3>

123

Figure 4. The relationship of the heart and circulatory system to major visceral organs.

Table 2.4

The circulatory song http://www.youtube.com/watch?v=q0s-1MC1hcE&NR=1 59

2.3.1.6 The Cardiac Cycle

• The human heart will undergo over 3 billion contraction cycles, as shown in Figure 5, during a normallifetime.

• The heart beats in a rhythmic cycle. A complete cardiac cycle is one round of the heart pumpingblood.

• The top half of the heart works as one unit.• The bottom half of the heart works as one unit.• The sino-atrial node (pacemaker) starts and regulates the process.• The cardiac cycle consists of two parts: systole (contraction of the heart muscle) and diastole

(relaxation of the heart muscle).

Atrial systole (0.1s)

• Atria contract simultaneously, pushing blood into the ventricles• Ventricles are relaxed• Atrio ventricular valves open• Semi lunar valves close

Ventricular systole (0.3s)

• Atria relax• Ventricles contract simultaneously• Blood enters aorta and pulmonary artery• Semi lunar valves open• Atrio ventricular valves close

Diastole (0.4s)

• Atria and ventricles relax• Atrio ventricular valves open• Blood enters atria and ventricles• Semi lunar valves close• Back �ow in arteries prevented

The heart beat can be heard as a sound that the valves make when they close. The `lub' sound is madewhen the atrio ventricular valves close and the `dub' sound is made when the semi lunar valves close.

59http://www.youtube.com/watch?v=q0s-1MC1hcE&NR=1

Available for free at Connexions <http://cnx.org/content/col11410/1.3>

124 CHAPTER 2. LIFE PROCESSES IN PLANTS AND ANIMALS

Figure 2.41

Figure 5from mindset � (please check permission from this, found it in Biology 6th edition Campbell and Reece)Cardiac Cycle: �ow of blood through the heartExcellent simple video illustrating the heart cycle.http://www.youtube.com/watch?v=D3ZDJgFDdk0 60

2.3.1.6.1 Blood Pressure

• The blood pressure is produced by the left ventricle contractions.• The rhythm of ventricle diastole, often just referred to as diastole, causes the pulse, which can be felt

by holding two �nders to the side of the throat.

60http://www.youtube.com/watch?v=D3ZDJgFDdk0

Available for free at Connexions <http://cnx.org/content/col11410/1.3>

125

• Blood pressure oscillates with the contraction of the left ventricle.

Ideal blood pressure for an adult is:Systolic pressure: 120 mm HGDiastolic blood pressure: 80 mm HGA usual rule is that systolic pressure should be 100 plus your age but never more than 140 and diastolic

pressure should not be over 90.

Figure 2.42

Table 2.5

Figure 6 The cardiac cycle. Image from Purves et al., Life: The Science of Biology , 4th Edition, bySinauer Associates ( www.sinauer.com 61 ) and WH Freeman ( www.whfreeman.com 62 ),(please getpermission)

Normal Heart Soundshttp://upload.wikimedia.org/wikipedia/commons/7/72/HROgg.ogg 63

61http://www.sinauer.com/62http://www.whfreeman.com/63http://upload.wikimedia.org/wikipedia/commons/7/72/HROgg.ogg

Available for free at Connexions <http://cnx.org/content/col11410/1.3>

126 CHAPTER 2. LIFE PROCESSES IN PLANTS AND ANIMALS

2.3.1.7

2.3.1.8 Lung and pulmonary system

Khan Academy video on the pulmonary system. Overview on breathing.http://www.khanacademy.org/video/the-lungs-and-pulmonary-system?playlist=Biology 64

• The lungs serve as the air-blood interface.• Blood from the lungs is pumped into the pulmonary arteries.• From the pulmonary arteries the vascular system branches into smaller and smaller vessels until the

blood is �owing through thin pulmonary capillaries.• These capillaries surround the alveoli in the lungs.• At this point there are only two layers of cells separating the blood from the air.• Carbon dioxide in deoxygenated blood di�used out of the blood.• Oxygen in the lungs di�use in to the blood oxygenating it Oxygen is absorbed.• Oxygenated blood then returns to the heart vial the pulmonary veins.

Figure : Details arteries and veins connecting the heart to the lungs Red blood has been oxygenated, blueblood is deoxygenated. . (Wikipedia - http://en.wikipedia.org/wiki/File:Illu_pulmonary_circuit.jpg)

Figure 2.43

64http://www.khanacademy.org/video/the-lungs-and-pulmonary-system?playlist=Biology

Available for free at Connexions <http://cnx.org/content/col11410/1.3>

127

Figure 2.44

Figure : Very detailed image of the lungs, it is not necessary or requiredto know all this detail but this is a fantastic image of the lungs � wikipedia(http://en.wikipedia.org/wiki/File:Respiratory_system_complete_en.svg).

Available for free at Connexions <http://cnx.org/content/col11410/1.3>

128 CHAPTER 2. LIFE PROCESSES IN PLANTS AND ANIMALS

2.3.1.9 Major organs and systemic system: associated major blood vessels the brain, small

intestines, liver, kidney.

Figure : Detailed illustration of the systemic circulation. http://en.wikipedia.org/wiki/File:Circulatory_System_en.svg65

• All the organs of the body are supplied by blood.• Each has an artery supplying the organ with blood from the heart, and veins returning blood to the

heart.• Arteries and veins have been named according to the organ which they supply blood to.

The circulatory system forms a closed system.

• Nutrients enter the circulatory system from the digestive system.• These nutrients �rst move to the liver via the hepatic portal vein, the liver then controls the nutrient

composition of the blood.• Blood passes from the liver to the heart through the hepatic vein.• Nutrients are then circulated throughout the body.• Cells consume the nutrients in the blood and produce metabolic waste. T• his metabolic waste is circulated in the blood, if it remains in the blood the blood would eventually

become toxic.• The kidneys are supplied with blood via the renal arteries and they remove metabolic waste from the

blood, passing it to urine.• The Brain is supplied with blood via the carotid arteries and the vertebral arteries. The blood is

drained via the jugular veins. The brain is supplied with 15% of the total amount of blood pumpedby the heart.

2.3.1.10 Mechanisms for controlling cardiac cycle and heart rate (pulse)

• The cardiac cycle is controlled by nerve �bers extending from nodes of nerve bundles through the heartmuscle.

65http://en.wikipedia.org/wiki/File:Circulatory_System_en.svg

Available for free at Connexions <http://cnx.org/content/col11410/1.3>

129

• An electrical signal is triggered in the node.• The electrical signal then spreads through the �bers and causes the heart muscle to contract.

2.3.1.10.1 There are two nodes:

1. The sinoatrial node (SA), which initiates the heart cycle. Electrical signals spread from the SAacross the atria causing it to contact.

2. The electrical signal also reaches the Atrioventricular node (AV) . Here the signal pauses, beforespreading through the ventricles causing them to contract.

• The SA is able to initiate the electrical signal without any stimulation for the nervous system, but itcan be controlled by the nervous system.

• The brain does not need to tell the heart to beat; it is able to beat on its own.

• The brain can make the heart rate increase, when for instance you are scared or are running.• Hormones are also able to increase the heart rate.

Simple simulation of how electrical activity spreads over the heart.Link : http://en.wikipedia.org/wiki/File:ECG_Principle_fast.gif 66

Measuring pulse rate: http://www.nlm.nih.gov/medlineplus/ency/article/003399.htm 67

2.3.1.10.2 How the Nodes cause contraction

• Human heartbeats originate from the sinoatrial node (SA node) near the right atrium.• Modi�ed muscle cells contract, sending a signal to other muscle cells in the heart to contract.• The signal spreads to the atrioventricular node (AV node).• Signals carried from the AV node, slightly delayed, through bundle of His �bers and Purkinjie �bers

cause the ventricles to contract simultaneously. Figure 13 illustrates several aspects of this.

66http://en.wikipedia.org/wiki/File:ECG_Principle_fast.gif67http://www.nlm.nih.gov/medlineplus/ency/article/003399.htm

Available for free at Connexions <http://cnx.org/content/col11410/1.3>

130 CHAPTER 2. LIFE PROCESSES IN PLANTS AND ANIMALS

Figure 13. The contraction of the heart and the action of the nerve nodes located on the heart. Imagesfrom Purves et al., Life: The Science of Biology , 4th Edition, by Sinauer Associates ( www.sinauer.com68 ) and WH Freeman ( www.whfreeman.com 69 ), (please get permission)

Figure 2.45

Figure 2.46

continued on next page

Available for free at Connexions <http://cnx.org/content/col11410/1.3>

131

Figure 2.47

Table 2.6

2.3.1.10.3 Electrical activity

• The electrical activity in the heart is so strong that is can be measured from the surface of the bodyas an electrocardiaogram (ECG).

• A normal heart has a very regular rhythm.• An abnormal heart may have an arrhythmia, or abnormal rhythm as shown in the �gures.

Figure 15. Normal cardiac pattern (top) and some abnormal patterns (bottom). Images from Purves etal., Life: The Science of Biology , 4th Edition, by Sinauer Associates ( www.sinauer.com 70 ) and WHFreeman ( www.whfreeman.com 71 ), (please contact for permission).

continued on next page

68http://www.sinauer.com/69http://www.whfreeman.com/

Available for free at Connexions <http://cnx.org/content/col11410/1.3>

132 CHAPTER 2. LIFE PROCESSES IN PLANTS AND ANIMALS

Figure 2.48

Figure 2.49

Available for free at Connexions <http://cnx.org/content/col11410/1.3>

133

Table 2.7

Investigation: Heart Health and Measuring Heart ratePart 1: Investigating your cardiovascular �tnessAim :To investigate your heart rate before, during and after strenuous aerobic exercise.Method :

1. Work in pairs on the �eld and ensure you have a stop watch.2. One partner performs the experiment and the other records the results. Partners then swap roles.3. Take the resting pulse rate before exercising.4. One partner runs quickly around the �eld twice.5. Immediately after the run take his pulse.6. Continue to take his pulse every minute for 5 minutes.7. Record the results and plot a graph using the data pertaining to you.

Results : Record results in a table like the one indicated below

TIME HEART RATE (BEATS PER MINUTE)

Before exercise (resting)

0 min (immediately after exercise)

1 min (after exercise)

2 min

3 min

4 min

5 min

Table 2.8

Draw a line graph to illustrate your results on the following axis (show the resting pulse rate as a separatedotted line on the axis).

Mark allocation: heading [U+F0FC][U+F0FC]x-axis scale [U+F0FC]x-axis label [U+F0FC]y-axis scale [U+F0FC]y-axis label [U+F0FC]plotting graph [U+F0FC][U+F0FC][U+F0FC]neat and done in pencil [U+F0FC]Questions:1.Write a hypothesis for this investigation.2.Write down the independent variable.3.Write down the dependent variable.4.Name ONE factor that must be kept constant during this investigation.5.Write down TWO ways in which the accuracy of this investigation can beimproved.6.What conclusions can be made about your cardiovascular �tness?7.Explain why the heart rate increases during exercise?Part 2: Investigating your family's heart health:Instructions:

1. Draw up a table to record the answers to the following yes/no questions:

70http://www.sinauer.com/71http://www.whfreeman.com/

Available for free at Connexions <http://cnx.org/content/col11410/1.3>

134 CHAPTER 2. LIFE PROCESSES IN PLANTS AND ANIMALS

i. Do you smoke?ii. Are you overweight?iii. Do you exercise regularly?iv. Do you follow a healthy diet (low fat, low salt)v. Do you have your blood pressure checked regularly?vi. Do you have a family history of heart and circulatory disease?

1. Survey two adult male family member (father, grandfather or uncle) and two adult female familymembers (mother, grandmother or aunt). Include the adults' �rst name, gender, age and relationshipto you.

3.Record the results in your table. Also indicated the score they obtained:i. yes=0; no=5ii. yes=0; no=5iii. yes=5; no=0iv. yes=5; no=0v. yes=5; no=0vii. yes=0; no=54.Analyse the results by comparing the total score with the following descriptors:30 marks- you take very good care of your heart. Well done!25 marks- you take good care of your heart. Keep it up!20 marks- you take reasonably good care of your heart but need to workon a few aspects where you scored 0.15 marks- you need to take better care of your heart.0-10 marks- you do not look after your heart at all. It's time to make achange to a healthier lifestyle.Assessment Rubric

• Results

0- not done1- poorly presented.2- average presentation of results,but missing some detail.3- aver-age presentation of results, in-cluding all salient features andinformation.4- good presentationof results, but missing somedetail.5- good presentation of re-sults, including all salient fea-tures and information.

5

Table 2.9

Rich media:Khan Academy

Available for free at Connexions <http://cnx.org/content/col11410/1.3>

135

http 72 :// 73 www 74 . 75 khanacademy 76 . 77 org 78 / 79 video 80 / 81 circulatory 82 -83 system 84 - 85 and 86 - 87 the 88 - 89 heart 90 ? 91 playlist 92 = 93 Biology 94

Cardiac Magnetic Resonance imaging of Beating heart: Large magnets are used to create images of theheart inside the body, without the need for surgery.

http://upload.wikimedia.org/wikipedia/commons/7/73/Four_chamber_cardiovascular_magnetic_resonance_imaging.gif 95

View from the tophttp://commons.wikimedia.org/wiki/File:Beating_Heart_axial.gif 96

View from the sidehttp://commons.wikimedia.org/wiki/File:Cardiac_mri_ani_sagittal_bionerd.gif 97

2.3.1.11 Blood Vessels

2.3.1.11.1 Structure and functioning of arteries, veins, capillaries and valves

2.3.1.11.1.1 Arteries

• Arteries carry blood from away from the heart. The pressure created by the pumping heart forcesblood down the arteries.

• Arteries have three layers.

1. Outside layer � connective tissue2. Middle layer � smooth muscle, allows contraction of the arteries to regulate blood �ow and pressure3. Inside layer � single layer of tightly connected simple squamous endothelial cells

• The large arteries close to the heart branch into smaller arterioles (smaller arteries) and eventuallybranch into capillaries.

72http://www.khanacademy.org/video/circulatory-%20system-and-the-heart?playlist=Biology73http://www.khanacademy.org/video/circulatory-%20system-and-the-heart?playlist=Biology74http://www.khanacademy.org/video/circulatory-%20system-and-the-heart?playlist=Biology75http://www.khanacademy.org/video/circulatory-%20system-and-the-heart?playlist=Biology76http://www.khanacademy.org/video/circulatory-%20system-and-the-heart?playlist=Biology77http://www.khanacademy.org/video/circulatory-%20system-and-the-heart?playlist=Biology78http://www.khanacademy.org/video/circulatory-%20system-and-the-heart?playlist=Biology79http://www.khanacademy.org/video/circulatory-%20system-and-the-heart?playlist=Biology80http://www.khanacademy.org/video/circulatory-%20system-and-the-heart?playlist=Biology81http://www.khanacademy.org/video/circulatory-%20system-and-the-heart?playlist=Biology82http://www.khanacademy.org/video/circulatory-%20system-and-the-heart?playlist=Biology83http://www.khanacademy.org/video/circulatory-%20system-and-the-heart?playlist=Biology84http://www.khanacademy.org/video/circulatory-%20system-and-the-heart?playlist=Biology85http://www.khanacademy.org/video/circulatory-%20system-and-the-heart?playlist=Biology86http://www.khanacademy.org/video/circulatory-%20system-and-the-heart?playlist=Biology87http://www.khanacademy.org/video/circulatory-%20system-and-the-heart?playlist=Biology88http://www.khanacademy.org/video/circulatory-%20system-and-the-heart?playlist=Biology89http://www.khanacademy.org/video/circulatory-%20system-and-the-heart?playlist=Biology90http://www.khanacademy.org/video/circulatory-%20system-and-the-heart?playlist=Biology91http://www.khanacademy.org/video/circulatory-%20system-and-the-heart?playlist=Biology92http://www.khanacademy.org/video/circulatory-%20system-and-the-heart?playlist=Biology93http://www.khanacademy.org/video/circulatory-%20system-and-the-heart?playlist=Biology94http://www.khanacademy.org/video/circulatory-%20system-and-the-heart?playlist=Biology95http://upload.wikimedia.org/wikipedia/commons/7/73/Four_chamber_cardiovascular_magnetic_resonance_imaging.gif96http://commons.wikimedia.org/wiki/File:Beating_Heart_axial.gif97http://commons.wikimedia.org/wiki/File:Cardiac_mri_ani_sagittal_bionerd.gif

Available for free at Connexions <http://cnx.org/content/col11410/1.3>

136 CHAPTER 2. LIFE PROCESSES IN PLANTS AND ANIMALS

2.3.1.11.1.1.1 Capillaries

• Capillaries are little more than a single layer or endothelial cells.• Capillaries form intricate networks throughout the tissues.• They allow water, nutrients and gasses to di�use out of the blood and waste materials to di�use into

the blood.• This exchange occurs between the blood and the interstitial �uid.• The interstitial �uid is the �uid surrounding the cells.• The blood never comes into contact with the cells.• The blood and interstitial �uid exchange material, and the interstitial �uid then exchanges material

with the cells.

2.3.1.11.1.1.2 Veins

• The intricate networks formed by the capillaries eventually converge to form venules, (small veins)• The venules then converge to form veins which return the blood to the heart.• Veins only consist of two layers.

1. The outer layer is made up of connective tissue2. The inner layer is made up of endothelial cells.

2.3.1.11.1.1.3 Valves

• Once the blood has passed through the capillaries very little blood pressure remains to return bloodto the heart.

• Instead of pressure from the heart veins use a series of valves to force blood to return to the heart.• Contraction of the muscles squeezes the veins, pushing the blood through them.• The valves cause the blood to �ow in only one direction, back to the heart.

Available for free at Connexions <http://cnx.org/content/col11410/1.3>

137

Figure 2.50

Figure showing capillaries as the transition between arteries and veinshttp://en.wikipedia.org/wiki/File:Illu_capillary.jpgInteractive diagram illustrating arterial and venous structure.http://www.phschool.com/science/biology_place/biocoach/cardio2/structure.html 98

98http://www.phschool.com/science/biology_place/biocoach/cardio2/structure.html

Available for free at Connexions <http://cnx.org/content/col11410/1.3>

138 CHAPTER 2. LIFE PROCESSES IN PLANTS AND ANIMALS

Figure 2.51

FigureShows the how valves cause blood to only �ow one way though veinshttp://upload.wikimedia.org/wikipedia/commons/thumb/4/4a/Venous_valve.svg /2000px-

Venous_valve.svg.png

2.3.1.11.1.2 Capillaries

• Capillaries are little more than a single layer or endothelial cells.• Capillaries form intricate networks throughout the tissues.• They allow water, nutrients and gasses to di�use out of the blood and waste materials to di�use into

the blood.• This exchange occurs between the blood and the interstitial �uid.• The interstitial �uid is the �uid surrounding the cells.• The blood never comes into contact with the cells.• The blood and interstitial �uid exchange material, and the interstitial �uid then exchanges material

with the cells.

2.3.1.11.1.2.1 Veins

• The intricate networks formed by the capillaries eventually converge to form venules, (small veins)• The venules then converge to form veins which return the blood to the heart.• Veins only consist of two layers.

Available for free at Connexions <http://cnx.org/content/col11410/1.3>

139

1. The outer layer is made up of connective tissue2. The inner layer is made up of endothelial cells.

2.3.1.11.1.2.2 Valves

• Once the blood has passed through the capillaries very little blood pressure remains to return bloodto the heart.

• Instead of pressure from the heart veins use a series of valves to force blood to return to the heart.• Contraction of the muscles squeezes the veins, pushing the blood through them.• The valves cause the blood to �ow in only one direction, back to the heart.

Figure 2.52

Figure showing capillaries as the transition between arteries and veinshttp://en.wikipedia.org/wiki/File:Illu_capillary.jpgInteractive diagram illustrating arterial and venous structure.http://www.phschool.com/science/biology_place/biocoach/cardio2/structure.html 99

99http://www.phschool.com/science/biology_place/biocoach/cardio2/structure.html

Available for free at Connexions <http://cnx.org/content/col11410/1.3>

140 CHAPTER 2. LIFE PROCESSES IN PLANTS AND ANIMALS

Figure 2.53

FigureShows the how valves cause blood to only �ow one way though veinshttp://upload.wikimedia.org/wikipedia/commons/thumb/4/4a/Venous_valve.svg /2000px-

Venous_valve.svg.png

2.3.1.11.1.3 Veins

• The intricate networks formed by the capillaries eventually converge to form venules, (small veins)• The venules then converge to form veins which return the blood to the heart.• Veins only consist of two layers.

1. The outer layer is made up of connective tissue2. The inner layer is made up of endothelial cells.

2.3.1.11.1.3.1 Valves

• Once the blood has passed through the capillaries very little blood pressure remains to return bloodto the heart.

• Instead of pressure from the heart veins use a series of valves to force blood to return to the heart.• Contraction of the muscles squeezes the veins, pushing the blood through them.• The valves cause the blood to �ow in only one direction, back to the heart.

Available for free at Connexions <http://cnx.org/content/col11410/1.3>

141

Figure 2.54

Figure showing capillaries as the transition between arteries and veinshttp://en.wikipedia.org/wiki/File:Illu_capillary.jpgInteractive diagram illustrating arterial and venous structure.http://www.phschool.com/science/biology_place/biocoach/cardio2/structure.html 100

100http://www.phschool.com/science/biology_place/biocoach/cardio2/structure.html

Available for free at Connexions <http://cnx.org/content/col11410/1.3>

142 CHAPTER 2. LIFE PROCESSES IN PLANTS AND ANIMALS

Figure 2.55

FigureShows the how valves cause blood to only �ow one way though veinshttp://upload.wikimedia.org/wikipedia/commons/thumb/4/4a/Venous_valve.svg /2000px-

Venous_valve.svg.png

2.3.1.11.1.4 Valves

• Once the blood has passed through the capillaries very little blood pressure remains to return bloodto the heart.

• Instead of pressure from the heart veins use a series of valves to force blood to return to the heart.• Contraction of the muscles squeezes the veins, pushing the blood through them.• The valves cause the blood to �ow in only one direction, back to the heart.

Available for free at Connexions <http://cnx.org/content/col11410/1.3>

143

Figure 2.56

Figure showing capillaries as the transition between arteries and veinshttp://en.wikipedia.org/wiki/File:Illu_capillary.jpgInteractive diagram illustrating arterial and venous structure.http://www.phschool.com/science/biology_place/biocoach/cardio2/structure.html 101

101http://www.phschool.com/science/biology_place/biocoach/cardio2/structure.html

Available for free at Connexions <http://cnx.org/content/col11410/1.3>

144 CHAPTER 2. LIFE PROCESSES IN PLANTS AND ANIMALS

Figure 2.57

FigureShows the how valves cause blood to only �ow one way though veinshttp://upload.wikimedia.org/wikipedia/commons/thumb/4/4a/Venous_valve.svg /2000px-

Venous_valve.svg.png

2.3.1.11.2 Di�erences between arteries and veins

Artery Vein

Small lumen Large lumen

Blood under high pressure Blood under low pressure

Valves absent Valves present

Carries blood away from heart Carries blood towards the heart

Carries oxygenated blood except pulmonary artery Carries deoxygenated blood except pulmonary vein

Table 2.10

2.3.1.11.3 Indigenous Knowledge Systems

Use and symbology of blood and heart in traditional black culture

Available for free at Connexions <http://cnx.org/content/col11410/1.3>

145

2.3.1.12 Fun facts about your heart

1. The average adult heart beats:

• 72 times a minute• 100,000 times a day• 3,600,000 times a year• A billion times during a lifetime.

1. Each day your heart creates enough energy to drive a truck for 32 kilometres.2. Your left lung is smaller than your right one to make room in your chest cavity for your heart.3. Clench your �st - the size of your �st is more or less the size of your heart.4. Laughing is good exercise for your heart. Whenever you laugh, the blood �ow in your heart is increased,

keeping your heart healthy.

Investigation: Practical investigation of sheep's heartVideo: Doing a dissection

http://www.hometrainingtools.com/images/videos/Dissection_Video/dissection_�vplayer.html?TB_iframe=true&height=390&width=405 102

Equipment:

Figure 2.58

• 1 sheep heart• Cutting board• Scalpel• textbook

• Cotton• water• funnel• scissors

Table 2.11

1. EXTERNAL(a)How would you describe the general shape of the heart?(b)Note the grooves on the surface of the heart. In which direction do they run.What do you observe in these grooves.(c)Identify the atria and ventricles. How do they di�er from each other inappearance. What di�erence do you notice between the atria and ventricles.2. If the venae cavae are su�ciently long, insert a funnel into the superior vena cava and tie o�

the inferior vena cava with a piece of cotton . When water is added through the superior vena caveinto the right atrium:

(a)What happens to the wall of the right ventricle?(b)Press the right ventricle. What do you observe?

102http://www.hometrainingtools.com/images/videos/Dissection_Video/dissection_�vplayer.html?TB_iframe=true&height=390&width=405

Available for free at Connexions <http://cnx.org/content/col11410/1.3>

146 CHAPTER 2. LIFE PROCESSES IN PLANTS AND ANIMALS

(c)Release the pressure. What happens?(d)Now press the left ventricle a few times. What do you notice?(e)Now attach funnel to one of the pulmonary veins and tie o� the others(if possible). Pour water down the funnel and press the left ventricle.What do you observe?(f)Release the pressure and press the right ventricle. What do you observe?Remove the funnel and tubes.3. Cut the superior vena cava from the atrium and cut open the wall of the atrium. Dothe same

with the pulmonary vein and left atrium.(a)Describe the appearance of the inner atrial surface.(b)Determine the position of the pulmonary artery and the aorta by inserting aglass rod through these vessel into the chambers of the heart.Name the artery that leaves the right ventricle.Name the artery that leaves the left ventricle.4. Make an incision in the right side of the left ventricle from the oblique groove to the a

pex of the heart.(a)What do you observe between the left atrium and left ventricle?(b)How many �aps do you see?(c)What is the function of these �aps?5. Similarly, make an incision in the left wall of the right ventricle from the oblique groove.(a)How many �aps do you see between the atrium and the ventricle?(b)What do these �aps collectively form?6. Compare the muscular walls of the:(a) atria and the ventricles(b) left and right ventricles7. What do you observe between the two halves of the heart.8. Examine the tendinous cords .(a)Where are their points of attachment?(b)What is their function9. If the pulmonary artery and aorta are long enough, do this question. Using a funnel, pour water

into the pulmonary artery and the aorta.(a)What do you notice?(b)What do you see at the base of these arteries?10. Cut the aorta and pulmonary arteries open longitudinally and examine the valves.(a)How many parts are there to each of these valves?(b)Compare the walls of the aorta and the pulmonary artery and suggest areason for any di�erence you many �nd.

2.3.2 2.3.2 Lymph circulatory system103

2.3.2.1 Lymph Circulatory System

Comparison of lymph and blood system: http://www.lymphnotes.com/article.php/id/150/ 104

Short videos on lymph: http://www.nlm.nih.gov/medlineplus/ency/article/002247.htm 105

YouTube video on lymph system: http://www.youtube.com/watch?v=qTXTDqvPnRk 106

103This content is available online at <http://cnx.org/content/m43110/1.1/>.104http://www.lymphnotes.com/article.php/id/150/105http://www.nlm.nih.gov/medlineplus/ency/article/002247.htm106http://www.youtube.com/watch?v=qTXTDqvPnRk

Available for free at Connexions <http://cnx.org/content/col11410/1.3>

147

2.3.2.1.1 Composition of the lymphatic system

• The lymphatic system is composed of lymph vessels, lymph nodes, and organs.• The organs associated with the lymphatic system are the spleen and thymus.• The spleen is the boundary between the blood and the lymphatic system.• Knots of lymphatic tissue in the spleen add lymphocytes to the blood.• The spleen also acts as a �lter for the blood, and helps to destroy worn out red-blood cells.• In the event of damage to the spleen, it can be removed and its functions will be carried out by the

liver, the bone marrow and the lymph nodes.• Most of the disease-�ghting function of the adult mammal is carried out by the lymph nodes which

occur along the lymph ducts.• Lymph nodes are small irregularly shaped masses through which lymph vessels �ow.• Clusters of nodes occur in the armpits, groin, and neck.• Cells of the immune system line channels through the nodes and attack bacteria and viruses traveling

in the lymph so they basically act as tiny �lters.

So. . .

• Lymph vessels are located as a network throughout all tissues in the body.• Lymph vessels assist the circulatory system and all the cells of the body by removing wastes, germs

and excess water from the tissue �uid.• Lymph vessels carry lymph �uid in ONE direction only , from the bottom of the body up towards

the heart.• Valves prevent the lymph �uid from �owing backwards.• Muscle contractions push the �uid upwards.

2.3.2.1.2 Functions of lymphatic system

The main functions of the lymphatic system are as follows:

• the main function of the lymphatic system is to collect and transport tissue �uids from theintercellular spaces in all the tissues of the body, back to the veins in the blood system;

• it plays an important role in returning plasma proteins to the bloodstream ;• digested fats are absorbed and then transported from the villi in the small intestine to the

bloodstream via the lacteals and lymph vessels.• new lymphocytes are manufactured in the lymph nodes ;• antibodies and anti (manufactures in the lymph nodes) assist the body to build up an e�ective

immunity to infectious diseases ;• lymph nodes play an important role in the defence mechanism of the body . They �lter out

micro-organisms (such as bacteria) and foreign substances such as toxins, etc.• it transports large molecular compounds (such as enzymes and hormones) from their manufactured

sites to the bloodstream.

Youtube videos on lymph http://www.youtube.com/embed/Kh-XdNnTZUo107

2.3.2.1.3 Relationship between blood system and lymphatic system

Table of comparison between the cardiovascular system and the lymphatic system

107http://www.youtube.com/embed/Kh-XdNnTZUo

Available for free at Connexions <http://cnx.org/content/col11410/1.3>

148 CHAPTER 2. LIFE PROCESSES IN PLANTS AND ANIMALS

Cardiovascular System (Blood) Lymphatic System (Lymph)

Blood is responsible for collecting and distributingoxygen, nutrients and hormones to the tissues ofentire body.

Lymph is responsible for collecting and removingwaste products left behind in the tissues.

Blood �ows in the arteries, capillaries, and veins. Lymph �ows in an open circuit from the tissuesinto lymphatic vessels.

Blood �ows towards the heart and away from theheart

Lymph �ows in one direction only (towards theheart)

Blood is pumped by the heart to all parts of thebody

Lymph is not pumped. It passively �ows from thetissues into the lymph capillaries.

Blood consists of the liquid plasma that transportsthe red and white blood cells and platelets.

Lymph that has been �ltered and is ready to re-turn to the cardiovascular system is a clear or milkywhite �uid.

Blood is visible and damage to blood vessels causesobvious signs such as bleeding or bruising.

Lymph is invisible and damage to the lymphaticsystem is di�cult to detect until swelling occurs.

Blood is �ltered by the kidneys. Lymph is �ltered by lymph nodes located through-out the body..

Table 2.12

2.3.3 2.3.3 Diseases of the heart and circulatory system108

2.3.3.1 Diseases of the heart and circulatory system

Khan: Heart diseases and heart attackshttp://www.khanacademy.org/video/heart-disease-and-heart- attacks?playlist=Biology 109

Khan: Thrombo-emboli and Thromboembolismshttp://www.khanacademy.org/video/thrombo-emboli-and- thromboembolisms?playlist=Biology 110

Khan: Stenosis, ischemia and heart failurehttp://www.khanacademy.org/video/stenosis�ischemia-and-heart- failure?playlist=Biology 111

2.3.3.2 Cardiovascular diseases

• Cardiovascular disorders are a major cause of death in modern societies like heart attacks andstrokes .

• Angina is caused when the heart muscle receives an inadequate supply of blood, resulting in verypainful cramps in the chest area.

2.3.3.2.1 Heart attack

Cardiac muscle cells are provided with oxygenated blood by a system of coronary arteries . Duringexercise the �ow through these arteries is up to �ve times normal �ow. Blocked �ow in coronary arteries canresult in death of heart muscle, leading to a heart attack .

Blockage of coronary arteries is usually the result of gradual buildup of lipids and cholesterol in the innerwall of the coronary artery. Occasional chest pain can result during periods of stress or physical exercise.

108This content is available online at <http://cnx.org/content/m43156/1.1/>.109http://www.khanacademy.org/video/heart-disease-and-heart-attacks?playlist=Biology110http://www.khanacademy.org/video/thrombo-emboli-and-thromboembolisms?playlist=Biology111http://www.khanacademy.org/video/stenosis�ischemia-and-heart-failure?playlist=Biology

Available for free at Connexions <http://cnx.org/content/col11410/1.3>

149

Angina indicates that oxygen needs are greater than the ability to deliver it and that a heart attack mayoccur in the future. Heart muscle cells that die are not replaced since heart muscle cells do not divide. Heartdisease and coronary artery disease are the leading causes of death.

The �gure below shows development of arterial plaque. Images from Purves et al., Life: The Scienceof Biology , 4th Edition, by Sinauer Associates ( www.sinauer.com 112 ) and WH Freeman (www.whfreeman.com 113 ), used with permission.

Figure 2.59

continued on next page

Available for free at Connexions <http://cnx.org/content/col11410/1.3>

150 CHAPTER 2. LIFE PROCESSES IN PLANTS AND ANIMALS

Figure 2.60

continued on next page

Available for free at Connexions <http://cnx.org/content/col11410/1.3>

151

Figure 2.61

Table 2.13

2.3.3.2.2 Hypertension

Hypertension , high blood pressure (the silent killer), occurs when blood pressure is consistentlyabove 140/90. Causes in most cases are unknown, although stress, obesity, high salt intake, and smokingcan add to a genetic tendency. Luckily, when diagnosed, the condition is usually treatable with medicinesand diet and exercise

112http://www.sinauer.com/113http://www.whfreeman.com/

Available for free at Connexions <http://cnx.org/content/col11410/1.3>

152 CHAPTER 2. LIFE PROCESSES IN PLANTS AND ANIMALS

Figure 2.62

2.3.3.2.3 Stroke

What Is a Stroke? What Causes a Stroke?A stroke is a condition where a blood clot or burst artery or blood vessel interrupts blood �ow to an

area of the brain. A lack of oxygen and glucose (sugar) �owing to the brain leads to the death of brain cellsand brain damage, often resulting in an impairment in speech, movement, and memory.

The outcome after a stroke depends on where the stroke occurs and how much of the brain is a�ected.Smaller strokes may result in small problems, such as weakness in an arm or leg. Larger strokes may lead toparalysis or death. Many stroke patients are left with weakness on one side of the body, di�culty speaking,and bladder problems.

Available for free at Connexions <http://cnx.org/content/col11410/1.3>

153

2.3.4 2.3.4 Summary114

2.3.4.1 Summary

• Nutrients and oxygen are required by cells for cellular respiration. These are transported by blood tothe various cells.

• Carbon dioxide and other wastes need to be transported from the cells to the exterior. This is alsotransported via blood.

• Blood is pumped through the heart under high pressure to the various parts of the body.• The heart is divided into the left side of the heart and the right side of the heart.• The left side receives deoxygenated blood from the body via veins and sends it to the lungs to be

oxygenated.• The right side of the heart receives oxygenated blood from the lungs and sends it via arteries to all

parts of the body.• The lymphatic system is composed of lymph vessels, lymph nodes, and organs.• Lymph vessels assist the circulatory system and all the cells of the body by removing wastes, germs

and excess water from the tissue �uid.

114This content is available online at <http://cnx.org/content/m43080/1.1/>.

Available for free at Connexions <http://cnx.org/content/col11410/1.3>

154 CHAPTER 2. LIFE PROCESSES IN PLANTS AND ANIMALS

Available for free at Connexions <http://cnx.org/content/col11410/1.3>

Chapter 3

Environmental studies

3.1 Biospheres to ecosystems

3.1.1 3.1.1 - Biosphere1

3.1.1.1 Concept of the Biosphere

In the past scientists have studied the various parts of the Earth. They have looked at botany (plants),zoology (animals), geology (rocks), and physics (forces), but few have studied how all of these work together.Now we are discovering that the Earth is much more than a bunch of parts. It is a whole. The Earth is awhole system that works together. This means that there is an interconnection between all of Earth's livingand non-living parts. Everything works together in important ways. Scientists divide the Earth's Systeminto four sub-systems:•biosphere (life)•lithosphere (land)•hydrosphere (water)•atmosphere (air)To see how the sub-systems of the Earth interact, watch the video: The Earth as a System:

http://www.oercommons.org/courses/earth-as-a-system/view 2

3.1.1.1.1 Biosphere

From: http://cnx.org/content/m16693/latest/?collection=col10548/latestThe biosphere is the region of the earth that encompasses all living organisms: plants, animals and

bacteria. It is a feature that distinguishes the earth from the other planets in the solar system. "Bio"means life, and the term biosphere was �rst coined by a Russian scientist (Vladimir Vernadsky) in the1920s. Another term sometimes used is ecosphere ("eco" meaning home). The biosphere includes the outerregion of the earth (the lithosphere) and the lower region of the atmosphere (the troposphere). It alsoincludes the hydrosphere, the region of lakes, oceans, streams, ice and clouds comprising the earth's waterresources. Traditionally, the biosphere is considered to extend from the bottom of the oceans to the highestmountaintops, a layer with an average thickness of about 20 kilometers. Scientists now know that someforms of microbes live at great depths, sometimes several thousand meters into the earth's crust.

Nonetheless, the biosphere is a very tiny region on the scale of the whole earth, analogous to the thicknessof the skin on an apple. The bulk of living organisms actually live within a smaller fraction of the biosphere,from about 500 meters below the ocean's surface to about 6 kilometers above sea level.

1This content is available online at <http://cnx.org/content/m43182/1.1/>.2http://www.oercommons.org/courses/earth-as-a-system/view

Available for free at Connexions <http://cnx.org/content/col11410/1.3>

155

156 CHAPTER 3. ENVIRONMENTAL STUDIES

Dynamic interactions occur between the biotic region (biosphere) and the abiotic regions (atmosphere,lithosphere and hydrosphere) of the earth. Energy, water, gases and nutrients are exchanged between theregions on various spatial and time scales. Such exchanges depend upon, and can be altered by, the envi-ronments of the regions. For example, the chemical processes of early life on earth (e.g. photosynthesis,respiration, carbonate formation) transformed the reducing ancient atmosphere into the oxidizing (free oxy-gen) environment of today. The interactive processes between the biosphere and the abiotic regions workto maintain a kind of planetary equilibrium. These processes, as well as those that might disrupt thisequilibrium, involve a range of scienti�c and socioeconomic issues.

The study of the relationships of living organisms with one another and with their environment is thescience known as ecology. The word ecology comes from the Greek words oikos and logos, and literally means"study of the home." The ecology of the earth can be studied at various levels: an individual (organism),a population, a community, an ecosystem, a biome or the entire biosphere. The variety of living organismsthat inhabit an environment is a measure of its biodiversity.

3.1.1.1.2 Lithosphere

From: http://www.curriki.org/xwiki/bin/view/Coll_NROCscience/Lesson14TheLithosphereandPlateTectonics_0 3

The layer of the mantle above the asthenosphere plus the entire crust make up a region called thelithosphere. The lithosphere, and therefore, the earth's crust, is not a continuous shell, but is broken intoa series of plates that independently "�oat" upon the asthenosphere, much like a raft on the ocean. Theseplates are in constant motion, typically moving a few centimeters a year, and are driven by convection inthe mantle. The scienti�c theory that describes this phenomenon is called plate tectonics. According to thetheory of plate tectonics, the lithosphere is comprised of some seven major plates and several smaller ones.Because these plates are in constant motion, interactions occur where plate boundaries meet.

http://www.�ickr.com/photos/izzymunchted/3436486360/sizes/l/in/photostream/ 4

Figure 3.1

3.1.1.1.3 Hydrosphere

From Open Source Earth Science Course ( www.opencollegetextbook.org 5 )The Hydrosphere contains all the water on Earth. As groundwater, the hydrosphere penetrates the soil

as far down as bedrock, mostly limestone, or other impermeable layers. It is found in aquifers as groundwaterand also between soil particles. As surface water, it is found in wetlands, marshes, estuaries, lakes, streams,rivers, lakes, seas, and oceans. In the atmosphere, water is found as a gas throughout the di�erent regions.Water appears to permeate all the other spheres.

3http://www.curriki.org/xwiki/bin/view/Coll_NROCscience/Lesson14TheLithosphereandPlateTectonics_04http://www.�ickr.com/photos/izzymunchted/3436486360/sizes/l/in/photostream/5http://www.opencollegetextbook.org/

Available for free at Connexions <http://cnx.org/content/col11410/1.3>

157

The Hydrosphere extends upward to about 15 kilometers in the Earth's atmosphere and downward todepths on the order of �ve kilometers in its crust. Indeed, the abundance of water on Earth is a uniquefeature that clearly distinguishes our "Blue Planet" from others in the solar system. Not a drop of liquidwater can be found anywhere else in the solar system.

Though it cannot be found on any other planet, water is the most abundant inorganic substance at thesurface of the Earth. About 1.4 billion cubic kilometers of water in liquid and frozen form make up theoceans, lakes, rivers, streams, glaciers, and groundwater.

Figure 3.2

Figure 3.3

Available for free at Connexions <http://cnx.org/content/col11410/1.3>

158 CHAPTER 3. ENVIRONMENTAL STUDIES

Figure 3.4

Figure 3.5

3.1.1.1.4 Atmosphere

The atmosphere, the gaseous layer that surrounds the earth, formed over four billion years ago. The earth'satmosphere extends outward to about 1,000 kilometers where it transitions to interplanetary space. However,most of the mass of the atmosphere (greater than 99 percent) is located within the �rst 40 kilometers. Thesun and the earth are the main sources of radiant energy in the atmosphere. The sun's radiation spans theinfrared, visible and ultraviolet light regions, while the earth's radiation is mostly infrared.

The vertical temperature pro�le of the atmosphere is variable and depends upon the types of radiationthat a�ect each atmospheric layer. This, in turn, depends upon the chemical composition of that layer(mostly involving trace gases). Based on these factors, the atmosphere can be divided into four distinctlayers: the troposphere, stratosphere, mesosphere, and thermosphere.

The troposphere is the atmospheric layer closest to the earth's surface. It extends about 8 - 16 kilometersfrom the earth's surface. The thickness of the layer varies a few km according to latitude and the season ofthe year. It is thicker near the equator and during the summer, and thinner near the poles and during thewinter. The troposphere contains the largest percentage of the mass of the atmosphere relative to the otherlayers. It also contains some 99 percent of the total water vapor of the atmosphere.

The temperature of the troposphere is warm (roughly 17º C) near the surface of the earth. This is due tothe absorption of infrared radiation from the surface by water vapor and other greenhouse gases (e.g. carbon

Available for free at Connexions <http://cnx.org/content/col11410/1.3>

159

dioxide, nitrous oxide and methane) in the troposphere. The concentration of these gases decreases withaltitude, and therefore, the heating e�ect is greatest near the surface. The temperature in the tropospheredecreases at a rate of roughly 6.5º C per kilometer of altitude. The temperature at its upper boundary isvery cold (roughly -60º C).

Because hot air rises and cold air falls, there is a constant convective overturn of material in the tropo-sphere. Indeed, the name troposphere means �region of mixing.� For this reason, all weather phenomenaoccur in the troposphere. Water vapor evaporated from the earth's surface condenses in the cooler upperregions of the troposphere and falls back to the surface as rain. Dust and pollutants injected into the tro-posphere become well mixed in the layer, but are eventually washed out by rainfall. The troposphere istherefore self cleaning.

A narrow zone at the top of the troposphere is called the tropopause. It e�ectively separates the under-lying troposphere and the overlying stratosphere. The temperature in the tropopause is relatively constant.Strong eastward winds, known as the jet stream, also occur here.

The stratosphere is the next major atmospheric layer. This layer extends from the tropopause (roughly12 kilometers) to roughly 50 kilometers above the earth's surface. The temperature pro�le of the stratosphereis quite di�erent from that of the troposphere. The temperature remains relatively constant up to roughly25 kilometers and then gradually increases up to the upper boundary of the layer. The amount of watervapor in the stratosphere is very low, so it is not an important factor in the temperature regulation of thelayer. Instead, it is ozone (O3) that causes the observed temperature inversion.

The third layer in the earth's atmosphere is called the mesosphere. It extends from the stratopause (about50 kilometers) to roughly 85 kilometers above the earth's surface. Because the mesosphere has negligibleamounts of water vapor and ozone for generating heat, the temperature drops across this layer. It is warmedfrom the bottom by the stratosphere. The air is very thin in this region with a density about 1/1000 that ofthe surface. With increasing altitude this layer becomes increasingly dominated by lighter gases, and in theouter reaches, the remaining gases become strati�ed by molecular weight.

The fourth layer, the thermosphere, extends outward from about 85 kilometers to about 600 kilometers.Its upper boundary is ill de�ned. The temperature in the thermosphere increases with altitude, up to 1500ºC or more. The high temperatures are the result of absorption of intense solar radiation by the last remainingoxygen molecules. The temperature can vary substantially depending upon the level of solar activity.

The lower region of the thermosphere (up to about 550 kilometers) is also known as the ionosphere.Because of the high temperatures in this region, gas particles become ionized. The ionosphere is importantbecause it re�ects radio waves from the earth's surface, allowing long-distance radio communication. Thevisual atmospheric phenomenon known as the northern lights also occurs in this region. The outer regionof the atmosphere is known as the exosphere. The exosphere represents the �nal transition between theatmosphere and interplanetary space. It extends about 1000 kilometers and contains mainly helium andhydrogen. Most satellites operate in this region.

Available for free at Connexions <http://cnx.org/content/col11410/1.3>

160 CHAPTER 3. ENVIRONMENTAL STUDIES

Figure 3.6

Figure 3.7

3.1.1.1.5 Interconnectedness with, and components of a global ecosystem

Concept: the earth is a systemText from Open Source Earth Science CourseWhile studying the parts of the Earth System it is important to look for the emergent properties of the

Earth System. How do the parts of the Earth System come together to form a sum that is greater than thesum of its parts? This question is best answered by focusing on the Earth's matter, energy, and life.

A system has two distinguishing characteristics. The �rst is that it has SYNERGY. Synergy means thatthe whole is greater than the sum of the parts. This sounds a lot more complicated than it is. What itmeans is that when all of the pieces of a system are put together they are more valuable than all of the pieceswould be if they were considered separately. A home is a good example. If you were to lay all the pieces and

Available for free at Connexions <http://cnx.org/content/col11410/1.3>

161

parts of your home in a pile you would have a big pile of wood, insulation, pipes, wires, drywall, etc. Yourpile of �house stu�� would be worth something but not nearly as much as your home is worth when all the�house stu�� is organized into a system.

The second distinguishing characteristic of a system is that it has EMERGENT PROPERTIES. Emergentproperties are properties that emerge as a result of how the system works together; properties that do notexist without the system. In other words, emergent properties are characteristics that are unique to thesystem as a whole. Let us consider the example of your home once again. Some emergent properties of yourhome may be its comfort and its safety. The comfort of your home is a function of the materials used tobuild it, the architectural design, and the furniture inside. The home's safety is a property dependent on thedesign, the strength and location of its doors and windows, and the neighbourhood in which it was built.Both the safety and comfort of your home are properties of the home that are a result of the �home system�;they are not dependent on just one aspect of the home.

Text from Earth as a System. " Teachers' Domain. 17 Dec. 2005. Web. 15 Oct. 2011.http://www.teachersdomain.org/resource/ess05.sci.ess.earthsys.hologlobe/�

Understanding our planet as an integrated system of components and processes is a fundamental part ofEarth and space science research. Just as the human body is composed of interrelated systems that controlspeci�c bodily functions, Earth's four principal components � the atmosphere (air), lithosphere (land),hydrosphere (water), and biosphere (life) � perform critical roles that, together, support and sustain life onthe planet.

Nothing in�uences the subsystems that contribute to Earth's dynamic behaviour more than heat. Heatcomes from two sources: solar energy and radioactivity in the Earth's core. Because of the angle at whichthe Sun strikes Earth, Earth's surface is heated unevenly. This creates Earth's three major climate zones �tropical, temperate, and polar � which then in�uence what types of life �ourish in di�erent locations.

The uneven heating also controls weather systems. The heat absorbed by the oceans and carried byits currents is constantly being released into the atmosphere. This heat and moisture drive atmosphericcirculation and set weather patterns in motion. The weather patterns then in�uence vegetation, as well aserosion and sediment transport.

The other heat source, deep within Earth's core, is responsible for plate tectonics, which gives the Earthits physical character: mountain ranges and valleys, ocean basins and lake beds, and islands and trenches.The heat from Earth's core generates convection cells within its mantle, which help drive plate activity.

Ever since the �rst photos were sent back from space, our view of Earth has changed. Remote sensinginstruments, such as satellites, allow us to better understand the interrelationships between the di�erentsubsystems. For instance, recordings made by remote and Earth-based instruments show that signi�cantsurface warming has occurred over the past three decades. Knowing this, scientists are working to determinehow this will a�ect � and already is a�ecting � the entire Earth system.

Possible slide-shows:http://www.slideshare.net/Alyssa10/earth-science-biosphere-ppt 6

http://www.slideshare.net/shoreyl/3-biosphere 7

Video: The Earth as a System: http://www.oercommons.org/courses/earth-as-a-system/view 8

QuestionsWhat are the parts of Earth's System?What are the properties of the Earth's System?How is the Earth's System part of a larger system?

6http://www.slideshare.net/Alyssa10/earth-science-biosphere-ppt7http://www.slideshare.net/shoreyl/3-biosphere8http://www.oercommons.org/courses/earth-as-a-system/view

Available for free at Connexions <http://cnx.org/content/col11410/1.3>

162 CHAPTER 3. ENVIRONMENTAL STUDIES

3.1.2 3.1.2 Biomes9

3.1.2.1 Biomes

3.1.2.1.1 Key concepts

In this session we will focus on summarising what you need to know about:

• Terrestrial and aquatic biomes of Southern Africa.• How climate, soil and vegetation in�uence the organisms found in each.• Location of the di�erent biomes in South Africa.

3.1.2.1.2 Terminology & De�nitions

Biomes can be de�ned as the major climatic regions of the world, classi�ed according to their predominantvegetation and characterised by adaptations of organisms to that particular environment.

3.1.2.1.3 Content

The biosphere can be divided into relatively large regions called biomes. A biome has a distinct climateand certain living organisms (especially vegetation) characteristic to the region and may contain manyecosystems. The key factors determining climate are average annual precipitation and temperature. Thesefactors, in turn, depend on the geography of the region, such as the latitude and altitude of the region, andmountainous barriers. The major types of biomes include: aquatic , desert , forest , grassland andtundra . Biomes have no distinct boundaries. Instead, there is a transition zone called an ecotone, whichcontains a variety of plants and animals. For example, an ecotone might be a transition region between agrassland and a desert, with species from both.

You will be required to learn about terrestrial and aquatic biomes.

3.1.2.1.4 Aquatic Biomes

Water covers a major portion of the earth's surface, so aquatic biomes contain a rich diversity of plants andanimals. Aquatic biomes can be subdivided into two basic types: freshwater and marine .

A freshwater region has a low salt concentration, usually less than 1 percent, and occurs in severaltypes of regions: ponds and lakes, streams and rivers, and wetlands.

• Ponds and lakes range in size, and small ponds may be seasonal. They sometimes have limited speciesdiversity due to isolation from other water environments. They can get their water from precipitation,surface runo�, rivers, and springs.

• Streams and rivers are bodies of �owing water moving in one general direction (i.e., towards the sea).Streams and rivers start at their upstream headwaters, which could be springs, snowmelt or even lakes.They continue downstream to their mouths, which may be another stream, river, lake or ocean. Theenvironment of a stream or river may change along its length, ranging from clear, cool water near thehead, to warm, sediment-rich water near the mouth. The greatest diversity of living organisms usuallyoccurs in the middle region.

• Wetlands are places of still water that support aquatic plants, such as cattails, pond lilies andcypress trees. Types of wetlands include marshes, swamps and bogs. Wetlands have the highestdiversity of species with many species of birds, mammals, amphibians and reptiles. Some wetlands,such as salt marshes, are not freshwater regions.

Marine regions cover nearly three-fourths of the earth's surface. Marine bodies are salty, having approx-imately 35 grams of dissolved salt per litre of water (3.5 percent). Oceans are very large marine bodies

9This content is available online at <http://cnx.org/content/m43240/1.1/>.

Available for free at Connexions <http://cnx.org/content/col11410/1.3>

163

that dominate the earth's surface and hold the largest ecosystems. They contain a rich diversity of livingorganisms. Ocean regions can be separated into four major zones: intertidal, pelagic, benthic and abyssal.

• The intertidal zone is where the ocean meets the land. Sometimes, it is submerged and at other timesexposed, depending upon waves and tides.

• The pelagic zone includes the open ocean further away from land.• The benthic zone is the region below the pelagic zone, but not including the very deepest parts of

the ocean. The bottom of this zone consists of sediments.• The deepest parts of the ocean are known as the abyssal zone . This zone is very cold (near freezing

temperatures), and under great pressure from the overlying mass of water. Mid-ocean ridges occur onthe ocean �oor in abyssal zones.

Figure 3.8

Figure Showing zonation(Source from http://studentweb.cortland.edu/knowles86/Intertidalzone.gif 10 )Coral reefs are found in the warm, clear, shallow waters of tropical oceans around islands or along

continental coastlines. They are mostly formed from calcium carbonate produced by living coral. Reefsprovide food and shelter for other organisms and protect shorelines from erosion.

10http://studentweb.cortland.edu/knowles86/Intertidalzone.gif

Available for free at Connexions <http://cnx.org/content/col11410/1.3>

164 CHAPTER 3. ENVIRONMENTAL STUDIES

Figure 3.9

Coral Reef. (Source: Coral Reef Alliance Photobank)Estuaries are partially enclosed areas where fresh water and silt from streams or rivers mix with

salty ocean water. They represent a transition from land to sea and from freshwater to saltwater. Estuariesare biologically very productive areas and provide homes for a wide variety of plants, birds and animals.

3.1.2.1.5 Terrestrial Biomes

Terrestrial biomes characterise ecosystems on land, and are usually identi�ed by the growth form of thedominant vegetation, climate, and/or where they are located on the earth. The major terrestrial biomesinclude the tundra biome , the forest biome , the grassland biome , and the desert biome . Note thatforests and grasslands are de�ned based on the growth form of the dominant vegetation whereas deserts areclassi�ed based on the dominant climatic conditions. The geographic distribution of terrestrial biomes ismostly in�uenced by climatic conditions such as rainfall and temperature. The most recent classi�cation ofthe biomes in South Africa divides the region into the following seven biomes:

• Grassland• Savanna• Succulent Karoo• Nama Karoo• Forest• Fynbos• Desert.• Thicket

Available for free at Connexions <http://cnx.org/content/col11410/1.3>

165

Figure 3.10

This map shows the di�erent biomes of South Africa(Source from http://cnx.org/content/m20153/latest/graphics1.png )

3.1.2.1.6 Grassland

Grasslands cover regions where moderate rainfall is su�cient for the growth of grasses, but not enough forstands of trees. There are two main types of grasslands: tropical grasslands (savannas) and temperategrasslands . Tropical grasslands occur in warm climates such as Africa and very limited regions of Australia.They have a few scattered trees and shrubs, but their distinct rainy and dry seasons prevent the formation oftropical forests. Most temperate grasslands are treeless, relatively �at and have rich soil, have been replacedby farmland.

Available for free at Connexions <http://cnx.org/content/col11410/1.3>

166 CHAPTER 3. ENVIRONMENTAL STUDIES

Figure 3.11

(Source from http://www.�ickr.com/photos/takver/5884439290/sizes/o/in/photostream 11 )The information shown below shows the e�ect of climate change on the grassland biome

11http://www.�ickr.com/photos/takver/5884439290/sizes/o/in/photostream

Available for free at Connexions <http://cnx.org/content/col11410/1.3>

167

Figure 3.12Available for free at Connexions <http://cnx.org/content/col11410/1.3>

168 CHAPTER 3. ENVIRONMENTAL STUDIES

Figure Reduction in grassland biome(Source from http://maps.grida.no/go/graphic/changing_biomes_in_south_africa 12 )Activity: Write down the advantages and disadvantages of burning grassland,

ADVANTAGES OF BURNING DISADVANTAGES OF BURNING

Table 3.1

Table 1See memorandum at the end of this section:

3.1.2.1.7 Savanna

The Savanna Biome is the largest biome in Southern Africa. Mainly found in Mpumalanga and Limpopoprovinces but also in the coastal belt of KwaZulu Natal and the Eastern Cape Province. Summers are hotand wet and the winters are cool with little or no rain. This biome is also known as the bushveld wheregrasses are mainly found and regular �res prevent the trees from dominating.

12http://maps.grida.no/go/graphic/changing_biomes_in_south_africa

Available for free at Connexions <http://cnx.org/content/col11410/1.3>

169

Figure 3.13

(Source from http://edu.glogster.com/media/2/11/47/83/11478364.jpg 13 )

3.1.2.1.8 Succulent Karoo

The Succulent Karoo biome can be found in the west coast of the Northern Cape Province and the northernparts of the Western Cape Province. This biome is hot in summer and cold in winter, although the rainfallin this area is very low. 40% of plant species found here are endemic to this biome. The Namaqualand regionof this biome is famous for its colourful wild �owers. Succulent plants are able to live through dry seasonsby using water stored in their leaves or stems.

Figure 3.14

13http://edu.glogster.com/media/2/11/47/83/11478364.jpg

Available for free at Connexions <http://cnx.org/content/col11410/1.3>

170 CHAPTER 3. ENVIRONMENTAL STUDIES

(Source from http://planet.uwc.ac.za/nisl/BDC321/ekapa%20Cape%20Towns%20lowlands/biomes/images/succulentkaroo-01.jpg 14 )

Figure 3.15

This map shows the succulent karoo region within the orange and the nama karoo within the pink.(Source from http://images-mediawiki-sites.thefullwiki.org/01/2/0/1/97579812567978300.jpg 15 )

3.1.2.1.9 Nama Karoo

The Nama Karoo is the second largest biome in South Africa. It forms the major part of the Northern CapeProvince. It is regarded as a semi-desert area receiving very little rain. The summers are very hot and thewinters are very cold. The dominant vegetation type is grasses.

14http://planet.uwc.ac.za/nisl/BDC321/ekapa%20Cape%20Towns%20lowlands/biomes/images/succulentkaroo-01.jpg15http://images-mediawiki-sites.thefullwiki.org/01/2/0/1/97579812567978300.jpg

Available for free at Connexions <http://cnx.org/content/col11410/1.3>

171

Figure 3.16

(Source from http://www.plantzafrica.com/vegetation/vegimages/namakaroo2.jpg)

3.1.2.1.10 Forest

The forest biome in South Africa occurs in patches in areas such as Knysna of the Western Cape as well asKwaZulu Natal, the Eastern Cape, Limpopo and Mpumalanga. Some of these forests experience rain onlyin winter, while others get rainfall throughout the year.

Forests are dominated by trees of which the Yellowwood is the largest. There are many herbaceous andbulbous plants that also occur.

Available for free at Connexions <http://cnx.org/content/col11410/1.3>

172 CHAPTER 3. ENVIRONMENTAL STUDIES

Figure 3.17

(Source from http://www.�ickr.com/photos/jlascar/4460866346/sizes/o/in/photostream/ 16 Jorge Las-car's photostream)

Trees are not only producers , but as a result of their size they also create a habitat for certainspecies. The leaf cover of trees provides shelter for animals, while the bark and �ssures in the trees alsoprovide a habitat for numerous insect species. The leaf cover also creates a shady environment in whichshade-loving, low-growing plants can �ourish.

When leaves or fruit fall from the trees and collect at the feet of the trees, another series of organismscan appear. The decomposers , such as micro-organisms that cause the dead material to decay anddecompose, contribute to the decomposition of the nutrients so that they may return to the soil. Humusis formed in this way. Humus is dead organic material. Other creatures that live o� decayed organicmaterial, namely the detrivores , also promote this process of decomposition.

ACTIVITY: Do a poster to illustrate the Role-players in a Tree Ecosystem.Bring pictures of animals, trees and other plants to class. The teacher will divide the class into groups.Each group will prepare a poster to illustrate the mutual dependence of the trees, other plants and

animals. Each group must present its poster to the rest of the class.Answer the following questions / follow the instructions arising from the class discussion:QUESTIONS / INSTRUCTIONS

1. Supposing the tree on your poster was to fall over.

16http://www.�ickr.com/photos/jlascar/4460866346/sizes/o/in/photostream/

Available for free at Connexions <http://cnx.org/content/col11410/1.3>

173

a. Which organisms would die?b. Which organisms would move away?c. Which organisms would increase in number?

2. Describe the role played by trees in an ecosystem.3. Ecologically speaking, why is it bad practice to rake up leaves under trees?4. Name three more examples where humans harm ecosystems.

3.1.2.1.11 Fynbos

Fynbos grows in a 100-to-200-km-wide coastal belt stretching from Clanwilliam on the West coast to PortElizabeth on the Southeast coast � forms part of the Cape �oral kingdom , where it accounts for half ofthe surface area and 80% of the plant varieties. The fynbos in the western regions is richer and more variedthan in the eastern regions of South Africa.

Of the world's six �oral kingdoms , this is the smallest and richest per area unit. Contrast it in size withthe Holarctic kingdom, which incorporates the whole of the northern hemisphere apart from the tropicalregions. The diversity of fynbos plants is extremely high, with over 9000 species of plants occurring in thearea, around 6200 of which are endemic , i.e. they do not grow anywhere else in the world.

The Cape Fynbos is the term given to a collection of plants that are mainly shrubs and is comprised ofspecies belonging to South Africa's south western and southern Cape. Fynbos can be de�ned as a shrublandwith an unusual mixture of plant types of di�erent shapes and sizes that have been termed, "growth forms".There are four of these growth forms; the proteoids - tall protea shrubs with large leaves; the ericoids �heath-like shrubs; reed-like plants � the restoids; and bulbous herbs � the geophytes.

The mountain fynbos is found from Nieuwoudtville and Vanrhynsdorp south along the main mountainranges to Cape Hangklip and the Cape Peninsula and then eastwards on the mountains to near Grahamstown.The vegetation is characterised by ericoids (heather), restoids (reeds) and proteoid shrubs like proteas andconebushes.

Coastal fynbos is found along the western and southern coastlines of the Cape Province from sea-level toabout 150m above sea-level, where the soil is usually alkaline to sour. There are two types:

Coastal fynbos of the West Coast sands, from the Cape Flats to Redelinghuys on the West Coast, andCoastal fynbos of the south coast limestone, from Danger Point to Mossel Bay.

The Strandveld veld type is found mainly on the lower parts of the western coastal plains and could beregarded as a transitional stage between coastal fynbos and Karooveld. Some patches may be found on thesouthern coast as far as close to Port Elizabeth.

The coastal rhenosterbosveldveld type is found on the lowlands along the coast on shale and granite,from sea level to 400 m above. As a veld type, it is rich in a wide variety of species and dominated by therhenosterbush and the characteristic wealth of spring �owers.

Available for free at Connexions <http://cnx.org/content/col11410/1.3>

174 CHAPTER 3. ENVIRONMENTAL STUDIES

Figure 3.18

(Source from http://www.itmonline.org/image/honey1.gif)

Available for free at Connexions <http://cnx.org/content/col11410/1.3>

175

Figure 3.19

(Source from http://4.bp.blogspot.com/_Dt7LueKRwF0/SduqKc1ZUPI/AAAAAAAABQ0/CgbnLf8LYX8/s400/2009_03290148.jpg 17 )

ACTIVITY: Read the passage below.The astonishing richness and diversity of the Western Cape's natural resources is matched only by the

resourcefulness and diversity of its many people. Historical patterns of unsustainable use of resources haveled to the Cape Floristic Region (CFR) being listed as one of the world's threatened bioregions, and thescars are deeply etched in the land and its people.

Now the people of the Western Cape are exploring new and sustainable ways to value and bene�t fromthese globally important assets.

South Africa's Cape Floristic region is legendary, and the unique nature of the fynbos biome has beencelebrated by biologists, conservationists, development experts, and ecologist worldwide.

(Adapted from speech by Tasneem Essop the Western Cape Provincial Minister for Environment, Plan-ning and Economic Development)

Write an essay on the `Fynbos' biome and discuss the following aspects.

• What is the meaning of the term `fynbos'?

17http://4.bp.blogspot.com/_Dt7LueKRwF0/SduqKc1ZUPI/AAAAAAAABQ0/CgbnLf8LYX8/s400/2009_03290148.jpg

Available for free at Connexions <http://cnx.org/content/col11410/1.3>

176 CHAPTER 3. ENVIRONMENTAL STUDIES

• Identify features of families/indicator species that make up this vegetation type• Describe it ecological role in the environment• Biological impacts on the environment of destroying this type of vegetation.• Economical importance of it for the people of the Western Cape.• Management strategies involved in protecting it.

Your essay may be written or typed . Marks will be awarded for originality and own interpretation.Include a bibliography of three of more resources. No marks will be awarded for plagiarism.

Factual info : 5x5 (25)Synthesis: (5)Total: 30

3.1.2.1.12 Thicket

The thicket biome occurs along the coasts of KwaZulu Natal and the Eastern Cape. Most thickets occur inriver valleys. Thickets develop in areas where the rainfall is fairly high however; there may be dry periodsthat prevent the vegetation from developing into forests. The vegetation of this biome includes short tress,low intertwining shrubs and vines. There are no distinct layers of trees and shrubs with many large openspaces as found in the forest biome.

Figure 3.20

Thickets in the Eastern Cape are comprised of dense impenetrable vegetation dominated by spiny, oftensucculent trees and shrubs such as seen in this photograph taken near Uitenhage, E.Cape.

(Source from http://www.plantzafrica.com/vegetation/thicket.htm 18 )

18http://www.plantzafrica.com/vegetation/thicket.htm

Available for free at Connexions <http://cnx.org/content/col11410/1.3>

177

3.1.2.1.13 Desert

The Desert Biome is found largely as the Namib Desert along the coast of Namibia. The Deserts are dryareas where evaporation usually exceeds precipitation. Rainfall is low � less than 25 centimeters per year �and can be highly variable and seasonal. The low humidity results in temperature extremes between day andnight. Deserts can be hot or cold. Hot deserts (e.g. the Namib and Kalahari) are very hot in the summerand have relatively high temperatures throughout the year and have seasonal rainfall. This combination oflow rainfall and high temperatures keeps the air very dry, increasing its evaporating power. Deserts haverelatively little vegetation and the substrate consists mostly of sand, gravel or rocks. The transition regionsbetween deserts and grasslands are sometimes called semi-arid deserts.

Figure 3.21

Distribution map of Welwitschia mirabilis and a detail of Welwitschia mirabilis.ActivityKnowledge Are : Diversity, change and continuityTopic: Advertisements on South African BiomesYou work for an Advertising Agency that is bidding for the account of a top travel agency. The bid

includes designing a full page advert (A4) for the Getaway Magazine. Presentation, appeal and accuracywill therefore be of top priority.

(DON'T FORGET TO CHECK MAPS, REFERENCE BOOK, ADVERTS, and BROCHURES FORIDEAS. DO NOT CUT AND PASTE OR COPY OTHER PEOPLE'S WORK )

The travel agency has speci�ed that they would like the following to be included in the ad, which isgeared towards people looking for a di�erent and fascinating holiday in a speci�c biome :

• A region in the biome of your choice, including cities and/or towns worth a visit• Climate (of interest to tourists)• Well-known geographical features in the region• Mention of some interesting wildlife (i.e. birds, animals, plants) that may be seen

Available for free at Connexions <http://cnx.org/content/col11410/1.3>

178 CHAPTER 3. ENVIRONMENTAL STUDIES

• Pictures• Tour dates• The name of the travel agency, with contact information

You should also include, very discreetly, the name of your own agency . (Study some ads for ideas). Re-member � THE SKILL IS IN THE CRISP, RELEVANT AND SUCCINCT WAY IN WHICH THE INFOR-MATION IS PRESENTED.

ARE YOU THE BEST? CAN YOUR AD AGENCY WIN THE BID?We have discussed the following six biomes from which you will select one:Savannah, Forest, Nama-Karoo, Succulent-Karoo, Fynbos, Grassland.You will have two periods and homework time to investigate and complete this assignment.Mark

Name of Travel Agency and Biome /3

Climate information /2

Wildlife /2

Geographical features and towns/cities /4

Local crafts and use of resources /4

Size of advert Layout is neat. Good use of space, Font shape & size is appropriate. /4

Eye-catching. Colorful Use of diagrams/pictures, etc. /4

Use of language. Age-appropriate. Own words. /4

Only relevant information included. /4

Interesting information throughout /4

Followed all instructions /4

Adequate Bibliography supplied on back of this page /4

TOTAL/40

Mark

Name of Travel Agency and Biome /3

Climate information /2

Wildlife /2

Geographical features and towns/cities /4

Local crafts and use of resources /4

Available for free at Connexions <http://cnx.org/content/col11410/1.3>

179

Size of advert Layout is neat. Good use of space, Font shape & size is appropriate. /4

Eye-catching. Colorful Use of diagrams/pictures, etc. /4

Use of language. Age-appropriate. Own words. /4

Only relevant information included. /4

Interesting information throughout /4

Followed all instructions /4

Adequate Bibliography supplied on back of this page /4

TOTAL/40

Mark

Name of Travel Agency and Biome /3

Climate information /2

Wildlife /2

Geographical features and towns/cities /4

Local crafts and use of resources /4

Size of advert Layout is neat. Good use of space, Font shape & size is appropriate. /4

Eye-catching. Colorful Use of diagrams/pictures, etc. /4

Use of language. Age-appropriate. Own words. /4

Only relevant information included. /4

Interesting information throughout /4

Followed all instructions /4

Adequate Bibliography supplied on back of this page /4

TOTAL/40

Available for free at Connexions <http://cnx.org/content/col11410/1.3>

180 CHAPTER 3. ENVIRONMENTAL STUDIES

MARK SCHEME Mark

Name of Travel Agency and Biome /3

Climate information /2

Wildlife /2

Geographical features and towns/cities /4

Local crafts and use of resources /4

Size of advert Layout is neat. Good use of space, Font shape & size is appropriate. /4

Eye-catching. Colorful Use of diagrams/pictures, etc. /4

Use of language. Age-appropriate. Own words. /4

Only relevant information included. /4

Interesting information throughout /4

Followed all instructions /4

Adequate Bibliography supplied on back of this page /4

TOTAL /40

Table 3.2

ActivityThe following activity is to be done in groups of fourINSTRUCTIONS:Brainstorm a suitable set of criteria for assessment for poster and verbal reportSelect ONE biome from the list given and do the following:Use suitable references to obtain as much information as possible on the plant and animals found in your

selected biomeMake notes about the climate, landscape, stating how some of these are adapted to their environmentDesign an attractive poster to illustrate the landscape as well as the dominant plants and animals that

make up a food chain.Display your poster on the classroom wall andEach person of the group is to give a verbal presentation on an aspect of the biome you studied.Assessment criteria:Written:Size A2 and bigger, heading font sizeLayout � organisation aesthetic appeal, use of colour, creativity, eye- catchingVisual � drawings, diagramsInformation � relevant, factual, main pointsOral:Con�denceSubject knowledgePoiseE�ortRUBRIC FOR ORAL PRESENTATION

Available for free at Connexions <http://cnx.org/content/col11410/1.3>

181

ASSESSMENTCRITERIAPERFORMANCE INDICATORS

3 2 1 0

Poise and con�-dence

Very con�dent,stands up straight,makes good eyecontact, does notshu�e

Less con�dent,leans against desk,shu�es around

No con�dence,does not look up,nervous, shu�es

No con�dence atall, ill prepared

Communication Stimulating, clearand concise

Clear at times Not clear, slightlyconfusing

None

Body of presenta-tion

Coherent, excel-lent subject knowl-edge,informationaccurate anddetailed

Clear at times,good subjectknowledge, accu-rate but not asdetailed

Unclear, little sub-ject knowledge

No subject knowl-edge, reads wordfor word fromnotes, nonsense

Time E�ective use oftime

Too long Too short

Enthusiasm /Ef-fort

Very enthusiastic,worthwhile e�ort

Very enthusiastic,Lacks e�ort

Little e�ortLittleenthusiasm

No e�ortNo enthu-siasm

Table 3.3

(15)Memo Activity 1: Discussing the value of a grassland ecosystem

ADVANTAGES OF BURNING DISADVANTAGES OF BURNING

• Hard seedpods that cover seeds crack open • Air pollution � smoke

• Species are rediscovered, e.g., the mountainrose

• Animals and plants are injured and damaged,or killed

• Plants that grow aggressively are restricted • Organisms in the soil are destroyed � humusis reduced

continued on next page

Available for free at Connexions <http://cnx.org/content/col11410/1.3>

182 CHAPTER 3. ENVIRONMENTAL STUDIES

• Younger plants provide better nutrition(green grass after a severe winter)

• Grasses are weakened if burning is practisedor occurs at the wrong time

Table 3.4

3.1.2.1.14 Rich media

http://www.curriki.org/xwiki/bin/view/Coll_NROCscience/Lesson25VideoBiosphere 19

The video of this lesson focuses on the biosphere, ecospheres, the lithosphere, the troposphere, the hy-drosphere, ecology, individuals, populations, communities, ecosystems, biomes, biospheres, and biodiversity.

http://www.oercommons.org/courses/biomes/view 20

This interactive resource adapted from NASA features some of the physical and biological characteristicsof seven of the world's biomes.

http://www.southafrica.info/about/animals/�ora.htm#ixzz1aqVbqENT 21

This classi�cation of di�erent biomes corresponds to decreasing average temperatures.

3.1.3 3.1.3 Environment22

3.1.3.1 Concept of an environment

Throughout history humans have in�uenced and been in�uenced by the natural world. While much of ourimpact has been detrimental to the natural environment, we have preserved and protected certain resourcesthat are important to us. The environment, while highly valued by most, is used and altered by a widevariety of people with many di�erent interests and values. Di�culties remain on how best to ensure theprotection of our environment and natural resources, and how to divide the earth's resources equally so thatall bene�t, and not just a select few.

There are currently many uncertainties regarding the future of the natural environment. However it isclear that humans have had a role in its destruction and will be responsible in the future for its conservation.Environmental problems are becoming more and more complex, especially as issues arise on a more globallevel, such as that of atmospheric pollution or global warming. There is a realization that such complexproblems will demand complex solutions and the participation of all.

The challenge is to �nd approaches to environmental management that give people the quality of life theyseek while protecting the environmental systems that are also the foundations of our well being. In order toface these challenges, students today will need more than super�cial knowledge or awareness of disconnectedenvironmental issues. A multidisciplinary approach to learning can build upon the strengths of a wide rangeof �elds of study, providing a deeper understanding of the technological, political, and social options andstrategies for both studying and managing the relationship between our society and the environment. Therewill always be tradeo�s and, many times, unanticipated or unintended consequences.

3.1.3.2 Abiotic and biotic factors: e�ects on the community

There a number of characteristics of your local environment that can be classi�ed into three broad categories,which can be called the �ABC's of the environment.�

In the ABC's of the environment,A-refers to the abiotic (physical, non-living) features of the areaB- identi�es the biotic (plant and animal) component of the environment.

19http://www.curriki.org/xwiki/bin/view/Coll_NROCscience/Lesson25VideoBiosphere20http://www.oercommons.org/courses/biomes/view21http://www.southafrica.info/about/animals/�ora.htm#ixzz1aqVbqENT22This content is available online at <http://cnx.org/content/m43180/1.1/>.

Available for free at Connexions <http://cnx.org/content/col11410/1.3>

183

C-C is the cultural (human) in�uences.Some ecologists think of the ABC's as forming a triangle with inter- relating sides. In a civilization as

complex as ours, no single side can exist unin�uenced by others.

3.1.3.3 How Humans have an impact on the environment

3.1.3.3.1 The e�ect of humans on the composition of the atmosphere and the climate

This section will summarize some of the impacts that human's have had on the environment. In particularwe will look at the e�ect of humans on our abiotic (non-living) environment.

SummaryOzone Depletion: the ozone layer which prevents UV light from entering the earth's surface is being

destroyed by CFC's and pollutionGreenhouse E�ect: The UV light is being trapped within the earth's atmosphere by a �blanket� of

carbon dioxide and other greenhouse gasses, such as methane and nitrous oxide. The main greenhouse gasesare carbon dioxide and methane. They absorb much of the energy radiated from Earth and then re-radiateit back to Earth. The levels of these gases in the atmosphere are slowly rising. Carbon dioxide is producedwhen fossil fuels are burnt. Methane is released by cattle and from rice �elds. An increase of only a fewdegrees Celsius may cause big changes in the Earth's climate, upsetting weather patterns and a�ecting thetypes of crops that can be grown in di�erent parts of the world. Polar ice caps may melt, causing the sealevel to rise.

Figure 3.22

Global warming: refers to increase in earth's temperature due to the greenhouse e�ectDeforestation: removal of natural forests. This destroys biodiversity, and also removes plants which

help to decrease the carbon dioxide burden.

Available for free at Connexions <http://cnx.org/content/col11410/1.3>

184 CHAPTER 3. ENVIRONMENTAL STUDIES

Acid Rain Burning fossil fuels may also produce sulphur dioxide and nitrogen oxide gases. These gasesrise up in the atmosphere from factories, power stations and vehicles and dissolve in the water in cloudsforming acid rain. Acid rain can damage trees directly.If the water in rivers and lakes becomes too acidic,plants and animals cannot survive. Acid rain also damages buildings

Destruction of biodiversity - hunting, pollution, destruction of the natural environment and intro-duction of alien species can result in the extinction of native species.

3.1.3.3.1.1 De�nitions and terms

Greenhouse gases: Methane, nitrous oxide, carbon dioxideFossil Fuels: Natural substances made deep within the earth by the decomposition of plants and

animal remains over millions of years

3.1.3.3.1.2 The Greenhouse E�ect

http://www.curriki.org/xwiki/bin/view/Coll_Athabasca/Unit5-Lesson3TheGreenhouseE �ect 23

With the rise to prominence of the issue of global warming, it is important to discuss the greenhousee�ect. The name comes from the everyday concept of a greenhouse, where sunlight is allowed to passthrough transparent panels and shine on the plants inside. This provides energy to the plants, but alsowarms everything inside the greenhouse. With the sealed layer of transparent panels, the warmth is trappedinside and the greenhouse becomes much warmer than the environment outside.

The Earth's atmosphere functions exactly like this, except there are no transparent panels. When sunlightshines down on the Earth, most of it is absorbed on the surface, giving us warmth and energy. Some of thelight is absorbed by the atmosphere before it hits the surface, and a very small amount of the light is alsore�ected back o� the surface toward outer space. Additionally, the surface of the Earth releases heat intothe atmosphere, such as can be seen over a road on a hot day.

The greenhouse e�ect is not limited to Earth. Any planet that has a signi�cant atmosphere has somekind of greenhouse e�ect. Venus has a signi�cant greenhouse e�ect that keeps the surface of the planetextremely hot, averaging around 460�C. A probe that was sent to study the planet survived for only twohours before melting, even though it was designed with durable metals.

23http://www.curriki.org/xwiki/bin/view/Coll_Athabasca/Unit5-Lesson3TheGreenhouseE�ect

Available for free at Connexions <http://cnx.org/content/col11410/1.3>

185

Figure 3.23

With the re�ection of light o� the surface and the surface radiation of heat, much of the energy fromsunlight would be lost back to space. Fortunately the atmosphere acts like the transparent panels from thegreenhouse trapping the heat. Natural gases in our atmosphere called greenhouse gases (such as carbondioxide and water vapour) are extremely good at absorbing various kinds of sunlight. So, rather thanescaping back into space, much of this re�ected light and heat is actually absorbed by the greenhouse gases.This has a signi�cant warming e�ect on our atmosphere.

Many people associate the greenhouse e�ect with global warming. In fact, there is so much confusion,that these terms are sometimes used interchangeably. The greenhouse e�ect is naturally occurring on mostplanets, and it is necessary on Earth to maintain life as we know it.

Without the greenhouse e�ect, the temperature of the Earth might be as much as 30�C cooler! Thatwould alter the surface of the Earth signi�cantly, covering much of it with ice. We need the greenhouse e�ectto survive on Earth.

However, there can be too much of a good thing. Human beings have begun adding a large amountof greenhouse gases, primarily carbon dioxide, into our atmosphere. This came mostly with the industrialrevolution when we began to burn coal and gasoline, and now many other fossil fuels (such as propane,natural gases), and even wood, in great quantities. With this increase in carbon dioxide in our atmosphere,

Available for free at Connexions <http://cnx.org/content/col11410/1.3>

186 CHAPTER 3. ENVIRONMENTAL STUDIES

there is more gas to absorb energy. With more energy being absorbed, the temperature of the atmosphere isbeginning to increase, causing changes within our weather patterns, and other in�uences on the ecosystemsof the Earth. This is called climate change.

In the past few decades the population of the Earth has doubled to over six billion people. These six billionpeople foster a large increase in automobile transportation; the major source of the increase in greenhousegases. The greater population has also required more resources such as land. Large amounts of forest havebeen cut down. Trees are one of the most important organisms that actually remove carbon dioxide from theatmosphere during photosynthesis. So not only are humans adding more carbon dioxide to the atmosphere,but they are also destroying trees that would otherwise be helping to absorb excess carbon dioxide from theatmosphere.

We will not know the full impact of global warming until perhaps the middle of this century. This isbecause it takes so long for the full impact to be felt. You may remember that water vapour and carbondioxide are a very small part of the makeup of our original atmosphere (see Module 5, Tutorial 1). So aswe add carbon dioxide from burning fossil fuels there is only a very small change in the makeup of ouratmosphere. In fact it takes a long time for the atmosphere to mix in the added greenhouse gases fully.Scientists say that even if we halted the release of greenhouse gases today, the climate would continue towarm until about the year 2050 as the atmosphere reaches a new stable state.

3.1.3.3.1.3 The e�ect of humans on biodiversity

This section will summarize some of the impacts that human's have had on the natural environment, focussingon our impact on the biotic (living) environment.

SummaryDestruction of biodiversity - hunting, pollution, destruction of the natural environment and intro-

duction of alien species can result in the extinction of native species.Introduction of alien invasive species: These are plants and animals that are NOT indigenous to

a particular country. The plants and animals grow rapidly as they are alien and have no natural predatorsto feed on them.

Chemical control: use of insecticides to kill insects that damage food crops. This also harms theenvironment and gets washed into rivers, dams and seas, negatively impacting on food chains.

Biological control: use natural predators to eradicate (get rid of) of pests that feed and damage onfood crops. This ensures that the useful plants and animals as well as the soil as a natural resource is notdamaged.

Deforestation: removal of natural forests. This destroys biodiversity, and also removes plants whichhelp to decrease the carbon dioxide burden.

Acid Rain : Burning fossil fuels may also produce sulphur dioxide and nitrogen oxide gases. Thesegases rise up in the atmosphere from factories, power stations and vehicles and dissolve in the water in cloudsforming acid rain. Acid rain can damage trees directly.If the water in rivers and lakes becomes too acidic,plants and animals cannot survive. Acid rain also damages buildings

Eutrophication: Pollution of water by fertilisers may cause eutrophication. The fertiliser causesrapid growth of aquatic plants such as algae. The increased amount of algae on the surface of the waterblocks out sunlight, preventing plants at the bottom of river from photosynthesising. These plants die andbegin to decompose. Micro-organisms feed and decompose dead plants using up oxygen in the water. Fishand other aquatic organisms die due to oxygen shortage.

Sewage pollutes the sea and fresh water, unless it is treated properly.This is a health hazard, but italso upsets the balance of organisms living in the water. Untreated sewage provides food for micro-organismsand causes eutrophication.

Toxic chemicals are released from industrial plants and by farmers. These can pollute the land andwater.Some toxins e.g. DDT can accumulate along food chains until the top predator has very high, oftenlethal, amounts in its body.DDT is carcinogenic (cancer causing)

Available for free at Connexions <http://cnx.org/content/col11410/1.3>

187

3.1.3.3.1.3.1 De�nitions and terms

Pollution: an undesirable change in the natural resources due to negative in�uences of human activities.Thermal pollution: hot waste water released by industries and mining into rivers and lakes increases

temperature of water `driving' out oxygen killing �sh and other aquatic life.Contamination: not �t for human consumption as it contains bacteriaSewage: waste water containing faecesFlora: refers to all plant lifeFauna: refers to all animal life

3.1.3.3.1.4 Activities

3.1.3.3.1.4.1 The Greenhouse e�ect

To see how greenhouse gases a�ect the climate try this simulation from PhET. Explore the atmosphere duringthe ice age and today. What happens when you add clouds? Change the greenhouse gas concentration andsee how the temperature changes. Then compare to the e�ect of glass panes. Zoom in and see how lightinteracts with molecules. Do all atmospheric gases contribute to the greenhouse e�ect?

Phet: The Greenhouse E�ect: http://phet.colorado.edu/en/simulation/greenhouse 24

3.1.3.3.1.4.2 Human's in�uence on greenhouse gas concentrations

Take a look at http://www.breathingearth.net/ to see how much CO2 is currently been released into theatmosphere.

Watch for 4 minutes. How many people were born in that time? How many people died?If the current grade 9's repeated this exercise exactly one year from today, at exactly the same time of

day, by how much will the world's population have grown? Do you think this is a problem? Why?How much CO2 will have been added to the atmosphere by that time? How does South Africa compare to

the rest of the world? Do you think all South African's contribute equally to CO2 emissions in our country?

3.1.3.3.1.4.3 Discovering your impact

What Is A Carbon Footprint?A carbon footprint is a measure of the impact our activities have on the environment, and in particular

climate change. It relates to the amount of greenhouse gases produced in our day-to-day lives throughburning fossil fuels for electricity, heating and transportation etc.

The carbon footprint is a measurement of all greenhouse gases we individually produce and has units oftonnes (or kg) of carbon dioxide equivalent.

A carbon footprint is made up of the sum of two parts, the primary footprint and the secondary footprint.The primary footprint is a measure of our direct emissions of CO2 from the burning of fossil fuels

including domestic energy consumption and transportation (e.g. car and plane). We have direct control ofthese.

The secondary footprint is a measure of the indirect CO2 emissions from the whole lifecycle ofproducts we use - those associated with their manufacture and eventual breakdown. To put it very simply� the more we buy the more emissions will be caused on our behalf.

To work out what your carbon footprint is visit: http://www.carbonfootprint.com/calculator.aspx 25

To discover how to reduce your carbon footprint visit: http://www.carbonfootprint.com/minimisecfp.html26

24http://phet.colorado.edu/en/simulation/greenhouse25http://www.carbonfootprint.com/calculator.aspx26http://www.carbonfootprint.com/minimisecfp.html

Available for free at Connexions <http://cnx.org/content/col11410/1.3>

188 CHAPTER 3. ENVIRONMENTAL STUDIES

3.1.3.3.1.4.4 Climate Change

Student's guide to climate changehttp://www.epa.gov/climatechange/kids/index.html 27

http://climate.nasa.gov/ 28

http://climate.nasa.gov/imagesVideo/climateReel/index.cfm 29

3.1.3.3.1.5 Assignment

Identify the ABC's (abiotic, biotic and cultural characteristics) of a natural environment near you. To makeyour ABC pro�le, follow the instructions below.

1. Select an area that is undeveloped (i.e. no buildings, no pavement, no bulldozing, no spraying ofpesticides, no farming, no grazing, etc.). Your area must be at least the size of a soccer �eld. For some thiswill be an easy walk from their homes. Others will have to travel quite a distance[U+2011][U+2011]luckyyou! You can think of it as a �eld trip. Make a map of your province and show, approximately, where yourarea is located.

2. Identify the at least 10 �A� (abiotic) features of your area. Consider factors such as:

• Landforms (mesa, mountain, valley, bench, etc..• Altitude

3. Identify at least 15 �B� (biotic) features of the area. (You may use common names.) Consider things suchas:

• Plants (trees, shrubs, grasses, �owers, etc.)• Insects (ants, bees, praying mantis, etc.)• Amphibians, reptiles, and/or �sh

4. Identify at least 3 �C� (cultural) components. Look for evidence of human in�uence. Consider thingssuch as:

• *Recycling, conservation e�orts• *Pollution• *Introduced species

ANALYSISNB- Come back and use South African examples for the model answer examplesExamine the data you collected when making your ABC pro�le. Use your collected data to answer the

following questions.1. What e�ect does the environment (abiotic) have on the organisms (biotic) living there? Give FIVE

speci�c examples from your pro�le. [For example: Lily pads (biotic) are able to grow in my area becauseit is a natural wetland that has standing, stagnant water (abiotic) all year long.]

2. What e�ect do the organisms (biotic) have on the environment (abiotic)? Give THREE speci�cexamples from your pro�le. [For example: The area is heavily shaded by spruce trees (biotic). The shadekeeps the soil moist (abiotic) and reduces the air temperature.]

3. How do natural forces a�ect the area? Give ONE speci�c example from your pro�le. Consider thedirection of the prevailing winds, the direction from which the sun's rays come, gravity (if you are on aslope), etc. . .

4. How have humans a�ected your area? Give ONE speci�c example.5. Predict how your area would change if the amount of rainfall doubled. Be sure to mention how this

increase in rainfall would a�ect the abiotic and biotic factors.

27http://www.epa.gov/climatechange/kids/index.html28http://climate.nasa.gov/29http://climate.nasa.gov/imagesVideo/climateReel/index.cfm

Available for free at Connexions <http://cnx.org/content/col11410/1.3>

189

3.1.3.3.2 Questions

Question 1Read the following text and answer the questions that follow :South Africa has o�ered to slow the growth of its greenhouse gas emissions by 34% by 2020. The

biggest producers of greenhouse gases in South Africa are Eskom, the producers of electricity, and Sasol, theproducers of petrol and diesel from fossil fuels.

1.1.Which is the most important greenhouse gas?(1)1.2.Explain why Eskom and Sasol produce the most greenhouse gases. (2)1.3.You are a scientist who has been employed by the government to check that the target is reached.

Brie�y explain the methodology you would use to do this. (3)1.4.Suggest two strategies that Eskom and Sasol could use to reduce their greenhouse gas emissions. (2)Question 2Study the table below on ozone depletion and answer the questions that follow:Amount of ozone depletion of stratosphere between 1982 & 1996

Year Ozone depletion (in tons)

1982 2000

1984 3000

1986 4000

1988 3500

1990 6000

1992 7000

1994 8000

1996 7000

Table 3.5

2.1.Comment on the ozone depletion during the period 1982 to 1996.(2)2.2.How much of the ozone was lost from the stratosphere in 1990? (1)2.3.CFC's can stay around for about 100 years. What is the implication of this for the ozone layer? (1)2.4.Give ONE possible reason for the decrease in ozone depletion after 1994.(2)Question 3The dying of trees due to acid rain is brought about by changes in the pH of the soil. Tshepo observed

that trees in his town were gradually dying. He decided to investigate the e�ect of acid rain on germinatingbean seeds. He did the following:

• Placed cotton wool on 6 saucers• Poured a solution with a di�erent pH (varying from pH 2 to pH 7) onto the cotton wool in each saucer• Placed 50 bean seeds onto the cotton wool in each saucer• Covered the seeds and left them on the shelf for a week• The following results were obtained:

Number of germinating seeds

2 0

3 4

Available for free at Connexions <http://cnx.org/content/col11410/1.3>

190 CHAPTER 3. ENVIRONMENTAL STUDIES

4 10

5 15

6 28

7 35

Number of germinating seeds

2 0

3 4

4 10

5 15

6 28

7 35

Number of germinating seeds

2 0

3 4

4 10

5 15

6 28

7 35

pH Number of germinating seeds

2 0

3 4

4 10

5 15

6 28

7 35

Available for free at Connexions <http://cnx.org/content/col11410/1.3>

191

Table 3.6

3.1.Name the independent variable in this investigation.(1)3.2.Name the dependent variable in this investigation.(1)3.3.Name TWO ways in which the investigation could be improved to obtain accurate and valid results.(2)3.4.Describe the relationship between pH and the number of germinating seeds. (2)3.5.Suggest THREE strategies that the government can use to help reduce the formation of acid rain.

(3)QUESTION 4The graph below shows the e�ect of biocontrol on an invasive alien plant population.Graph showing the e�ect of biocontrol on an invasive alien plant population.

Figure 3.24

4.1.Explain what is meant by the term `biocontrol'.(1)4.2.Give two reasons that you can see from the graph to support the fact that biocontrol has been

successful in controlling the alien plant population. (2)4.3.Why is host speci�city so important when introducing a biocontrol agent?(2)Read the following text and answer the questions that follow:QUESTION 5:GLOBAL BURDEN OF DISEASE DUE TO INDOOR AIR POLLUTIONIn the year 2000, indoor air pollution from solid fuel use was responsible for more than 1,6 million annual

deaths. Smoke and gases that come from the burning of fossil fuels, such as coal, charcoal and wood, to heathomes and to cook contributes to indoor pollution.

Dependence on burning fossil fuels to meet basic energy needs is one of the underlying causes of asthma,bronchitis, pneumnia and other respiratory illnesses among children.

Adapted from: http://www.who.int/indoorair/health_impacts/burden_global/enprint.html 30

30http://www.who.int/indoorair/health_impacts/burden_global/enprint.html

Available for free at Connexions <http://cnx.org/content/col11410/1.3>

192 CHAPTER 3. ENVIRONMENTAL STUDIES

5.1 Name THREE illnesses that are thought to be made worse by exposure to indoor pollution.(3)5.2 Why do you think indoor pollution causes so many medical problems?(2)Suggest TWO ways in which communities can try to decrease the negative e�ects of indoor pollution.(2)Studies show that the average South African has a �carbon footprint� that is higher than the world

average because most of our electricity is generated from burning coal.5.3 State THREE ways to solve this problem.(3)Question 6(NSC Additional Exemplar 2008 Life Science Paper 2 Question 1)Study the table below showing the amount of medical waste produced by three provinces over a number

of yearsAmount of medical waste (tones)

19951997199920012003

Province A 357 398 410 426 450

Province B 283 290 300 312 330

Province C 230 240 245 270 290

Amount of medical waste (tones)

Available for free at Connexions <http://cnx.org/content/col11410/1.3>

193

19951997199920012003

Province A 357 398 410 426 450

Province B 283 290 300 312 330

Province C 230 240 245 270 290

Amount of medical waste (tones)

19951997199920012003

Province A 357 398 410 426 450

Available for free at Connexions <http://cnx.org/content/col11410/1.3>

194 CHAPTER 3. ENVIRONMENTAL STUDIES

Province B 283 290 300 312 330

Province C 230 240 245 270 290

Year Amount of medical waste (tones)

1995 1997 1999 2001 2003

Province A 357 398 410 426 450

Province B 283 290 300 312 330

Province C 230 240 245 270 290

Table 3.7

6.1Which province has shown the most rapid increase in the amount of medical waste produced?(1)6.2What was the percentage increase of medical waste produced by the province named in QUESTION

2.1 over the period 1995 to 2003? Show ALL workings. (3)6.3Give TWO negative e�ects of dumping medical waste.(2)Question 7The graph below shows the e�ect of di�erent amounts of sewage discharged into a river, on the amount

of dissolved oxygen in the water. Study the graph and answer the questions that follow.

Available for free at Connexions <http://cnx.org/content/col11410/1.3>

195

Figure 3.25

7.1.Why does the sewage discharge cause oxygen levels to decrease?(2)7.2.Why does the minimum amount of oxygen occur some distance downstream of the point of sewage

discharge?(2)7.3.Why does the level of dissolved oxygen in the river then increase again further downstream?(2)7.4.Give TWO possible reasons why sewage would be discharged into river systems in a developing urban

environment.(4)7.5.Name one positive and one negative e�ect that �quite heavy pollution� could have on a rural farming

community living at point X down the river.(2)Question 88.1Humans can in�uence the environment in ONE of the following positive ways.ABurning fossil fuelsBConservationCPollutionDDeforestation8.2Pollutant gases have an adverse e�ect on the environment and on our health. The amounts of these

gases have been steadily increasing over the years. The table below shows the source and amount of somepollutant gases produced by human activities.

SourceAmount produced per year (millions of tons)

Carbon dioxide Vehicle exhausts 350

Available for free at Connexions <http://cnx.org/content/col11410/1.3>

196 CHAPTER 3. ENVIRONMENTAL STUDIES

Sulphur dioxide Burning coal and oil, industry 200

Nitrogen oxide Vehicle exhaust 55

SourceAmount produced per year (millions of tons)

Carbon dioxide Vehicle exhausts 350

Sulphur dioxide Burning coal and oil, industry 200

Nitrogen oxide Vehicle exhaust 55

SourceAmount produced per year (millions of tons)

Carbon dioxide Vehicle exhausts 350

Sulphur dioxide Burning coal and oil, industry 200

Nitrogen oxide Vehicle exhaust 55

Pollutant gas Source Amount produced per year (millions of tons)

Carbon dioxide Vehicle exhausts 350

Sulphur dioxide Burning coal and oil, industry 200

Nitrogen oxide Vehicle exhaust 55

Table 3.8

8.3.Which ONE of the following statements is correct?A150 million tons of nitrogen oxide is produced.BCarbon monoxide is produced by industry.

Available for free at Connexions <http://cnx.org/content/col11410/1.3>

197

CSulphur dioxide is produced by vehicle exhaust.DCarbon monoxide is the gas produced in the largest amount.8.4.Possible measures for the reduction of air pollution would NOT includeAproviding subsidies for solar panelsBproviding subsidies for truck driversCrewards for cleaner production by industriesDimproving the public transport system8.5Adding harmful substances to the environment is termedAexploitationBforestationCpollutionDdetoxi�cationQuestion 9The map below shows a region of coastline close to where a giant oil tanker was wrecked at sea. The

shallow waters of the coastline provided a rich source of edible crabs. Oil does not kill the crabs but harmtheir �esh, making them inedible and they cannot be sold. Samples of crabs were collected at sites A toD. The number of crabs is indicated by the size of the circle. The extent of the shaded part at each siterepresents the proportion of crabs with diseased �esh after the disaster.

Figure 3.26

9.1Which sample site (A to D) had the highest number of crabs? (1)9.2.In which sample site was the crabs only rarely found?(1)9.3Name the agent of pollution that a�ected the crabs.(1)9.4In which sample site was the most crabs a�ected compared to the population size? (1)9.5Explain your answer to QUESTION 2.4(2)9.6.List TWO strategies that could reduce the e�ects of oil pollution at sea. (2)

Available for free at Connexions <http://cnx.org/content/col11410/1.3>

198 CHAPTER 3. ENVIRONMENTAL STUDIES

3.1.3.3.3 Discussion Points

3.1.3.3.3.1 The Tragedy of the Commons

From: http://cnx.org/content/m16743/latest/?collection=col10548/latest (AP Environmental Science:Environmental Ethics) from Connexions

In his essay, The Tragedy of the Commons, Garrett Hardin (1968) looked at what happens when humansdo not limit their actions by including the land as part of their ethic. The tragedy of the commons developsin the following way: Picture a pasture open to all. It is to be expected that each herdsman will try tokeep as many cattle as possible on the commons. Such an arrangement may work satisfactorily for centuries,because tribal wars, poaching and disease keep the numbers of both man and beast well below the carryingcapacity of the land. Finally, however, comes the day of reckoning (i.e., the day when the long- desiredgoal of social stability becomes a reality). At this point, the inherent logic of the commons remorselesslygenerates tragedy.

As a rational being, each herdsman seeks to maximize his gain. Explicitly or implicitly, more or lessconsciously, he asks: "What is the utility to me of adding one more animal to my herd?" This utility hasboth negative and positive components. The positive component is a function of the increment of one animal.Since the herdsman receives all the proceeds from the sale of the additional animal, the positive utility isnearly +1. The negative component is a function of the additional overgrazing created by one more animal.However, as the e�ects of overgrazing are shared by all of the herdsmen, the negative utility for any particulardecision-making herdsman is only a fraction of -1.

The sum of the utilities leads the rational herdsman to conclude that the only sensible course for him topursue is to add another animal to his herd, and then another, and so forth. However, this same conclusionis reached by each and every rational herdsman sharing the commons. Therein lies the tragedy: each manis locked into a system that compels him to increase his herd, without limit, in a world that is limited. Ruinis the destination toward which all men rush, each pursuing his own best interest in a society that believesin the freedom of the commons. Freedom in the commons brings ruin to all.

Hardin went on to apply the situation to modern commons. The public must deal with the overgrazingof public lands, the overuse of public forests and parks and the depletion of �sh populations in the ocean.Individuals and companies are restricted from using a river as a common dumping ground for sewage andfrom fouling the air with pollution. Hardin also strongly recommended restraining population growth.

The "Tragedy of the Commons" is applicable to the environmental problem of global warming. Theatmosphere is certainly a commons into which many countries are dumping excess carbon dioxide fromthe burning of fossil fuels. Although we know that the generation of greenhouse gases will have damaginge�ects upon the entire globe, we continue to burn fossil fuels. As a country, the immediate bene�t from thecontinued use of fossil fuels is seen as a positive component. All countries, however, will share the negativelong-term e�ects.

3.1.3.3.4 Additional Resources

Plants can tell us about climate changeSee how the general public are helping scientists monitor climate change by observing the timing of

lea�ng, �owering, and fruiting of plants (plant phenophases).Project Budburst http://neoninc.org/budburst/ 31

Ecology sitehttp://www.ecology.com 32

The Story of Stu�To see how humans can a�ect the environment: watch �The story of stu��:

http://youtu.be/9GorqroigqM

31http://neoninc.org/budburst/32http://www.ecology.com/

Available for free at Connexions <http://cnx.org/content/col11410/1.3>

199

3.1.3.3.5 Answers

Question 11.1B1.2D1.3B1.4.CQuestion 22.1.Sample site A2.2.Sample site D2.3.Oil2.4.Sample B2.5.Sample B has greater shaded portion2.6.Regular service of oil tankersLegislation on the amount of oil the oil tankers transportMonitoring of travel routesUse bacteria to dissolve the oil spillEXCELLENT RESOURCES, QUESTIONS, ACTIVITIES IN MINDSET LEARN MA-

TERIAL!!!!!

3.1.4 3.1.4 Ecosystems33

3.1.4.1 Key concepts

In this session we will focus on summarising what you need to know about:

• Concept of an ecosystem• Structure and ecosystem function related to abiotic factors and biotic factors• Factors that in�uence terrestrial and aquatic primary production

3.1.4.2 Terminology & De�nitions

Ecosystem - a community plus the physical environment that it occupies at a given timeAbiotic factors � non-living factors . Eg soil, air.Biotic factors � living factors. Eg trees, insects.Producer � organism that manufactures food by photosynthesis.Herbivore � an organism that feeds on plants only.Carnivore � an organism that feeds on animals only.Omnivore � an organism that feeds on both plants and animals.Saprotroph � an organism that feeds on dead and decaying matter. (fungi)Decomposers � an organism that causes the decay of dead and dying organisms. (bacteria)Scavenger � an organism that feeds on remains of animals.Predator � an animal that hunts for its food.Prey � an animal that is hunted and is food for the predator.Food chain � a chain showing feeding relationships between organisms.

33This content is available online at <http://cnx.org/content/m43207/1.1/>.

Available for free at Connexions <http://cnx.org/content/col11410/1.3>

200 CHAPTER 3. ENVIRONMENTAL STUDIES

3.1.4.3 Content

An ecosystem is a community of living organisms interacting with each other and their environment. Ecosys-tems occur in all sizes. A tidal pool, a pond, a river, and a yellowwood forest are all examples of ecosystems.The living organisms (plants and animals) are referred to as biotic factors.The non-living components makeup the abiotic factors. Organisms living in a particular ecosystem are adapted to the prevailing abiotic andbiotic conditions.

3.1.4.4 Abiotic Components

Abiotic factors may be grouped together as:

• Physical / Climatic factors: sunlight, water, temperature and gases.• Edaphic factors: soil type, soil pH.• Physiographic factors: slope, aspect and altitude and latitude.

3.1.4.4.1 Climatic Factors:

3.1.4.4.1.1 Sunlight

• Light is a fundamental necessity.• The amount of sunlight an ecosystem receives is called insolation. This is critical for plant survival,

especially in aquatic ecosystems. Light decreases and water depth increases.• Amount of light changes from season to season.• In summer � days are longer, ecosystems gets more hours of sunlight than during short winter days.• The amount of sunlight a�ects other features of the ecosystem such as temperature, and the types of

plants and animals that are found there.

3.1.4.4.1.2 Temperature

• Temperature increases with an increase in both latitude and altitude.• Temperature a�ects the rate at which photosynthesis, respiration and decomposition take place. The

higher the temperature, the faster the processes.

3.1.4.4.1.3 Water

• Water is one of the most important factors in the ecosystem.• It is the main component of living cells and is essential for all living organisms.• About 80% of the human body and 90% of the plant body consists of water.• Water is evenly distributed over the earth.• It is abundant in aquatic ecosystems and least in deserts• Plants are adapted to the available amount of water in the following ways:

· Xerophytes are plants that are able to live in dry habitats, or in regions with low annual rainfall.These plants are resistant to drought, have to cope with shortage of water, high temperatures andlight intensities and dry warm winds. ie Cactus, Aloe

· Hydrophytes are plants that are able to live entirely or partially in submerged water ot in verywet soil. These plants have to cope with a water surplus. ie Water lily, Water hyacinth

· Mesophytes are plants that need an average, regular supply of water. ie Fruit trees, wheat

Available for free at Connexions <http://cnx.org/content/col11410/1.3>

201

3.1.4.4.1.4 Air / Gases

• The movement of air is called wind.• Wind a�ects many things in the ecosystem ie speeds up evaporation, assist in pollination of plants and

the dispersal of their seeds.• Air is composed of 78% Nitrogen, 21% Oxygen, 4% Carbon dioxide and water vapour.

· Oxygen � used in cellular respiration and combustion and is returned to the environment by theprocess of photosynthesis.

· Carbon dioxide � is a product of cellular respiration and decayed organic matter. It is removedfrom the atmosphere by plants during the process of photosynthesis.

· Nitrogen � is needed by all living organisms for the synthesis of proteins.· Water vapour - the amount of water vapour found in the air remains constant on average, however,it can vary greatly from one place to another. Some parts of the Earth are prone to high humiditylevels, why other locations have very dry air. Much of what we consider weather is caused bywater vapour. The clouds in the sky are largely made up of it, and it is the condensation of thisvapour into droplets that creates rain and snow.

3.1.4.4.2 Edaphic Factors:

3.1.4.4.2.1 Soil Structure

• Humus content is the decomposed remains of dead organic matter. It gives• the topsoil its dark colour. It supplies plants with nutrients and helps the soil• absorb (retain) water. Soils rich in humus are fertile soils.• Texture of soil is determined by the size of the soil particles.

3.1.4.4.2.2 Soil Properties

• How acid or alkaline a soil is, can be measured by the pH scale .• pH is a measure of the acidity or alkalinity of a solution.• The pH scale ranges from 0 to 14. Neutral solutions have a pH value of 7 . Acid

solutions have a pH value of less than 7 and alkaline solutions greater than 7 .• Litmus paper or universal indicator can be used to determine whether a solution is acid or

alkaline.

3.1.4.4.3 Physiographic Factors:

3.1.4.4.3.1 Aspect

• Refers to the position of an area in relation to the sun or wind or wave action.• It is the direction that the slope faces ie North, South, East, West .• In Southern Hemisphere -

· North facing habitats receive:

* more solar radiation than a South facing one.* are warmer and drier* greater variety of animals

· South facing habitats are:

* cooler* have a higher water capacity* plant growth is luxurious

Available for free at Connexions <http://cnx.org/content/col11410/1.3>

202 CHAPTER 3. ENVIRONMENTAL STUDIES

3.1.4.4.3.2 Slope

• Also referred to as the gradient which represents the steepness of a slope.• A�ects the rate of water run-o�

· Ie Steep slope � encourages fast run-o� of water and causes soil erosion. Soils tend to be shallowand infertile and plant growth is reduced. Plants are small and few animals present

• Gentle slope � favours slower �ow of surface water, reduces erosion, and increases availability ofwater to plants.

3.1.4.4.3.3 Altitude

• Is the height of the land above sea level.• At high altitudes :

· the temperature is lower ,· the wind speed is greater,· and the rainfall less and more likely to get snow.· Less plant and animal species found· Plants more stunted than at lower elevations.

Available for free at Connexions <http://cnx.org/content/col11410/1.3>

203

3.1.4.4.4 BIOTIC COMPONENTS

Figure 3.27

Living organisms in an ecosystem are usually grouped according to how they obtain food.

• Autotrophs that make their own food are known as producers, while• Heterotrophs that eat other organisms, living or dead, are known as consumers.• The producers include land and aquatic plants, algae and microscopic phytoplankton in the ocean.

Available for free at Connexions <http://cnx.org/content/col11410/1.3>

204 CHAPTER 3. ENVIRONMENTAL STUDIES

• They all make their own food by using chemicals and energy sources from their environment.

For example, plants use photosynthesis to manufacture sugar (glucose) from carbon dioxide and water.Using this sugar and other nutrients (e.g., nitrogen, phosphorus) assimilated by their roots, plants producea variety of organic materials. These materials include: starches, lipids, proteins and nucleic acids. Energyfrom sunlight is thus �xed as food used by themselves and by consumers.

The consumers are classed into di�erent groups depending on the source of their food. Herbivores (e.g.deer, squirrels) feed on plants and are known as primary consumers. Carnivores (e.g. lions, hawks, killerwhales) feed on other consumers and can be classi�ed as secondary consumers. They feed on primaryconsumers. Tertiary consumers feed on other carnivores. Some organisms known as omnivores (e.g., bears,rats and humans) feed on both plants and animals. Organisms that feed on dead organisms are calledscavengers (e.g., vultures, ants and �ies). Detritivores (detritus feeders, e.g. earthworms, termites, crabs)feed on organic wastes or fragments of dead organisms.

Decomposers (e.g. bacteria, fungi) also feed on organic waste and dead organisms, but they digest thematerials outside their bodies. The decomposers play a crucial role in recycling nutrients, as they reducecomplex organic matter into inorganic nutrients that can be used by producers. If an organic substance canbe broken down by decomposers, it is called biodegradable.

In every ecosystem, each consumer level depends upon lower-level organisms (e.g. a primary consumerdepends upon a producer, a secondary consumer depends upon a primary consumer and a tertiary consumerdepends upon a secondary consumer).

3.1.4.4.4.1 Activity:

Let's study an example of an ecosystem:(a) What is a forest?A forest is much more than a large area of land covered with trees. Shrubs, vines, ferns, mosses and

toadstools live in the shade of trees. The forest also swarms with birds, mammals, reptiles, amphibians andinsects. A forest is therefore all these plants and animals living together.

The animals depend on plants for food, while plants use sunlight, carbon dioxide, water and minerals tomake food for themselves and other organisms in the forest.

The living organisms (plants and animals) together with the non-living environment (air, water, sun andsoil) constitute an ecosystem.

(b) Tabulate an example of each of the following found in a forest.

Species My example of...

Bird

Mammal

Reptile

Insect

Amphibian

Table 3.9

(c) Now sketch these �ve examples in the drawing to complete it!(d) The Ecosystem conceptThe organism-environment interaction leads to the ecosystem concept, elaborating the interaction be-

tween matter, energy and organism.(e) How big is an ecosystem?An ecosystem can be of any size, from a puddle of water on the pavement to the entire rain forest in the

Amazon or an even larger area.

Available for free at Connexions <http://cnx.org/content/col11410/1.3>

205

A forest with its trees, plants, insects, birds, etc., is an ecosystem of certain kinds of organisms thatoccupy a certain environment. On the other hand, a rock in the shade of a forest with its mosses and otherrock plants, insect larvae and centipedes, is also an ecosystem. The system is therefore integrated, with partsthat are intimately related to one another. Anything a�ecting a part of the system will also a�ect the rest.

Assignment 1:(f) Study the sketch of the forest ecosystem below:Name the:

• producer• primary consumer• secondary consumer• tertiary consumer

Primary means �rst.Secondary means second.Tertiary means the third in this case.2. The ecosystem is the living organisms together with the

____________________________________invisibly linked and mutually interactive.The living part can be divided into:(a) food producers - mainly green plants(b) food consumers - which are macro consumers (animals which eat other organisms) or decomposers

(bacteria or fungi which break up dead organisms)

3.1.4.4.4.2 Activity:

To discuss rock pool ecosystemsThis ecosystem is extremely sensitive. The main factor that plays a role here is TIDES.The tide changes twice every 24 hours. As a result of the wave action, cold, oxygen-rich sea water washes

over the rock pool communities. During low tide the water in the rock pools heats up and evaporation takesplace. These changes contribute to the adaptation of the organisms that live in the rock pools so that theymay survive the various challenges of nature.

1. Describe what you understand by �a rock pool�.2. List all the abiotic factors that have an impact on rock pools.Plants in rock poolsThe most common plants are sea-weeds or sea-algae. They are red, green or brown.All contain chlorophyll but in some the green is masked by red / brown pigments. Although they are not

always green, they can also produce nutrients through photosynthesis. Therefore they are also producers.They also provide nutrition for a wide range of other organisms that live o� them, such as mussels and

some sea-snails.1. Do research in order to describe what each of the following is:(a) �lter feeders:(b) scavengers:2. What would the purpose of tentacles be in sea-anemones?3. What kind of mutual dependence exists between the organisms in a rock pool?Assessment of the interpretation of the SKETCHESCould you distinguish the basic rock pool components?

3.1.4.4.4.3 Environmental Investigation

Brief � you are going to investigate the number of grass plants growing on the lawns of the north side andthe south side of South High.

Investigate � design your investigation.

Available for free at Connexions <http://cnx.org/content/col11410/1.3>

206 CHAPTER 3. ENVIRONMENTAL STUDIES

Your method must be fair, reliable, precise, valid and accurate.You should record your results in an appropriate table.Write up � write up your investigation.Use the following headings �

• Hypothesis• Method• Justi�cation• Results• Analysis• Evaluation

Hypothesis � Write a Hypothesis for your investigation. It is a statement of your expected results (2)Method � Write a bullet pointed Method for your investigation procedure. Remember to include the

steps taken to make it Fair, Reliable, Precise and Valid. (8)Justi�cation � Explain what you did to make your experiment Fair, Reliable, Precise and Valid. (4)Results � Display your results in a table and in a graph if appropriate. (8)Analysis � Use maths to analyse your results and state clearly what the results show. (4)Evaluation - Evaluate your experiment. What was good about it and why? What could you improve

and how? (6)Practical Technique (8)Total 40 marksMemo: Environmental InvestigationHypothesis � Write a Hypothesis for your investigation. What are you trying to prove? (2)

• There is a di�erence/ no di�erence One side has more grass plants/greater density of grass plants thanthe other (2)

Method � Write a bullet pointed Method for your investigation procedure. Remember to include the stepstaken to make it Fair, Reliable, Precise and Valid. (8)

• Use random sampling/Construct square/Count stems/Record/Repeat/Find average/Repeat for otherside/in exactly the same way (Max 7)

• Bullet pointed or numbered (1)

Justi�cation � Explain what you did to make your experiment Fair, Reliable, Precise and Valid. (4)

• Fair linked to doing the same to both sides• Reliable linked to repeats• Precise linked to using ruler/precise counting• Valid linked t no bias/random

Results � Display your results in a table and in a graph if appropriate. (8)

• Caption (2)• Table headings( 2)• Correct data (2)• Total and average (2)

Analysis � Use maths to analyse your results and state clearly what the results show. (4)

• Correct calculation of total shown (2)• Correct calculation of average shown (1) AND LINKED TO HYPOTHESIS (1)

Available for free at Connexions <http://cnx.org/content/col11410/1.3>

207

Evaluation - Evaluate your experiment. What was good about it and why? What could you improve andhow? (6)

• 3 Good points mentioned (3) eg Reliable, Precise, Fair, Unbiased• 3 Improvements mentioned (3) More samples, Bigger square, Extend to other areas

Practical Technique (8)

• Candidate fully involved in all aspects of investigation (2)• Candidate often involved (1)• Candidate lets others do the work (0)• Candidate counted patiently and accurately (2)• Candidate did some counting (1)• Candidate did no counting (0)• Candidate fully involved in planning (2)• Candidate involved t some extent in planning (1)• Candidate did no planning (0)• Candidate ensured results were recorded properly by group (2)• Candidate involved t some extent in recording (1)• Candidate did no recording (0)•

Total 40 marks

3.1.4.4.4.4 Investigative Project

Background information on the state of our environmentPopulation growth � The increase in population no's together with the world unsustainable consump-

tion pattern is putting ever-increasing stress on the land, water, air, energy and other essential resources ofthe planet. Rapid population growth is usually accompanied by serious environmental degradation, includingsoil erosion, deserti�cation and deforestation.

Biological diversity � is essential for human survival. Biological resources provide food, medicines,clothing, housing and a wide range of raw materials. They are important for the future improvement oflivestock and crops, and for the development of new medicines and products. Plants and animals are alsoessential in the maintenance of soil productivity, the degradation of waste, and in pest and �ood control.

Fresh water � is essential for the maintenance of life on earth. It is vital for drinking, sanitation, indus-try, food production, urban developments, power generation, transportation inland �sheries and recreation.Adequate supplies of fresh water are available to satisfy projected worldwide demands into the twenty-�rstcentury if the resource is e�ciently used.

Energy stands as one of the most essential of all the earth's resources. Human life would not existwithout the heat, light and food that depend on it. The world consumption of energy has quadrupled in thepast �ve decades. There has been a very great increase in the use of non-renewable energy sources, notablyfossil fuels, yet most energy is ine�ciently used.

Your presentations must focus on any of the following themes for creating a sustainablecity

• Conserving : including terrestrial, freshwater and marine ecosystems; restoration of; monitoringand conserving plants, animals and ecosystems.

• Water conservation : monitoring and reducing water use; water harvesting & re-use• Waste management : waste / litter audits; reducing, reusing and recycling; composting; avoid-

ing/appropriately disposing hazardous wastes• Energy conservation : monitoring and reducing energy use; alternative energy; global climate

change

Available for free at Connexions <http://cnx.org/content/col11410/1.3>

208 CHAPTER 3. ENVIRONMENTAL STUDIES

• Environmental health : nutrition and food gardens; controlling air and water pollution; hygieneand sanitation; creating a healthy environment

• Transport : reducing the use of fossil fuels and pollution through public / shared transport, walkingand cycling programmes,

Procedural Guideline :

• Decide on a topic. Choose a topic in which you have a particular interest and about which you wouldlike to know more. It is quite probable that your �eld of interest has nothing to do with your schoolsubject syllabus. The important thing is the investigative process that you follow. Get ideas frommagazines/ TV/ attached list.

• Find a question that needs an answer. Read available literature as background information. Startplanning your investigations/experiments that will hopefully answer your question.

• Now consult with your teacher and possibly some expert in the �eld. Phone a relevant Com-pany/University/Technicon Department. Ask for advice. Be polite when asking!!

• Your typed report:

1. This should be a detailed account of your research and investigation .2. It should be neat , attractive and illustrated if possible. Concise. Use photocopies, scanned

pictures or preferably use your own illustrations.3. Computer printed reports, using graphics are recommended.4. The format for the report should be as follows:

TITLE : Name of your projectAIM : What problem are you trying to solve?METHOD : Show programmes followed to solve the problem. Include experiments, clearly illustrated.

This could be done in a diary format.RESULT : Preferably tabulate results of experiments and observations. Include graphs of results, if

appropriate.CONCLUSIONS : Very importantTHEORY : Any theory, which needs explanation, can be included at any stage.APPLICATION : Can these conclusions have any practical application in everyday life?FURTHER RESEARCH: Very often your research opens up further questions related to your topic

which someone else might like to investigate.ACKNOWLEDGEMENTS AND REFERENCES :Include titles of books used, publishers and pages. Include names of people who advised and helped you.

Be honest in this regard.ASSESSMENT:PRESENTATIONNeatness10Visual impact10CONTENTFactual accuracy10Adequate coverage10EXPERIMENTAL WORK/RESEARCHTechnique and controls10Display of results10Interpretation of results10VERBAL COMMUNICATIONFactual accuracy10Insight and understanding10EVIDENCE OF PERSONAL GROWTH

Available for free at Connexions <http://cnx.org/content/col11410/1.3>

209

Enjoyment and learning experience5PLANNING5TOTAL100DATES AND DEADLINESPLANNING - Decide on topic. You must hand in a written report on what you are doing and how you

are planning to set about it. You will be penalised if your report is late. (This may be written)DUE:PROGRESS - You need to write about how far you've got with yourresearch, any complications/problems encountered, how youpropose to move on from there. . .DUE:FINAL PROJECT DUE:

3.1.4.4.4.5 Rich media

INTERACTIVE QUIZ http://www.oercommons.org/courses/ecology-quiz 34

This link is useful for consolidating the concepts of ecology.http://www.oercommons.org/courses/ecology-and-the-ecosystem-quiz 35

This link is useful for consolidating the concepts of ecosystems.http://www.oercommons.org/courses/ecological-footprint-calculator 36 In this exercise, you will utilize

an online calculator to examine your ecological footprint, compare it to the average footprint in your countryand other countries, and critically examine ways to reduce it.

VIDEO: http://www.curriki.org/xwiki/bin/view/Coll_NROCscience/APBiologyIIChapter33PopulationsandEcosystems 37

Video on the study of the interaction between organisms and their environ-ment. http://www.curriki.org/xwiki/bin/view/Coll_NROCscience/APBiologyIIChapter33Lesson64CommunityEcology?bc=;Coll_NROCscience.APBiologyIIChapter33PopulationsandEcos ystems 38

Video that examines the types of interactions that exist among organisms in a community, fromcompetition between consumers for speci�c resources to the relationship between predators and prey.http://www.curriki.org/xwiki/bin/view/Coll_NROCscience/APBiologyIIChapter34Ecosy stems 39 In thisInteractive Exercise, you will use a marine ecosystem to investigate the dynamic nature of ecosystems. Youwill explore the various regions of the bay, and then investigate some of the natural and human-imposedchanges to which the bay responds.

3.1.5 3.1.5 - Energy �ow40

3.1.5.1 Energy Flow and Nutrient Cycles

Organisms such as plants and animals need energy for growth, movement and reproduction. They get this inthe form of nutrients from the food they eat. The main source of energy for life on earth is the sun. The sunprovides energy to producers that use photosynthesis to grow and become food for consumers. Consumersinclude herbivores, carnivores and omnivores. Decomposers break down discarded plant and animal (organic)materials into simpler substances, which returns nutrients to the soil and atmosphere for new plants to useto grow.

34http://www.oercommons.org/courses/ecology-quiz35http://www.oercommons.org/courses/ecology-and-the-ecosystem-quiz36http://www.oercommons.org/courses/ecological-footprint-calculator37http://www.curriki.org/xwiki/bin/view/Coll_NROCscience/APBiologyIIChapter33PopulationsandEcosystems38http://www.curriki.org/xwiki/bin/view/Coll_NROCscience/APBiologyIIChapter33Lesson64CommunityEcology?bc=;Coll_NROCscience.APBiologyIIChapter33PopulationsandEcosystems39http://www.curriki.org/xwiki/bin/view/Coll_NROCscience/APBiologyIIChapter34Ecosystems40This content is available online at <http://cnx.org/content/m43176/1.1/>.

Available for free at Connexions <http://cnx.org/content/col11410/1.3>

210 CHAPTER 3. ENVIRONMENTAL STUDIES

3.1.5.1.1 Food chain

A food chain is a series of nutrients and energy moving through a chain of organisms. Below is an example ofa simple food chain in a grassland. The arrows show the movement of energy from one organism to another.

Figure 3.28

IMAGES sourced from: http://www.�ickr.com/photos/blueridgekitties/4625665988/sizes/o/in/photostream/

http://www.�ickr.com/photos/zest-pk/924783392/sizes/m/in/photostream/http://www.�ickr.com/photos/e3000/5922771249/sizes/l/in/photostream/

3.1.5.1.1.1 Activity 1:

Can you trace a food chain of the vegetables, fruit, cheese, eggs or meat that you had for breakfast or willhave for dinner?

3.1.5.1.1.2 Activity 2:

1. In the food chain shown above which of the three organisms is the

a. Herbivoreb. Carnivorec. Producer

Available for free at Connexions <http://cnx.org/content/col11410/1.3>

211

2. Draw a food chain showing at least 4 organisms.3. Producers use sunlight to manufacture their own food. Write a word equation as well as a balanced

equation to depict this process.4. Draw in the decomposers in the above food chain. Ensure that the direction of the arrows is correct.5. What animal will feed on the leopard?

3.1.5.1.2 Food web

A food web is made up of a number of food chains. It represents the di�erent feeding relationships in anecosystem or a biome. It is usually more complicated than a food chain because organisms can get theirenergy or food from more than one source. The presence of a number of food sources makes the system morestable. If one organism is removed, the whole system will not collapse, unlike in a single food chain.

Figure 3.29

(image source: http://per8eocreview.wikispaces.com/ )

3.1.5.1.2.1 Activity 3: Human Food Web

1. Divide into teams of at least �ve students each and stand in a circle.2. One person in each group takes a ball of wool or string. This person is the sun and starts the food

web.3. The �rst person (sun) holds onto the end of the wool and throws the ball of string to another person

in the group.4. The person who catches the ball has to name themselves something that uses energy from the sun

(primary producer).

Available for free at Connexions <http://cnx.org/content/col11410/1.3>

212 CHAPTER 3. ENVIRONMENTAL STUDIES

5. The person holding the ball (primary producer) has to hold the string with one hand and toss the ballto another student in the circle with his/her other hand.

6. The person who catches the ball has to name something that eats or is eaten by the previous organismnamed.

7. Carry on until everyone in the circle is holding the ball. You can throw the ball to someone who hasalready named themselves, as long as they eat you or are eaten by you

8. Look at the web you have created and the ones the other groups have created.9. Are some webs more complex than others? Why?

(source: http://www.oercommons.org/courses/got-energy-spinning-a-food-web/view)

3.1.5.1.3 Trophic levels and the Food Pyramid:

The trophic level of an organism is the position it holds in a food chain and depends on how much energyit consumes or produces. The trophic level of each organism can be drawn as a pyramid starting with theproducers at the bottom and moving up through the food chain.

The organism at the bottom gives the most energy and needs the least and the organism at the top needsthe most energy and releases the least. Energy is lost from activities at every level - through heat, egestion,urination and reproduction (pregnancy and egg-laying). This is why there is less and less energy as youmove up the pyramid.

Producers, eg. Plants are on the �rst level, or bottom of the pyramid, because they produce their ownnutrients using energy from the sun and therefore have a lot of energy to pass on.

Primary Consumers, eg. Herbivores are on the second level because they feed o� plantsSecondary Consumers, eg. Carnivores feed on herbivores so they get their energy from plants indirectly

and are on the third level.Tertiary Consumers, eg. Carnivores feed on organisms below them in the pyramid(image source: http://per8eocreview.wikispaces.com/)Figure 3. Food pyramidTrophic levels can be drawn as a pyramid of numbers, where each level shows the number of organisms

(look at �gure 3). They can also contain the biomass of a population. The biomass is the mass of livingorganisms in an ecosystem.

3.1.5.1.3.1 Activity 4:

Look at the food web and the diagram showing the di�erent trophic levels.

1. Identify a food chain that has three trophic levels.2. Identify a food chain that has four trophic levels.3. Name 2:

a. Producersb. Primary consumersc. Secondary consumersd. Tertiary consumers

4. There are very few tertiary consumers compared to the primary consumers. Why?5. What will happen if the impala is removed from the food web?

(source: http://www.learner.org/courses/envsci/interactives/ecology/ )VIDEO: http://www.youtube.com/watch?v=TE6wqG4nb3MThis a catchy song about food chains to help you remember.VIDEO: http://www.youtube.com/watch?v=-YwW-iWxLr4&NR=1Bill Nye the Science Guy talks about the Food Web

Available for free at Connexions <http://cnx.org/content/col11410/1.3>

213

3.1.6 3.1.6 Nutrient Cycles41

3.1.6.1 Nutrient cycles

Organisms rely on nutrients in order to survive. These include carbon, oxygen, nitrogen, water andmineral salts.

These nutrients need to be cycled through the ecosystem so that they can be re- used. This is callednutrient recycling . In biology, this is the movement of nutrients from the physical environment into livingorganisms and back into the environment. The �ow of energy you saw before from the sun to herbivoresand then to carnivores is part of this process. In any environment the �ow of nutrients must be stable andbalanced so that organisms can survive. If the cycle stops at any point, nutrients will become locked in placeand cannot be used in the next step.

The water cycle, carbon cycle, oxygen cycle and nitrogen cycle are examples of nutrientre-cycling.

Video: http://www.youtube.com/watch?v=AXWDbAYb-5c 42

Here is a simple video explaining nutrient cycling

3.1.6.1.1 Water

(ex http://cnx.org/content/m16470/latest/?collection=col10548/latest )The earth is sometimes known as the "water planet" because over 70 percent of its surface is covered by

water. All living organisms need water for their survival.In this cycle, water is transported between water reservoirs in the environment and living organisms.

This happens through these processes:

• Liquid water in oceans and lakes is converted to water vapour by evaporation . This is caused byheating of the water by the sun. The water vapour rises into the atmosphere.

• Plants release water into the air as vapour during photosynthesis, this is called transpiration .• When water vapour in the atmosphere cools, it can transform into tiny droplets of liquid water to form

clouds by the process of condensation• When condensed water droplets grow so large that the air can no longer support them against the pull

of gravity, they fall to the earth through precipitation . If the water droplets fall as liquid, it iscalled rain. If the temperature of the surrounding air is cold enough to freeze the water droplets, thenthe water falls as snow, sleet or hail.

• Water that falls onto the earth runs into lakes, rivers or oceans.

41This content is available online at <http://cnx.org/content/m43195/1.1/>.42http://www.youtube.com/watch?v=AXWDbAYb-5c

Available for free at Connexions <http://cnx.org/content/col11410/1.3>

214 CHAPTER 3. ENVIRONMENTAL STUDIES

Figure 3.30: (image source: http://per8eocreview.wikispaces.com/ 43 )The Water Cycle.

Animation: http://www.epa.gov/ogwdw/kids/�ash/�ash_watercycle.html 44

This is an animation of the water cycle

3.1.6.1.2 Oxygen

Oxygen is one of the main gases found in the air, along with nitrogen.Oxygen is re-cycled between the air and living organisms in the following ways:

• Organisms take in oxygen during respiration , which they use for cellular processes to break downenergy rich nutrients.

• When wood or fossil fuels burn, they consume oxygen and release carbon dioxide and water into theatmosphere through combustion .

• Plants release oxygen into the air as a by-product of photosynthesis .

43http://per8eocreview.wikispaces.com/44http://www.epa.gov/ogwdw/kids/�ash/�ash_watercycle.html

Available for free at Connexions <http://cnx.org/content/col11410/1.3>

215

Because animals trap oxygen during respiration, the release of oxygen by plants during photosynthesis is themain way oxygen is released into the atmosphere.

Figure 3.31: (image source: http://gellertclass.wikispaces.com/Chapter+3 45 )The Oxygen Cycle.

Video: http://www.youtube.com/watch?v=mUVX5rg1E0I 46

This is a video explaining the oxygen cycle

3.1.6.1.3 Carbon

Carbon is the basic building block of all organic materials, and therefore, of living organisms. Most of thecarbon on earth can be found in the crust. Other reservoirs of carbon include the oceans and atmosphere.

Carbon moves from one reservoir to another by these processes:

• Combustion of wood and fossil fuels transfers carbon to the atmosphere as carbon dioxide.

45http://gellertclass.wikispaces.com/Chapter+346http://www.youtube.com/watch?v=mUVX5rg1E0I

Available for free at Connexions <http://cnx.org/content/col11410/1.3>

216 CHAPTER 3. ENVIRONMENTAL STUDIES

• Carbon dioxide is taken up by plants during photosynthesis and gets converted into energy richsources, such as glucose, that contain carbon.

• Animals eat plants for food, taking up the carbon. They release carbon into the atmosphere as carbondioxide during respiration .

• Organisms convert carbon into organic molecules like fats, carbohydrates and proteins when they eatplants or animals.

• Carbon dioxide in the atmosphere can also precipitate as carbonate in ocean sediments. Theseocean sediments are melted by the movement of tectonic plates and then returned to the surfaceduring volcanic activity.

• Carbon dioxide gas is released into the atmosphere during volcanic eruptions.

Photosynthesis and respiration are the main carbon cycling processes involving living organisms.

Available for free at Connexions <http://cnx.org/content/col11410/1.3>

217

Figure 3.32: (image source: http://per8eocreview.wikispaces.com/ 47 )The Carbon Cycle

GAME: http://www.windows2universe.org/earth/climate/carbon_cycle.html 48

This is a game you can play to learn more about the carbon cycle

3.1.6.1.4 Nitrogen

Nitrogen (N2) makes up most of the gas in the atmosphere (about 78%). Nitrogen is important to livingorganisms and is used in the production of amino acids, proteins and nucleic acids (DNA, RNA). Only afew single-cell organisms, like bacteria can use nitrogen from the atmosphere directly. For multi-cellular

47http://per8eocreview.wikispaces.com/48http://www.windows2universe.org/earth/climate/carbon_cycle.html

Available for free at Connexions <http://cnx.org/content/col11410/1.3>

218 CHAPTER 3. ENVIRONMENTAL STUDIES

organisms, like plants and animals, nitrogen has to be changed into other forms, eg. nitrates or ammonia.This process is known as nitrogen �xation .

The nitrogen cycle involves these steps:During decomposition , bacteria and fungi break down proteins and amino acids from plants and

animals into nitrogen in the form of ammonia (NH3) by the process of ammoni�cation and convert theammonia to nitrate (NO3-) by nitri�cation .

Nitrogen can be changed to nitrates directly by lightning . The rapid growth of fungi and algae afterthunderstorms is because of this process, which increases the amount of nitrates that fall onto the earth inrain water, acting as fertilizer.

Ammonia and nitrates are absorbed by plants through their roots.Humans and animals get their nitrogen supplies by eating plants or plant-eating animals.The nitrogen is returned to the cycle when bacteria decompose the waste or dead bodies of these

higher organisms, and in the process, convert organic nitrogen into ammonia.In a process called denitri�cation , other bacteria convert ammonia and nitrate into nitrogen and

nitrous oxide (N 2 O). Nitrogen is returned to the atmosphere to start the cycle over again.

Figure 3.33: The Nitrogen Cycle

SIMULATION: http://www.teachersdomain.org/asset/lsps07_int_nitrogen/You can play with this simulation to learn more about the Nitrogen cycle.Animation:

Available for free at Connexions <http://cnx.org/content/col11410/1.3>

219

Here are some animations of the nitrogen cycle:http://www.mhhe.com/biosci/genbio/tlw3/eBridge/Chp29/animations/ch29/1_nitrogen_ cycle.swf 49

https://www.classzone.com/books/ml_science_share/vis_sim/em05_pg20_nitrogen/em05_pg20_nitrogen.html 50

3.1.7 3.1.7 Ecotourism51

Ecotourism Tourism in natural environments to observe wildlife, often that are under protection or containendangered species. It also refers to the practise of travelling to areas in order to support conservation e�ortsand uplift the lives of local people.

3.1.7.1 The attractions of touring South Africa

South Africa is a beautiful country that boasts great diversity in its �ora and fauna. There are manyinteresting cultural, historical and environmental place that people from South Africa and other countrieswant to visit.

From what you learned from the di�erent ecosystems, you can see that South Africa has a range ofsystems from desert, wetland, mountains, sea and our own unique Fynbos biome.

South Africa encompasses about 1,200,000 km 2 and has about 10% of all plant species on Earth. It isthe third most biodiverse country in the world, and together with seventeen other countries, is consideredmega diverse which means those countries contain 70% of the planet's biodiversity. South Africa's uniquegeography allows the country to support such a diverse population of plants and animals. This makes SouthAfrica an interesting travel destination to many.

3.1.7.2 Bene�ts to visitors, locals and the environment

Eco-tourism is a mutually bene�cial practice for visitors, locals and the environment.Eco-tourism has the potential to alleviate poverty in South Africa through bringing money into the

economy and creating jobs for locals, while at the same time turning our great biodiversity and naturalresources into a national asset that will be nurtured, protected and grown. Tourism is the fastest growingpart of the South African economy. In fact, tourism generates more money in South Africa than gold mining.

3.1.7.3 Ethical Issues

While tourism has great economic potential and gives people access to unique places and cultures, it canhave a negative impact. Sensitive ecosystems such as wetlands and coasts need to be protected so that thebalance of organisms can be maintained. Too many visitors and visitors who are not informed about theirimpact on the environment can have a harmful e�ect. In the same way tourists need to be sensitive to thecultures and people that they visit.

To protect the plants and animals in the unique ecosystems of South Africa, many areas have been declaredNational Parks and have strict rules about how to behave. You can visit the South African National Parkswebsite to learn more about them: www.sanparks.org 52

In the same way, places that are historically or culturally important have been declared national heritagesites that are protected and maintained. South Africa is also proud to have eight UNESCO (The UnitedNations Educational, Scienti�c and Cultural Organisation) sites:

CULTURAL• Fossil Hominid Sites of Sterkfontein, Swartkrans, Kromdraai, and Environs (1999)• Mapungubwe Cultural Landscape (2003)• Robben Island (1999)

49http://www.mhhe.com/biosci/genbio/tlw3/eBridge/Chp29/animations/ch29/1_nitrogen_cycle.swf50https://www.classzone.com/books/ml_science_share/vis_sim/em05_pg20_nitrogen/em05_pg20_nitrogen.html51This content is available online at <http://cnx.org/content/m43191/1.1/>.52http://www.sanparks.org/

Available for free at Connexions <http://cnx.org/content/col11410/1.3>

220 CHAPTER 3. ENVIRONMENTAL STUDIES

• Richtersveld Cultural and Botanical Landscape (2007)MIXED• UKhahlamba / Drakensberg Park (2000)NATURAL• Cape Floral Region Protected Areas (2004)• Greater St. Lucia Wetland Park (1999)• Vredefort Dome (2005)You can �nd out more about them here: http://www.sa-venues.com/unesco_world_heritage_sites.htm

53

3.1.7.4 How to be a responsible ecotourist

Many areas of South Africa are protected and to travel to these areas you need to respect the area and thepeople that you are visiting. These are a few tips:

• Learn a little about the place you are visiting before you go to know the do's and don'ts. For example,littering is not allowed in any National Park in South Africa.

• South Africa is rich in cultural diversity, which means that people from di�erent areas have di�erentways of doing things. Learn about the culture of local people so that you can make sure not to o�endanyone by your behaviour.

• When you are in a protected area, do not damage plants or animals or buildings. For example, writinggra�ti on historical buildings or sites. Remember the saying �take only pictures, leave only footprints�.

http://ethemes.missouri.edu/themes/1382 54

This an interesting website where you can learn about being a responsible tourist

53http://www.sa-venues.com/unesco_world_heritage_sites.htm54http://ethemes.missouri.edu/themes/1382

Available for free at Connexions <http://cnx.org/content/col11410/1.3>

221

3.1.8 3.1.8 Summary55

3.1.8.1 Summary

Biosphere:The biosphere is the region of the earth that encompasses all living organisms: plants, animals and

bacteria. The biosphere includes the outer region of the earth (the lithosphere) and the lower region of theatmosphere (the troposphere) as well as the lakes, oceans, streams, ice and clouds comprising the earth'swater resources (hydrosphere).

Lithosphere:The layer of the mantle above the atmosphere plus the entire crust make up a region called the lithosphere.Hydrosphere:The Hydrosphere contains all the water on Earth. As groundwater, the hydrosphere penetrates the soil

as far down as bedrock. It is found in aquifers as groundwater and also between soil particles. As surfacewater, it is found in wetlands, marshes, estuaries, lakes, streams, rivers, lakes, seas, and oceans. In theatmosphere, water is found as a gas throughout the di�erent regions.

Atmosphere:The gaseous layer that surrounds the earth. It extends outward to about 1,000 kilometers where it

transitions to interplanetary space.Biomes Biomes are, the categories into which ecologists organise similar communities of plants, animals,

and the environmental conditions in which they live. The four major types of biomes are aquatic, grasslands,forests, and desert. Aquatic biomes are probably the most important of all the biomes. Their medium, water,is a major natural resource.

The distribution of plants and animals around the world is anything but random. Instead, it is a result ofthe interplay of individual environmental tolerances of species and the environmental conditions, especiallyvariations in temperature and precipitation. These interactions result in

EnviromentIn this unit we explore the di�erent abiotic and biotic factors that interact or the relationships within an

ecosystem. Also we have included a variety of possible investigations and activities to explore each factorwithin any given ecosystem.

Energy �owIn this unit, you have learnt that organisms get their energy either directly or indirectly from the sun.All organisms �t together in a complicated food web depending on whether they supply energy or get

energy from each other. Food webs are made up of food chains that show the �ow of energy from producers(plants) to consumers (animals) to decomposers (bacteria).

The �ow of nutrients between the atmosphere and organisms is called nutrient cycling.The four importantcycles for life on earth are the water, carbon, oxygen and nitrogen cycles.

55This content is available online at <http://cnx.org/content/m43081/1.1/>.

Available for free at Connexions <http://cnx.org/content/col11410/1.3>

222 CHAPTER 3. ENVIRONMENTAL STUDIES

Available for free at Connexions <http://cnx.org/content/col11410/1.3>

Chapter 4

Diversity, change and continuity

4.1 Biodiversity and classi�cation

4.1.1 4.1.1 Classi�cation Schemes1

4.1.1.1 Biodiversity and Continuity

4.1.1.1.1 Classi�cation Schemes

Image of classi�cation:http://www.tutorvista.com/content/biology/biology-iii/animal-kingdom/animal- classi�cation.php 2

AP Biology: Microbe evolution and classi�cation videohttp://outreach.mcb.harvard.edu/materials.htm 3

Arti�cial classi�cation systems, such as the grouping of vehicles into those that provide transport on land/ water / air etc., are based on arbitrary groupings and have little meaning. The biological classi�cationsystem, however, is based on research in biology, chemistry, genetics, etc. It is a scienti�c method ofclassi�cation used in biology to group similar organisms that share common features and is and is moreuniversally accepted.

It is always necessarily hierarchical where the important features inherited from a common ancestordetermine the group in which the organisms are placed. For example humans and whales both feed theiryoung on milk and it is a characteristic inherited from a common ancestor which places them under the sameclass �mammals� even though their habitat is completely di�erent.

Each organism is grouped into one of 5 large groups or kingdoms , which are subdivided into smallergroups called phyla (singular: phylum) and then smaller and smaller groups with other names.

KingdomPhylumClassOrderFamilyGenusSpecies

4.1.1.1.2 History of Classi�cation

Aristotle a 4th century (384 to322 BC) Greek philosopher divided organisms into two main groups namelyplants and animals. His system was used into the 1600's. People who wrote about animals and plants either

1This content is available online at <http://cnx.org/content/m43187/1.1/>.2http://www.tutorvista.com/content/biology/biology-iii/animal-kingdom/animal-classi�cation.php3http://outreach.mcb.harvard.edu/materials.htm

Available for free at Connexions <http://cnx.org/content/col11410/1.3>

223

224 CHAPTER 4. DIVERSITY, CHANGE AND CONTINUITY

used their common names in various languages or adopted more-or-less standardized descriptions. CasparBauhin (1560�1624) took some important steps towards the binomial system by modifying many of the Latindescriptions to two words.

Figure 4.1

Carolus Linnaeus (Carl Von Linne) was an 18th century (1707�1778) Swedish botanist and physi-cian. He classi�ed plants and animals according to similarities in form and divided living things into twomain kingdoms namely - plant and animal kingdoms. He named the plants and animals in latin or usedlatinised names in his book Species Plantarum (1753) and Systema Naturae (1758).

You can watch a video about Carolus Linnaeus at Photo of Carolus Linnaeushttp://www.youtube.com/watch?v=Gb_IOSzLgk&#38;feature=related

Since Latin was once the universal language of science among western scholars in medieval Europe thesenames were typically in Latin. Latin is used to name and classify living organisms, since it is a dead language- it's no longer changing and is regarded as international.

His classi�cation system is still used today; however, we use a �ve kingdom system instead of two kingdomsystem.

He designed a scienti�c system of naming organisms called binomial nomenclature ("bi-two, nomial-names�. He gave each organism a two part scienti�c name - genus (plural = genera) and species (plural= species) names. The genus and species names would be similar to your �rst name and surname. Genusname is always written with a capital letter whereas species name is written with a small letter. Speciesname belongs to that speci�c type of organism which are only able to interbreed and produce fertile o�spring.The scienti�c name must always be either underlined or written in italics.

For example the scienti�c name of the African elephant is...

Available for free at Connexions <http://cnx.org/content/col11410/1.3>

225

Figure 4.2: Loxodonta africana

Genus Specieshttp://www.�ickr.com/photos/12333120@N00/3679975496/sizes/l/in/photostream/ 4

An organism will always have only one scienti�c name even though they might have more than onecommon name. For example � Blue crane, indwe (for amaXhosa) and mogolori (for Batswana) are allcommon names for South Africa's national bird. However, it has got only one scienti�c name which isAnthropoides paradiseus.

The scienti�c name of our human race is Homo sapiens . We are the only surviving members of thegenus Homo � other more ancient / ancestral types have all become extinct, such as Homo ergaster andHomo neanderthalensis.

4.1.1.1.2.1 Prokaryotes and eukaryotes

Prokaryotes are uni- or multicellular organisms made up of cells that do not have a nuclear envelope (pro-,�before�, karyon, �nucleus�). The genetic material is not bound in a nucleus. They also lack cell organellessuch as an endoplasmic reticulum, a Golgi apparatus, lysosomes, and mitochondria. Prokaryotes are dividedinto two main groups namely the Bacteria and the Archaea (ancient bacteria).

Eukaryotes are multicellular organisms made up of cells (eu-, �true�, karyon, �nucleus�) that possess amembrane- bound nucleus (that holds genetic material) as well as membrane-bound cell organelles. Geneticmaterial in eukaryotes is contained within a nucleus. Eukaryotic organisms include organisms such as plants,animals, fungi, and protists.

Table: Di�erences between prokaryotes and eukaryotes.Eukaryotes

Small cells Large cells

Unicellular or multicellular Often (but not always) multicellular

Genetic material is not contained within a nucleus Genetic material is contained in a membrane-boundnucleus

4http://www.�ickr.com/photos/12333120@N00/3679975496/sizes/l/in/photostream/

Available for free at Connexions <http://cnx.org/content/col11410/1.3>

226 CHAPTER 4. DIVERSITY, CHANGE AND CONTINUITY

Lacks cell organelles such as endoplasmic reticulum, a Golgi apparatus, lysosomes, and mitochondriaHas cell organelles

Divided into bacteria and Archea Divided into protists, fungi, plants and animals.

Eukaryotes

Small cells Large cells

Unicellular or multicellular Often (but not always) multicellular

Genetic material is not contained within a nucleus Genetic material is contained in a membrane-boundnucleus

Lacks cell organelles such as endoplasmic reticulum, a Golgi apparatus, lysosomes, and mitochondriaHas cell organelles

Divided into bacteria and Archea Divided into protists, fungi, plants and animals.

Eukaryotes

Small cells Large cells

Unicellular or multicellular Often (but not always) multicellular

Genetic material is not contained within a nucleus Genetic material is contained in a membrane-boundnucleus

Lacks cell organelles such as endoplasmic reticulum, a Golgi apparatus, lysosomes, and mitochondriaHas cell organelles

Divided into bacteria and Archea Divided into protists, fungi, plants and animals.

Prokaryotes Eukaryotes

Small cells Large cells

Unicellular or multicellular Often (but not always) multicellular

Genetic material is not contained within a nucleus Genetic material is contained in a membrane-bound nucleus

Lacks cell organelles such as endoplasmic reticulum, a Golgi apparatus, lysosomes, and mitochondria Has cell organelles

Divided into bacteria and Archea Divided into protists, fungi, plants and animals.

Table 4.1

Available for free at Connexions <http://cnx.org/content/col11410/1.3>

227

4.1.2 4.1.2 Five kingdom system5

4.1.2.1 Five kingdom system

This is the most common way of grouping living things based on simple distinctive characteristics. Clas-si�cation systems are always changing as new information is made available. Modern technologies such aselectron microscopy make it possible to observe microscopic organisms in greater detail. The current systemwas developed by Robert H. Whittaker in 1969 and was built on the work of previous biologists such asCarolus Linnaeus. The highest grouping is called a kingdom.

Five kingdoms: http://www.tutorvista.com/content/biology/biology-iii/modern-classi�cation/�ve-kingdom-classi�cation.php 6

Bug scope: Images of microscopic organisms http://bugscope.beckman.uiuc.edu/ 7

Neok12: Animals and wildlife videos http://www.neok12.com/Animals-Wildlife.htm 8

Encyclopedia of life: Images and explanations of terms http://eol.org/index 9

Living things can be classi�ed into �ve major kingdoms:

5This content is available online at <http://cnx.org/content/m43221/1.1/>.6http://www.tutorvista.com/content/biology/biology-iii/modern-classi�cation/�ve-kingdom-classi�cation.php7http://bugscope.beckman.uiuc.edu/8http://www.neok12.com/Animals-Wildlife.htm9http://eol.org/index

Available for free at Connexions <http://cnx.org/content/col11410/1.3>

228 CHAPTER 4. DIVERSITY, CHANGE AND CONTINUITY

Figure 4.3

Kingdom Monera (Bacteria)Kingdom ProtistaKingdom FungiKingdom PlantaeKingdom Animaliahttp://www.youtube.com/watch?v=5uJ8QeFRvJA&feature=related 10 A video showing a brief sum-

mary of the �ve kingdoms

10http://www.youtube.com/watch?v=5uJ8QeFRvJA&feature=related

Available for free at Connexions <http://cnx.org/content/col11410/1.3>

229

4.1.2.1.1 Distinctive Features of the �ve kingdoms

4.1.2.1.1.1 Kingdom Monera

• Prokaryotic, unicellular.• No nuclear membrane or membrane bound organelles such as

chloroplasts, Golgi complex, mitochondria and endoplasmic reticulum.

• Have a cell wall made without cellulose.• Reproduction is mainly asexual by binary �ssion.• Important examples: Archaea, cyanobacteria (blue-green algae), bacteria

Interesting fact: Bacteria are found everywhere and are the most numerous organisms on Earth. In a singlegram of soil, there are about 40 million bacterial cells. The human body also contains 10 times as manybacterial cells as human cells!!

4.1.2.1.1.2 Kingdom Protista

• Eukaryotic, can be unicellular or simple multicellular.• Reproduction can be asexual or sexual.• Important examples: Plasmodium (causes malaria), amoeba, euglena

4.1.2.1.1.3 Kingdom Fungi

• Eukaryotic, multicellular (some unicellular like yeasts).• Have a cell wall made of chitin.• Non-motile with long extensions called hyphae.• Nutrition is heterotrophic: important as decomposers (saprophytes), can be parasitic.• Store food as glycogen• Reproduction is by spore formation (both asexual and sexual).• Important examples: Mushrooms, Penicillium (a fungus which was used to make penicillin), bread

mould

http://blog.ted.com/2008/05/06/paul_stamets/ 11 A TED video on the many uses of Fungi

4.1.2.1.1.4 Kingdom Plantae

• Eukaryotic, multicellular.• Distinct cell wall made of cellulose.• Have plastids and photosynthetic pigments such as chlorophyll.• Non-motile.• Nutrition is autotrophic (make their own food by photosynthesis).

11http://blog.ted.com/2008/05/06/paul_stamets/

Available for free at Connexions <http://cnx.org/content/col11410/1.3>

230 CHAPTER 4. DIVERSITY, CHANGE AND CONTINUITY

• Sexual reproduction.• Important examples: Green algae, mosses, ferns, conifers, �owering plants.

4.1.2.1.1.5 Kingdom Animalia

• Eukaryotic and multicellular but have no cell wall or photosynthetic pigments• Mostly motile• Heterotrophic nutrition.• Sexual or vegetative (asexual) reproduction• Important examples: Porifera (sponges), Mammalia, Insects

Additional resource:

• Tree of life project: collaborative e�ort of biologists and nature enthusiasts from around the worldproviding information about biodiversity, the characteristics of di�erent groups of organisms, and theirevolutionary history ( phylogeny 12 ). Link: http://tolweb.org/tree/phylogeny.html 13

• ARKive project: For pictures and information on a wide range of life forms• http://bugscope.beckman.uiuc.edu/ 14 : For high magni�cation pictures of insects using a scanning

electron microscope.• http://www.neok12.com/Microorganisms.htm 15 : For interactive videos on microorganisms.

Projects and assignments:1. Research one bene�cial and one harmful application of one member from each kingdom, with examples

from their use in South Africa. Students can be grouped into smaller groups and each one is given onekingdom to research. (Use www.arkive.org 16 as a research tool for your favourite animal or plant orhttp://bugscope.becnkman.uiuc.edu/ 17 for nice pictures of insects). Results can be presented in the formof a poster.

2. Go to your nearest supermarket or garden and �nd one representative organism for each kingdom.Present this information by drawing diagrams.

12http://tolweb.org/tree/learn/concepts/whatisphylogeny.html13http://tolweb.org/tree/phylogeny.html14http://bugscope.beckman.uiuc.edu/15http://www.neok12.com/Microorganisms.htm16http://www.arkive.org/17http://bugscope.becnkman.uiuc.edu/

Available for free at Connexions <http://cnx.org/content/col11410/1.3>

231

4.1.3 4.1.3 Summary18

4.1.3.1 Summary

In Section 4.1 Biodiversity and Classi�cation you learnt:

• De�nition of biological classi�cation and hierarchical grouping of living organisms based on similaritiesand di�erences.

• History of classi�cation and scienti�c naming of organisms using the binomial nomenclature. Allorganisms have only ONE scienti�c name but many common names!

• Division of organisms into prokaryotes (simple, Unicellular) and Eukaryotes (mostly multicellular) andthe major di�erences between the two.

• Classi�cation of living organisms into 5 major kingdoms: - Monera, Protista, Fungi, Plantae andAnimalia and their unique characteristics.

4.2 History of life on Earth

4.2.1 4.2.1 Life's History19

4.2.1.1 Introduction

In this section you will be introduced to some scienti�c theories about life's history. One of the populartheories of life's history is the theory of Evolution. Another is the theory of Intelligent Design. In order tobe able to evaluate information critically, it is important to �rst understand how people form knowledge,and to be able to di�erentiate between data and conclusions.

4.2.1.1.1 How do we know?

How do we know what happened in life's history? We cannot do experiments on the origin of life. Thereare also no historical records about the origin of life. We have to rely on data we �nd. From this we drawconclusions about what might have happened in life's history.

http://www.youtube.com/watch?v=pXs-693ERSc

How do we know?

This media object is a Flash object. Please view or download it at<http://www.youtube.com/v/pXs-693ERSc?version=3&hl=en_US>

Figure 4.4: This is a clip movie about how people get knowledge in general, and how people can getknowledge about origins, in particular.

4.2.1.1.2 Data and Conclusions

Data means information people collect using their senses : sight, touch, hearing, feeling, smell. Usuallywhen a scientist collects data, other scientists will agree with him/her about this data. Sometimes otherscientists might question whether the data was correctly recorded, or whether the data is a forgery, butusually scientists trust that the data was collected correctly.

18This content is available online at <http://cnx.org/content/m43088/1.1/>.19This content is available online at <http://cnx.org/content/m43241/1.1/>.

Available for free at Connexions <http://cnx.org/content/col11410/1.3>

232 CHAPTER 4. DIVERSITY, CHANGE AND CONTINUITY

Conclusions are patterns people think up to help to make sense of data. When a scientist draws aconclusion from some data, he/she makes various assumptions. Assumptions are thoughts which people taketo be true, without proof. Assumptions should be justi�ed so that people can evaluate their validity(how likely they are to be true). It is common for scientists to disagree on the validity of assumptionsand conclusions, even when they do agree on the data from which the conclusions are made. This is becausedi�erent conclusions can often be drawn from the same data.

http://www.youtube.com/watch?v=AVCPfrp-VDo

Data Conclusion

This media object is a Flash object. Please view or download it at<http://www.youtube.com/v/AVCPfrp-VDo?version=3&hl=en_US>

Figure 4.5

4.2.1.2 Life's History

APBiology: Molecular Evolution and the early earth: http://outreach.mcb.harvard.edu/materials.htm 20

The extremely long period of time over which life has developed on earth can be represented in variousways. Examine the diagrams below:

20http://outreach.mcb.harvard.edu/materials.htm

Available for free at Connexions <http://cnx.org/content/col11410/1.3>

233

Figure 4.6: Taken from http://combattemple.com/clamping-102420-geologist-clyde-smith/ 21

The various time periods are related to dramatic climate changes that the earth has experienced overtime. This is partly due to what is called continental drift :

4.2.1.2.1 Continental drift:

This theory proposed that all land was at one stage joined to form the supercontinent Pangaea ,which split into Laurasia in the north and Gondwana (or Gondwanaland) in the south. Seehttp://www.exploratorium.edu/origins/antarctica/ideas/gondwana2.html 22

There is much evidence that continental drift occurred and is still continuing today:There is biogeographic evidence of related species in widely isolated areas, such as the very similar

�ightless birds like the rhea in South America, the ostrich in Africa, the moa in New Zealand, the emu and

21http://combattemple.com/clamping-102420-geologist-clyde-smith/22http://www.exploratorium.edu/origins/antarctica/ideas/gondwana2.html

Available for free at Connexions <http://cnx.org/content/col11410/1.3>

234 CHAPTER 4. DIVERSITY, CHANGE AND CONTINUITY

cassowary in Australia. They are thought to have developed from a common ancestor on Pangaea. As theclimate gradually changed, organisms slowly adapted and underwent speciation in response to changes inthe temperature and vegetation around them.

OTHER EVIDENCE for continental drift include the following, showing that the climate in some areasis now very di�erent from what it once was:

• The discovery of fossilized tropical plants under Greenland's ice caps• Glacial landscapes in central Africa and Central America• Whale fossils in the Sahara desert• The discovery of subtropical plant fossils in Antarctica, indicating that it once had a much warmer

climate and lush vegetation.• South African examples of continental drift include the discovery of the fossils of marine organisms in

places that are VERY far from the sea, such as bivalves and ammonites in the Makhatini �ats innorthern KZN, and marine trilobite fossils in the Karoo.

Figure 4.7: Ammonite fossil

Figure 4.8: Trilobite fossil

Available for free at Connexions <http://cnx.org/content/col11410/1.3>

235

4.2.1.2.2 Plate tectonics

This theory provides a mechanism for continental drift. The continents we know today rest on large,interlocking plates of land called tectonic plates, which �oat on a hot, molten layer that shifts them. Slowmovements of these plates move continents further apart, but more rapid movements of the plates are evidentwhen earthquakes occur. The continents are still moving apart at the rate of just a few cm per year.

Available for free at Connexions <http://cnx.org/content/col11410/1.3>

236 CHAPTER 4. DIVERSITY, CHANGE AND CONTINUITY

4.2.2 4.2.2 Geological timescale23

4.2.2.1 Geological Timescale

Figure 4.9

23This content is available online at <http://cnx.org/content/m43214/1.2/>.

Available for free at Connexions <http://cnx.org/content/col11410/1.3>

237

Note that the time scale above shows the Carboniferous period as represented by the Pennsylvanian andMississippian separately. This is not always done.

Geological time: interactive site: http://learn.genetics.utah.edu/content/variation/time/ 24

Geological time: http://www.pbs.org/wgbh/evolution/change/deeptime/index.html 25

4.2.2.1.1 The three eras of the geological time scale

The earth's history can be traced over MANY millions of years, so scientists have developed a geological timescale to help visualize these periods. This vast amount of time is divided into eons, eras and periodsfor easier reference. You MUST know the names of the three ERAS:

So, for example, the most recent eon is divided into 3 eras, called thePaleozoic (meaning ancient life)Mesozoic (meaning middle life)Caenozoic (meaning recent life)You don't have to memorize the periods, only the eras . It is important that you become familiar with

these names, so that you can use such information in a test or exam. The end of each era is marked by aseries of catastrophic extinctions, which wiped out many of the previously successful species. Examine thediagram below, showing eras & periods.

Figure 4.10

24http://learn.genetics.utah.edu/content/variation/time/25http://www.pbs.org/wgbh/evolution/change/deeptime/index.html

Available for free at Connexions <http://cnx.org/content/col11410/1.3>

238 CHAPTER 4. DIVERSITY, CHANGE AND CONTINUITY

(Ma = million years)Examine the pie chart and the table, which has the main events of each periodCambrian periodExplosion of multi-cellular life, many trilobites in seas, modern groups develop

Ordovician and Silurian periodsInvertebrates with shells, �rst �sh, �rst plants with vascular tissue

Devonian periodArthropods on land, �rst trees, many primitive �sh, �rst amphibians and insects

Carboniferous periodCoal formed, Gondwana is under ice sheets, 1st reptiles develop, many diverse insects

Permian periodGlossopteris trees in Gondwana, many marine Molluscs, mammal-like reptiles

MESOZOICTriassic periodFirst dinosaurs, �rst small mammals develop

Jurassic periodDinosaurs develop many forms, 1st birds develop, conifers form, ammonites in seas

Cretaceous periodFlowering plants and insects evolve, more dinosaurs develop, placental mammals

CAENOZOICMany di�erent forms of mammals and birds develop, the earth cools down after widespread heating,

modern animals develop, hominids develop

Cambrian periodExplosion of multi-cellular life, many trilobites in seas, modern groups develop

Ordovician and Silurian periodsInvertebrates with shells, �rst �sh, �rst plants with vascular tissue

Devonian periodArthropods on land, �rst trees, many primitive �sh, �rst amphibians and insects

Carboniferous periodCoal formed, Gondwana is under ice sheets, 1st reptiles develop, many diverse insects

Permian periodGlossopteris trees in Gondwana, many marine Molluscs, mammal-like reptiles

MESOZOICTriassic periodFirst dinosaurs, �rst small mammals develop

Jurassic period

Available for free at Connexions <http://cnx.org/content/col11410/1.3>

239

Dinosaurs develop many forms, 1st birds develop, conifers form, ammonites in seas

Cretaceous periodFlowering plants and insects evolve, more dinosaurs develop, placental mammals

CAENOZOICMany di�erent forms of mammals and birds develop, the earth cools down after widespread heating,

modern animals develop, hominids develop

Cambrian periodExplosion of multi-cellular life, many trilobites in seas, modern groups develop

Ordovician and Silurian periodsInvertebrates with shells, �rst �sh, �rst plants with vascular tissue

Devonian periodArthropods on land, �rst trees, many primitive �sh, �rst amphibians and insects

Carboniferous periodCoal formed, Gondwana is under ice sheets, 1st reptiles develop, many diverse insects

Permian periodGlossopteris trees in Gondwana, many marine Molluscs, mammal-like reptiles

MESOZOICTriassic periodFirst dinosaurs, �rst small mammals develop

Jurassic periodDinosaurs develop many forms, 1st birds develop, conifers form, ammonites in seas

Cretaceous periodFlowering plants and insects evolve, more dinosaurs develop, placental mammals

CAENOZOICMany di�erent forms of mammals and birds develop, the earth cools down after widespread heating,

modern animals develop, hominids develop

Available for free at Connexions <http://cnx.org/content/col11410/1.3>

240 CHAPTER 4. DIVERSITY, CHANGE AND CONTINUITY

PALEOZOIC Cambrian period Explosion of multi-cellular life, many trilobites in seas, modern groups develop

Ordovician and Silurian periods Invertebrates with shells, �rst �sh, �rst plants with vascular tissue

Devonian period Arthropods on land, �rst trees, many primitive �sh, �rst amphibians and insects

Carboniferous period Coal formed, Gondwana is under ice sheets, 1st reptiles develop, many diverse insects

Permian period Glossopteris trees in Gondwana, many marine Molluscs, mammal-like reptiles

MESOZOIC Triassic period First dinosaurs, �rst small mammals develop

Jurassic period Dinosaurs develop many forms, 1st birds develop, conifers form, ammonites in seas

Cretaceous period Flowering plants and insects evolve, more dinosaurs develop, placental mammals

CAENOZOIC Many di�erent forms of mammals and birds develop, the earth cools down after widespread heating, modern animals develop, hominids develop

Table 4.2

4.2.2.1.2 The following are the major events in each era of the time scale:

• During the Pre-Cambrian time, life in general consisted of bacteria, simple algae and simple unicellularorganisms. The best examples of such early life are the stromatolites, large mounds of cyanobacteriain sediment along the continental shelf that oxygenated the early atmosphere and allowed other aerobiclife forms to exist.

Figure 4.11: Stromatolites

• Paleozoic : Started with an �explosion� of multicellular life, called the Cambrian Explosion. Marinetrilobites were common. The �rst invertebrates and �sh are found, later the �rst amphibians andinsects. Diverse land plants develop and coal swamps form in certain areas. The �rst reptiles form.This era ended with the massive Permian Extinction, wiping out many successful species

Available for free at Connexions <http://cnx.org/content/col11410/1.3>

241

Figure 4.12: Several trilobite fossils

• Mesozoic : The Mesozoic era starts with the Triassic period, which saw the rise of the dinosaursas the world's dominant organisms. The dominant plants were Gymnosperms (cone bearers, likecycads and pine trees). Later, the �rst mammals develop and �owering plants or Angiosperms areformed. Birds develop, as well as the �rst placental mammals. This era ends with the CretaceousExtinction (the so-called KT-boundary), that wiped out the dinosaurs 65 million years ago.

Dinosaur

Available for free at Connexions <http://cnx.org/content/col11410/1.3>

242 CHAPTER 4. DIVERSITY, CHANGE AND CONTINUITY

Figure 4.13: cycad

• Coenozoic : Mammals developed further, including primates. Development of birds and �oweringplants is evident. Global cooling occurs (i.e. the most recent ice ages, which caused a drop in sealevels. This allowed development of land bridges between North America and Asia, also between Indiaand Sri Lanka and between Australia and the islands to its north. These land bridges assisted greatlyin the migration of species to new land masses).

Hominid evolution started during the Coenozoic. This will be discussed more fully in Grade 12.

Available for free at Connexions <http://cnx.org/content/col11410/1.3>

243

Figure 4.14

Figure 4.15

Two types of �owering plants (Angiospermae)

Available for free at Connexions <http://cnx.org/content/col11410/1.3>

244 CHAPTER 4. DIVERSITY, CHANGE AND CONTINUITY

Figure 4.16: Models of Australopithecus, an early hominid

Figure 4.17: Baby mammoth

Available for free at Connexions <http://cnx.org/content/col11410/1.3>

245

4.2.2.1.2.1 Critical evaluation

The geological column summarises a commonly accepted evolutionary theory about life's history. As withmost scienti�c theories, not all scientists agree about the theory. Criticisms include the following:

• The sequence (order) of fossils given in the geological column is not found anywhere on earth.Instead, there are many places where `older' layers are found on top of `younger' layers.

• There are many gaps in every sequence. To explain this, palaeontologists suggest that about 2/3 ofall the kinds of organisms that ever lived could not have left any fossils. (Palaeontologists are scientistswho study fossils and try to use then to reconstruct the past.) It is reasonable to expect that manysoft-bodied creatures would not have been fossilised. However, it is strange that there are gaps in thefossil records even for creatures with hard shells and strong bones.

• The dating methods used to date rocks might not give valid dates. This is discussed in the sectionabout fossils and dating.

4.2.2.1.2.2 THE �MISSING LINK� BETWEEN DINOSAURS AND BIRDS

It's been believed for many years that modern birds developed from reptilian ancestors, as dinosaur fossilsoften showed AVIAN or `bird-like' features. There was thus much excitement when the �rst fossil of a�missing link� between birds and dinosaurs was found, the so-called Archaeopteryx. Around 10 such fossilshave been found to date. Read the information below:

Figure 4.18

Available for free at Connexions <http://cnx.org/content/col11410/1.3>

246 CHAPTER 4. DIVERSITY, CHANGE AND CONTINUITY

Figure 4.19

Left is a specimen of Archaeopteryx , and right is an artist's impression of the skeleton in an uprightposition. Note the very �ne feather impressions, including the �ight feathers of the wings. Despite its obviousavian nature, Archaeopteryx has a hand virtually identical to other dinosaurs such as Velociraptor .

Available for free at Connexions <http://cnx.org/content/col11410/1.3>

247

Figure 4.20: Artist's impression of the animal

Available for free at Connexions <http://cnx.org/content/col11410/1.3>

248 CHAPTER 4. DIVERSITY, CHANGE AND CONTINUITY

Figure 4.21: A mounted archeopteryx model. Note the teeth.

Exercise 4.2.2.1 (Solution on p. 281.)

Use the pictures below and on the previous page to compare the skeletons of adinosaur, Archaeopteryx and a modern bird. In your answer, give di�erences andsimilarities between Archaeopteryx and dinosaurs, and between Archaeopteryx and birds.

Available for free at Connexions <http://cnx.org/content/col11410/1.3>

249

Available for free at Connexions <http://cnx.org/content/col11410/1.3>

250 CHAPTER 4. DIVERSITY, CHANGE AND CONTINUITY

4.2.2.1.2.2.1 Activity: Critical Analysis of Archaeopteryx

Some scientists consider Archaeopteryx to be a transitional fossil between dinosaurs and birds. Otherscientists disagree. Below are two arguments given by scientists to support their interpretation that Ar-chaeopteryx is not a transitional fossil between dinosaurs and birds. For each of these arguments, statewhether the scientists are questioning the DATA, or the CONCLUSIONS about the Archaeopteryx fossil.

Exercise 4.2.2.2 (Solution on p. 284.)

a. Watkins, Hoyle, Wickramasinghe, Watkins, Rabilizirov and Spetner (1985) published an arti-cle where they claimed that the feather impressions of Archaeopteryx were forged. They pro-vide evidence suggesting that chicken feathers were pressed into a cement mix, and these wereapplied over dinosaur fossils: Watkins, R.S., Fred Hoyle, N.C. Wickramasinghe, J. Watkins,R. Rabilizirov, and L.M. Spetner (1985), � Archaeopteryx: A Photographic Study,� BritishJournal of Photography, 132:264-266, March 8. These scientists are questioning the DATA /CONCLUSIONS made by other scientists about Archaeopteryx.

b. Some scientists argue that even if the feathers on Archaeopteryx were not forged, there is noevidence that the fossil is transitional between reptiles and birds, since the fossil is compatiblewith the fossil of a bird. These scientists are questioning the DATA / CONCLUSIONS madeby other scientists about Archaeopteryx.

c. Some scientists argue that birds have been found in younger rocks than the rocks in whichArchaeopteryx was found, and so Archaeopteryx cannot be the ancestor to birds. Thesescientists are questioning the DATA / CONCLUSIONS made by other scientists about Ar-chaeopteryx.

4.2.2.1.2.3 The �missing link� between �sh and amphibians:

In 1938, an East London �sherman caught a deep-sea �sh he had never seen, so it was taken to the EastLondon museum, where it was identi�ed as a coelacanth by Professor JLB Smith and his assistant, MarjorieCourtenay-Latimer. This caused an international uproar, because the �sh was previously known only fromfossils and had been believed to be extinct. People were amazed that fossil coelacanth found in rock strata70 �350 million years old could look exactly like the large blue �sh before their eyes! This started a searchfor more specimens and 14 years later, another one was caught o� the Comoro Islands in the Indian Ocean,north of Madagascar and more have been found since, including in deep crevices at St. Lucia in northernKZN.

Available for free at Connexions <http://cnx.org/content/col11410/1.3>

251

Figure 4.22: Coelacanth display

Figure 4.23: Madge Courtenay-Latimer and the coelacanth

Available for free at Connexions <http://cnx.org/content/col11410/1.3>

252 CHAPTER 4. DIVERSITY, CHANGE AND CONTINUITY

Scientists agree that the coelacanth is probably the oldest living vertebrate and they are thought tobe closely related to the freshwater �sh that is seen as the ancestor of all land animals. The coelacanth'sinternal organs are in many ways similar to those of amphibians, which are the earliest life forms to havecolonized land. Unlike amphibians, its spine is supported by a permanent notochord, a cartilage and oiltube � it never develops a bony vertebral column like other �sh do. The body is lobe-�nned, not ray-�nnedlike modern �sh, although it is not known to �walk� on the ocean �oor. Many scientists had thought thatthe coelacanth's �ns looked like �legs� (it was referred to as `old four legs') and that it was in the process ofdeveloping limbs for use on land, but here was a specimen, millions of years later, that still had the same�n structure. Its nostrils are also not blind-ending as in �sh; they open into the mouth and can be used forbreathing.Assignment:The modern coelacanth-�nding throws doubt on an assumption palaeontologists use when reasoning aboutwhat the fossil record says about extinctions.

a. What is the assumption the modern coelacanth-�nding throws doubt on?b. How does the modern coelacanth-�nding throw doubt on this assumption ?

An extinction event can be recognised by fossils of that type not being found in higher (i.e. younger) rock.Although no coelacanth fossils are found in rock younger than that assumed to be 100 million years old,coelacanths are not extinct: they are alive today (extant).

4.2.3 4.2.3 Cambrian Explosion26

4.2.3.1 What does this mean?

Cambrian explosion refers to the tremendous increase in the number and type of multicellular organisms atthe start of the Cambrian period of the Paleozoic era. Ancestral forms of all organisms emerged then, whichhave gradually changed to become the life forms we see today.

Cambrian explosion: explanation http://www.fossilmuseum.net/Paleobiology/CambrianExplosion.htm27

4.2.3.2 Signi�cant changes have taken place in all groups of plants and animals.

4.2.3.2.1 Plants:

From being purely aquatic, plants gradually colonized land and had to cope with life in a completely di�erentmedium. They had to develop several structural changes, such as:

• They developed true roots, stems and leaves with di�erent tissues in these organs. Some plants devel-oped �owers and fruit.

• They developed supporting tissue like xylem and thick secondary walls with lignin.• They developed a means of protecting themselves against dehydration, such as the development of a

cuticle and sunken stomata.• They became less dependent on water for fertilization, using wind and or animals to transfer pollen

grains with sperm nuclei.

4.2.3.2.2 Animals:

Animals also became more complex than their primitive ancestors. Some colonized land and had to makefurther changes.

• They developed a head with a brain and limbs on both sides of the body.

26This content is available online at <http://cnx.org/content/m43205/1.1/>.27http://www.fossilmuseum.net/Paleobiology/CambrianExplosion.htm

Available for free at Connexions <http://cnx.org/content/col11410/1.3>

253

• The digestive system became more complex, developing a gut with two openings and various glandsassociated with digestion.

• They had to protect themselves against dehydration, so many developed internal breathing systems,protecting the delicate lungs.

• Birds and mammals became endothermic, i.e. able to keep their body temperatures constant despitechanges in environmental temperature.

See: http:www.khanacademy.org/video/introducti-to-evolution-and-natural- election?playlist=Biology

4.2.4 4.2.4 Mass extinctions28

4.2.4.1 What does this mean?

When all the members of a species die out the species has become extinct. A species is deduced to be extinctif it stops appearing in apparently later (more recent) rock. There seem to have been times in the earth'shistory when many species have become extinct at about the same time. Such an event is called a massextinction.

4.2.4.2 Five major mass extinctions

Evolutionists think that there were �ve large mass extinctions, with the two largest being:

• The �rst major mass extinction is said to have occurred at the end of the Permian period. This isthought to have been about 225 million years ago. At that time 90% of marine life seems to havebecome extinct.

• The second is thought to have occurred about 65 million years ago, at the end of the Cretaceous Period.This led to the disappearance of the dinosaurs.

Each time a mass extinction seems to have happened, many new kinds of fossils appear in later (more recent)rocks. These are considered new because their fossils were absent from older-dated rocks. Transitionalevolutionary ancestors of these new organisms are also not found in older rock. This sudden appearance ofnew species is explained as follows. After a mass extinction a large part of the environment is cleared of thelife forms which had previously dominated (controlled) the earth. This makes way for new forms to evolveand �ll the many new vacant ecological niches (open spaces in the ecosystem).

4.2.4.2.1 Possible causes of mass extinctions

Mass extinctions can be caused by various factors. These may be extra-terrestrial, due to climate change,caused by humans, genetic, or due to continental drift:

• Extra-terrestrial events are caused by something outside of the earth, e.g. by a meteorite. At least twomass extinctions are thought to have been caused by huge meteorites striking the earth. A meteorite iss chunk of rock from outer space, which enters the earth's atmosphere. These meteorites are thoughtto have been about 10 km in diameter. Some scientists believe that such impacts happened manymillions of years ago. Other data suggests that a meteor of diameter 200km hit the earth only a fewthousand years ago. This does not �t in with the geological time scale and is usually ignored. (Thisis based on observations of the changing tilt of the earth's axis, made over a period covering most ofrecorded history.

• Climate change can cause extinction.• Link to simulation - glaciers http://phet.colorado.edu/en/simulation/glaciers29

28This content is available online at <http://cnx.org/content/m43235/1.1/>.29http://phet.colorado.edu/en/simulation/glaciers

Available for free at Connexions <http://cnx.org/content/col11410/1.3>

254 CHAPTER 4. DIVERSITY, CHANGE AND CONTINUITY

• Human activity can cause extinction. Humans are destroying the habitat of many types of plantsand animals. This is done by chopping down forests and draining swamps to grow crops and to buildhouses.

• Continental drift might have caused extinction. According to the theory of continental drift, all landused to be joined in a supercontinent called Pangaea. This later split into Laurasia in the north andGondwanaland in the south. There is much evidence for this. As the continents moved apart, somepopulations were split and moved apart. This could have caused extinction as habitats and climateschanged as a result of the continental drift.

See http://www.exploratorium.edu/origins/antarctica/ideas/gondwana2.html30

What killed the dinosaurs: http://www.pbs.org/wgbh/evolution/extinction/dinosaurs/31

Biodiversity: the 6th great wave: http://news.bbc.co.uk/2/hi/science/nature/3667300.stm32

Vertebrate extinction crisis (videos): http://www.arkive.org/newsletter/?u=722a3ad7755a24108f2cd402e&id=db50e9fd5d&e=&utm_source=ARKive&utm_campaign=db50e9fd5d-IUCN_Press_Release10_27_2010&utm_medium=email33

4.2.4.2.1.1 Activity: Mass extinctions

1. How do extinction events seem to help the process of evolution?2. Name any four possible causes of mass extinctions.3. Complete this table:

Geological time period when ex-tinction is thought to have oc-curred.

Years before present when evolu-tionists think this happened

Main feature of extinction event

End of the Permian period.

Dinosaurs became extinct.

Table 4.3

Activity: Answers1. How do extinction events seem to help the process of evolution? They possibly cleared the earth of

previously dominant species, allowing new species to evolve into the ecological niches this opened up.2. Name any four possible causes of mass extinctions. Meteorites, volcanoes, climactic changes (e.g. ice age/ global warming), man-caused (e.g. over-hunting), due to genetic deterioration3. Complete this table:

Geological time period when ex-tinction is thought to have oc-curred.

Years before present when evolu-tionists think this happened

Main feature of extinction event

End of the Permian period. 225 million years ago 90% of marine life became extinct

End of Cretaceous era 65 million years ago Dinosaurs became extinct.

Table 4.4

30http://www.exploratorium.edu/origins/antarctica/ideas/gondwana2.html31http://www.pbs.org/wgbh/evolution/extinction/dinosaurs/32http://news.bbc.co.uk/2/hi/science/nature/3667300.stm33http://www.arkive.org/newsletter/?u=722a3ad7755a24108f2cd402e&id=db50e9fd5d&e=&utm_source=ARKive&utm_campaign=db50e9fd5d-

IUCN_Press_Release10_27_2010&utm_medium=email

Available for free at Connexions <http://cnx.org/content/col11410/1.3>

255

4.2.5 4.2.5 Fossils34

4.2.5.1 Introduction

Fossils are traces of organisms which used to be alive. Fossils can tell us something about the creatures theycame from. Hard tissue, such as bone or shell, is fossilized better than softer body parts. This is to beexpected, since soft tissue is eaten or it decays soon after death, leaving the bones and shells to petrify (turnto stone).However, examples of perfectly fossilized soft creatures, like jelly�sh, do exist. In this section, youlearn about theories of how fossils form, types of fossils, how fossils are dated, and fossil tourism. Peoplewho study fossils are called paleontologists.

Neok12: http://www.neok12.com/Fossils.html 35

The way fossils form: Interactive site: http://www.�ashyourbrain.com/pieces/fossils/index.php 36

4.2.5.2 Fossil formation

Di�erent kinds of fossils are formed in di�erent ways. Many form by petrifaction, i.e. turning into stone.The following may petrify:

• Parts of the body• Sediments surrounding the body. Sediments are layers of material (e.g. sand) deposited (dropped) by

wind / water / ice.• Cavities (spaces) left by the body.

Scientists have observed petrifaction occurring both in nature and in laboratory experiments. Min-erals(inorganic compounds) enter the body, hardening it and turning it to stone. Fossilization has beenobserved to happen very quickly when high concentrations of suitable minerals were present, and when theconditions were favourable for certain bacteria which aid the petrifaction process.

4.2.5.3 Fossil types

There are several kinds of fossils. These include footprints, dung, moulds, casts, permineralised, and tracefossils.

Footprints, made when creatures walked in soft sand or mud, can be preserved (kept safe) when the sandor mud dries, hardens and then petri�es. From the footprints we can deduce the shape and size of the feetand how the creature which made them walked. A fossilized dinosaur footprint is shown in Figure 1.

34This content is available online at <http://cnx.org/content/m43253/1.1/>.35http://www.neok12.com/Fossils.html36http://www.�ashyourbrain.com/pieces/fossils/index.php

Available for free at Connexions <http://cnx.org/content/col11410/1.3>

256 CHAPTER 4. DIVERSITY, CHANGE AND CONTINUITY

Figure 4.24: A fossilized dinosaur footprint

Dung can also become petri�ed. Petri�ed dung is called a coprolite. From this we can deduce what kindof food a creature ate.

Mould fossils form around a cavity which the organism used to �ll. A dead creature can be buried insediments. These may petrify, forming a mould around the body. The soft body of the creature decays(rots), leaving the mould empty. The shape of the body can be seen from the mould.

Cast fossils look the same as the original body, except that they are made of minerals. Minerals (e.g.calcite) �ll the cavity inside a mould, making a cast fossil. A cast fossil of a shelled sea animal is shown inFigure 2.

Figure 4.25: A cast fossil of a shelled sea animal

Permineralized fossils form when the body of an organism becomes petri�ed. Water moves through thetissues of the dead creature and leaves minerals behind. These minerals replace the living tissue. Thesefossils can give a very good idea of the original structure of a dead creature. A permineralised fossil of a treestump is shown in Figure3.

Available for free at Connexions <http://cnx.org/content/col11410/1.3>

257

Figure 4.26: Petri�ed wood. This tree stump turned into stone as minerals replaced the living tissue,and hardened

Film fossils form where leaves have left black or brown marks, made of carbon, on rock surfaces. A �lmfossil of a leaf is shown in Figure 4.

Figure 4.27: A �lm fossil of a leaf

Available for free at Connexions <http://cnx.org/content/col11410/1.3>

258 CHAPTER 4. DIVERSITY, CHANGE AND CONTINUITY

4.2.5.4 Fossil dating

4.2.5.4.1 Radiometric dating

Radiometric dating is sometimes used when scientists want to date fossils. Some elements are unstable andbreak down into lighter elements over time. Such substances are said to be radioactive. The original elementis called the parent and the element it changes into is called the daughter. For example, uranium-238 decaysinto lead-206. What is the data which scientists use in this method, and the assumptions they might makewhen drawing conclusions about fossil dates?:

Data: A rock has a certain ratio of the parent to daughter elements, e.g. a certain ratio of uraniumto lead.Conclusions: In radiometric dating, the ratio of parent to daughter elements is compared. From this theage of the rock is concluded.Assumptions: A scientist has to make many assumptions in order to conclude the age of a rock fromratios between elements. These assumptions include:

The rock must have had only parent, and no daughter, elements when the rock formed.The rate at which the parent element changed into the daughter element is known from the rate this occurstoday.For example, if a rock is found containing both uranium-238 and lead-206, it is assumed that all this leadcame from the uranium by radioactive decay and that the rate at which that happened is the same as therate observed today.

4.2.5.4.2 Carbon dating

Carbon dating is the only type of radiometric dating which can be done directly on fossils. However, carbondating gives young ages, and so is generally not accepted, except for dating organisms which have been deadfor less than about 30 000 years.

How does carbon dating work? All organisms incorporate carbon atoms into their bodies when they arealive. This stops at death. There are two isotopes (forms) of carbon. The most common isotope is C12.The other isotope, C14, is radioactive. The proportion C12:C14 in the atmosphere is known. Both carbonisotopes combine with oxygen to form CO2 and both are incorporated into plants during photosynthesis.Therefore both carbon isotopes form part of animals' food. The organism dies. The C12 in the dead bodydoes not break down radioactively, but the C14 in the body does break down radioactively. We can measurethe rate at which C14 breaks down radioactively over time today. Scientists compare the ratio of C12:C14inthe body to the ratio of C12:C14in the atmosphere. This is the data used. The scientists may assume thatthe C12:C14 ratio at the time of death was the same as the atmosphere's C12:C14ratio, and that the rateat which C14broke down in the past was the same as it is today. From this they may conclude the age ofthe rock.

4.2.5.4.3 Dating fossils by dating rocks

For fossils which are believed to be older than 30 000 years, carbon dating cannot be used, so other radiometricdating methods are used. However, these methods can only be done on igneous rocks (rocks formed fromvolcanoes).Igneous rock cannot contain fossils. Fossils are only found in sedimentary rock. In order toconclude the date of a fossil the following process is followed:

The scientist �nds some igneous rock which he assumes to be the same age as the sedimentary rock inwhich the fossil was found.

The scientist measures the ratio of parent to daughter elements in this igneous rock.The scientist concludes the age of the igneous rock, as previously explained.The scientist concludes the age of the fossil from the age of the igneous rock.Link to movie: Fossil dating 6'14�

Available for free at Connexions <http://cnx.org/content/col11410/1.3>

259

<iframe width="425" height="349" src="http://www.youtube.com/embed/K5GWSbBkjuk?hl=en&fs=1"frameborder="0" allowfullscreen></iframe>

Caption: This is a 6'14� video in which radiometric dating is explained.

4.2.5.5 Fossil tourism

South Africa has a number of fossil-rich areas which attract tourists. The Cradle of Mankind (Maropeng,Sterkfontein, Kromdraai and surrounding areas) and Langebaan are examples of places in South Africa whichattract fossil tourism.

Maropeng is a Tswana word for the origin of mankind.

4.2.5.5.1 Living fossils

Many creatures which disappeared from the fossil record millions of geological years ago have been found tobe alive still and almost identical to their fossilized counterparts.

There are many examples of living fossils. The Tuatara, which looks like a lizard with a beak-shapedhead, disappears from the record 135 million geological years ago, but is still alive. Lingula, a worm-likecreature which disappears from the record about 400 million years ago, not very far from the beginning ofmetazoan evolution, is still alive and shows no sign of signi�cant evolution from its fossilized ancestors. Thissuggests that we may be interpreting the fossil record incorrectly, and our conclusions based on it may bewrong.

4.2.5.5.1.1 Activity: Fossils

1.Name / give the term for:

• Turning to stone• A fossil in which the surroundings became stone, leaving a cavity where the body had been• The time period and era, according to the geological column, when dinosaurs �rst appeared• The time period and era, according to the geological column, when dinosaurs became extinct• A person who studies fossils• A person who studies rocks• Layers of deposited material• No members of that organism exist anymore• The time era, according to the geological column, when mankind evolved

2.For each of the following sentences, say whether the information is: (Circle i / ii / iii)

• Accepted by de�nition if people have de�ned this to be so, or• Known by observation if people have observed this to be so, or• Deduced if people have come to this knowledge by reasoning.

• During petrifaction, minerals enter, harden, and turn a body to stone.[ i / ii / iii]• The Triassic Period is more recent than the Permian Period. [ i / ii / iii]• The oldest fossils are 650 million years old. [ i / ii / iii]• A �lm fossil is a carbon mark left on rock e.g. by leaves. [ i / ii / iii]• The deepest rocks with reptile fossils are classed as Carboniforous. [ i / ii / iii]• Reptiles �rst evolved 350 million years ago, at the start of the Carboniforous period. [ i / ii / iii]• Only 2/3 of all types of organisms which have ever lived have left any fossils.[ i / ii / iii]

3.The modern coelacanth-�nding throws doubt on an assumption palaeontologists use when reasoningabout what the fossil record says about extinctions.

Available for free at Connexions <http://cnx.org/content/col11410/1.3>

260 CHAPTER 4. DIVERSITY, CHANGE AND CONTINUITY

• What is the assumption the modern coelacanth-�nding throws doubt on?• How does the modern coelacanth-�nding throw doubt on this assumption?

4.Look at these premises:

• A cadborosaurus was found in the stomach of a whale in 1993 .• A pterodactyl was killed at Culment, France, in 1856 .• Blood vessels, red blood cells, osteocytes and proteins have been found in unfossilized dinosaur bones .• Undamaged DNA pieces have been found in insects fossilised in amber (a sticky plant resin).• Unfossilised bones, blood vessels, blood cells and DNA break down quickly .• The extinction of the dinosaurs is assumed at more than 60 million years ago.

Give a conclusion which follows from these premises5.A rock is found to have a K:Ar ratio of 1:10. Its age is calculated as 4,37 x109 years.

a. K (potassium) is the parent element. Ar (argon) is the _________________ element.b. What data is used to do this calculation?c. What conclusion/inference is made from this data?d. Name two assumptions used when making this conclusion from this data:e. Give a counterargument, showing how this same data could �t with a young age.

Activity: Fossils: Suggested Answers1.Name / give the term for:

a. Turning to stone Petrifactionb. A fossil in which the surroundings became stone, leaving a cavity where the body had been Mouldc. The time period and era, according to the geological column, when dinosaurs �rst appeared Triassic

period; Mesozoic erad. The time period and era, according to the geological column, when dinosaurs became extinct Cretaceous

period; Mesozoic erae. A person who studies fossils Palaeontologistf. A person who studies rocks Geologistg. Layers of deposited material Sediments / Stratah. No members of that organism exist anymore Extincti. The time era, according to the geological column, when mankind evolved Cenozoic

2.For each of the following sentences, say whether the information is: (Circle i / ii / iii)

i. Accepted by de�nition if people have de�ned this to be so, orii. Known by observation if people have observed this to be so, oriii. Deduced if people have come to this knowledge by reasoning.

a. During petrifaction, minerals enter, harden, and turn a body to stone.[ i / ii / iii]Observed and De�nedb. The Triassic Period is more recent than the Permian Period. [ i / ii / iii]c. De�nedd. The oldest fossils are 650 million years old. [ i / ii / iii]Deducede. A �lm fossil is a carbon mark left on rock e.g. by leaves. [ i / ii / iii]De�nedf. The deepest rocks with reptile fossils are classed as Carboniforous. [ i / ii / iii]De�nedg. Reptiles �rst evolved 350 million years ago, at the start of the Carboniforous period. [ i / ii / iii]Deducedh. Only 2/3 of all types of organisms which have ever lived have left any fossils.

i / ii / iii Deduced

3.The modern coelacanth-�nding throws doubt on an assumption palaeontologists use when reasoning aboutwhat the fossil record says about extinctions.

Available for free at Connexions <http://cnx.org/content/col11410/1.3>

261

a. What is the assumption the modern coelacanth-�nding throws doubt on? An extinction event can berecognised by fossils of that type not being found in higher (i.e. younger) rock.

b. How does the modern coelacanth-�nding throw doubt on this assumption? Although no coelacanthfossils are found in rock younger than that assumed to be 100 million years old, coelacanths are notextinct: they are alive today (extant).

4.Look at these premises:

• A cadborosaurus was found in the stomach of a whale in 1993.• A pterodactyl was killed at Culment, France, in 1856.• Blood vessels, red blood cells, osteocytes and proteins have been found in unfossilized dinosaur bones.• Undamaged DNA pieces have been found in insects fossilised in amber (a sticky plant resin).• Unfossilised bones, blood vessels, blood cells and DNA are known to break down quickly.• The extinction of the dinosaurs is assumed at more than 60 million years ago.

Give a conclusion which follows from these premises: Either the assumption that dinosaurs became extinct60 million years ago is faulty or cadborosauruses and pterodactyls are not dinosaurs.

5.A rock is found to have a K:Ar ratio of 1:10. Its age is calculated as 4,37 x109 years.a) K (potassium) is the parent element. Ar (argon) is the daughter element.b) What data is used to do this calculation? The K:Ar ratio .c) What conclusion/inference is made from this data? The rock is 4,37 x109 years old .d) Name two assumptions used when making this conclusion from this data:All the Ar in the rock came from radioactive decay of the K. This radioactive decay happened at the samerate as it had in the past.e) Give a counterargument, showing how this same data could �t with a young age.The rock might have started with a lot of Ar in it. The conditions in the past might have caused K to decayinto Ar at a higher rate than today. Further, Argon is known to move easily through rocks, so the Ar mightnot all have come from radioactivity.

4.2.6 4.2.6 Key Events in Life's History37

4.2.6.1 Key Events in Life's History

South Africa is home to many di�erent kinds of fossils, including living fossils. Many of these fossils are keyevents in life's history.

37This content is available online at <http://cnx.org/content/m43229/1.1/>.

Available for free at Connexions <http://cnx.org/content/col11410/1.3>

262 CHAPTER 4. DIVERSITY, CHANGE AND CONTINUITY

4.2.6.1.1 Fossilised Bacteria In Baberton Area

Figure 4.28: the microbes broke down volcanic glass to extract nutrients

Figure 4.29: the rocks were altered after they were formed but the burrows were preserved

Available for free at Connexions <http://cnx.org/content/col11410/1.3>

263

Figure 4.30: Stromatolite formation near Barberton area

The earliest signs of life are about 3500 million years old. Some of the most ancient fossils that are knownto exist have been found in the rocks from the Barberton area of Mpumalanga. They are more than 3000million years old. These tiny fossils look like modern blue-green bacteria. Later other types of blue- greenalgae grew that formed a mat in shallow water. These are called stromatolites and can be found in theancient rocks near Barberton.

Evolution of the cell:http://learn.genetics.utah.edu/content/begin/cells/organelles/ 38

4.2.6.1.1.1 Interesting Information

Conditions on Earth 4 billion years ago were very di�erent than they are today. The atmosphere lackedoxygen, and an ozone layer did not yet protect Earth from harmful radiation. Heavy rains, lightning andvolcanic activity were common. Yet the earliest cells originated in this extreme environment. Today, a groupof single-celled organisms called archaeabacteria, or archaea, still thrive in extreme habitats.

Astrobiologists are now using archaea to study the origins of life on Earth and other planets. Becausearchaea inhabit places previously considered incompatible with life, they may provide clues that will improveour ability to detect extraterrestrial life. Interestingly, current research suggests archaea may be capable ofspace travel by meteorite. Such an event could have seeded life on Earth or elsewhere.

Figure 4.31: Lake that supports Archaea bacteria

38http://learn.genetics.utah.edu/content/begin/cells/organelles/

Available for free at Connexions <http://cnx.org/content/col11410/1.3>

264 CHAPTER 4. DIVERSITY, CHANGE AND CONTINUITY

4.2.6.1.2 Soft Bodied Animals In Namibia And The Northern Cape

Figure 4.32

Figure 4.33

These soft-bodied animal-like creatures were unrelated to anything we see today and died out about 450million years ago. Fossils of these ancient forms of life have been found in Namibia and the Northern Cape,and also in Europe and North America. For animals without skeletons, like worms or jelly�sh, fossilizationis a very rare event. Paleontologists seldom �nd a well- preserved fossil of a soft-bodied animal. For asoft-bodied animal to be fossilized, its body must be protected from decomposition. The body is usuallyexposed to air and water with a lot of oxygen, so it decomposes quickly. The animal is likely to be fossilizedonly if it is buried soon after it dies (or when it is buried alive!).

Available for free at Connexions <http://cnx.org/content/col11410/1.3>

265

Soft bodied fossils in Namibia: (notes) http://fairley.ca/sclips_�les/Eblossom.htm 39

Soft bodied animals and fossil formation: http://www.k5geosource.org/1content/1sc/fossils/pg6.html 40

4.2.6.1.3 Early Land Plants In Grahamstown Area

Figure 4.34

Figure 4.35

39http://fairley.ca/sclips_�les/Eblossom.htm40http://www.k5geosource.org/1content/1sc/fossils/pg6.html

Available for free at Connexions <http://cnx.org/content/col11410/1.3>

266 CHAPTER 4. DIVERSITY, CHANGE AND CONTINUITY

Figure 4.36

Figure 4.37

By the end of the Devonian period of the Palaeozoic era there were more club mosses, ferns and horsetails,which are all plants that had primitive conducting tissue. Some of their relatives are alive today. There arewell preserved fossils of these club mosses and the simple relatives of conifers like cycads in the late Devonianrocks near Grahamstown in the Eastern Cape.

Cycads � early land plants near Grahamstown:http://www.arkive.org/eastern-cape-blue-cycad/encephalartos-horridus/ 41

41http://www.arkive.org/eastern-cape-blue-cycad/encephalartos-horridus/

Available for free at Connexions <http://cnx.org/content/col11410/1.3>

267

4.2.6.1.4 Forests Of Primitive Plants Such As Glossopteris Near Mooi River And Estcourt

Figure 4.38: Distribution of Glossopteris in Pangaea

Available for free at Connexions <http://cnx.org/content/col11410/1.3>

268 CHAPTER 4. DIVERSITY, CHANGE AND CONTINUITY

Figure 4.39: Glossopteris

Near the end of the Palaeozoic Era, in the Permian period, Southern Africa had moved away from the SouthPole. When the climate became warmer, the land became covered in new types of cone-bearing plants.The best known plant in the Southern hemisphere from that time is a tree called Glossopteris. Fossils ofGlossopteris tree trunks are common near Estcourt and Mooi River in Kwa-Zulu Natal. Plants like theseformed the huge coal deposits that are mined in South Africa today. The �rst true seed-bearing plants,which were simple conifers, appeared at the very end of the Permian Period.

Glossopteris and coal deposits:http://www.scienceinafrica.co.za/2006/february/coal.htm 42

Map of coal deposits in SA:http://www.mapsofworld.com/business/industries/coal-energy/south-africa-coal- deposits.html 43

42http://www.scienceinafrica.co.za/2006/february/coal.htm43http://www.mapsofworld.com/business/industries/coal-energy/south-africa-coal-deposits.html

Available for free at Connexions <http://cnx.org/content/col11410/1.3>

269

4.2.6.1.5 The Coelacanth As A Living Fossil Of The Group That Is Ancestral To Amphibians

Figure 4.40: The coelacanth is a �living fossil�; it is similar to 350 million year old fossils but is stillalive today.http://www.�ickr.com/photos/26573031@N05/2903081204/

The coelacanth is a lobe-�nned �sh from the Permian Period that was once thought to be extinct and wasonly known as a fossil. In 1938, a South African �shing boat netted a large �sh with deep blue scales and�ns on short �legs�. Professor J.L.B. Smith recognised it as a fossil �sh from rocks up to 350 million yearsold. It was thought to have been extinct for at least 65 million years. A second coelacanth was found in1952 in the Comores Islands. Many have been found since then. Another population has also been foundnear Sodwana Bay on the Kwazulu-Natal north coast.

See article athttp://animals.nationalgeographic.com/animals/�sh/coelacanth.html 44

4.2.6.1.6 Mammal-Like Reptiles In The Karoo

Figure 4.41: Therapsid fossil in the Karoo (�ickr)

44http://animals.nationalgeographic.com/animals/�sh/coelacanth.html

Available for free at Connexions <http://cnx.org/content/col11410/1.3>

270 CHAPTER 4. DIVERSITY, CHANGE AND CONTINUITY

Figure 4.42: Lystrosaurus (Wikipedia)

Reptiles diversi�ed and spread as the most successful and largest land vertebrates in the Permian Period.One important group was reptiles with some mammal features that are called Therapsids. Thrinaxodon andLystrosaurus were small therapsids that grew to about 50cm long. Fossils of these mammal-like reptiles havebeen found in the Karoo. These animals are seen as transitional between reptiles and mammals. They haveseveral developmental advances such as extra ear bones and a bony palate, which are not normally found inreptiles.

4.2.6.1.7 Dinosaurs

Figure 4.43: Euskelosaurus browni

The special type of reptile called a dinosaur appeared and spread during the Triassic Period of the MesozoicEra. A South African example of a dinosaur is Euskelosaurus browni. It was one of the �rst dinosaursdiscovered in Africa. It lived in the area now called the Eastern Cape and fossils have been found nearLadybrand.

One of the best places in the whole world to look for dinosaur fossils is the sedimentary rocks of theDrakensberg Mountains and Maluti Mountains of southern Africa.

Fossils of ferns and cone-bearing plants have also been found in the same areas as the dinosaurs.

Available for free at Connexions <http://cnx.org/content/col11410/1.3>

271

4.2.6.1.8 First Mammals

Figure 4.44: Megazostrodon � �rst mammal during the Jurassic period. Mammalscould not dominate during the reign of the dinosaurs, as the dinosaurs were huge.(Flickr)http://www.�ickr.com/photos/nordelch/2052121799/ 45

The �rst true mammals appeared during the Jurassic Period of the Mesozoic Era. They were small, aboutthe size of a mouse and lived side by side with the dinosaurs. South Africa is the only place in the worldwhere there are fossils that show changes from the earliest Therapsid (mammal-like) reptiles from the Karoorocks of the Permian Period to the �rst true mammals from the Drakensberg rocks in the Eastern Cape andLesotho.

Early mammal evolution: Interactive sitehttp://www.mnh.si.edu/mammals/pages/how/index.htm 46

4.2.6.1.9 Prehumans

Discovery of early Hominids in SA:http://anthro.palomar.edu/hominid/australo_1.htm 47

The Hominids are the �human-like� organisms that diversi�ed from the apes. The apes lived in tropicalforests, while the hominids lived in woodlands and savannah. About 4mya, several di�erent species ofHominids with large brains appeared in Africa. These prehumans are in the genus Australopithecus. Themodern human Homo sapiens appeared in Africa about 200 000 years ago and spread all around the Earth.Many people believe that the eastern side of Africa and South Africa are the original home of humans.The Cradle of Humankind at Sterkfontein and Kromdraai has yielded some of the most exciting and oldesthominid fossils in the world.

45http://www.�ickr.com/photos/nordelch/2052121799/46http://www.mnh.si.edu/mammals/pages/how/index.htm47http://anthro.palomar.edu/hominid/australo_1.htm

Available for free at Connexions <http://cnx.org/content/col11410/1.3>

272 CHAPTER 4. DIVERSITY, CHANGE AND CONTINUITY

Figure 4.45

Figure 4.46: Ron Clarke with Little Foot and Mrs Ples. Both were discovered at Sterkfontein.

4.2.6.1.10 Activity: Map Key Events In Life's History In Southern Africa

The text on the following pages describes some of the key events in Life's History for which there is evidencein Southern Africa.

Available for free at Connexions <http://cnx.org/content/col11410/1.3>

273

• When you read each description in the text that follows, identify the places where evidence has beenfound and mark them on the map.

• For each place you have marked, [provide a brief description of the event that happened]

Figure 4.47

Available for free at Connexions <http://cnx.org/content/col11410/1.3>

274 CHAPTER 4. DIVERSITY, CHANGE AND CONTINUITY

4.2.7 4.2.7 Summary48

4.2.7.1 Summary

a. Life's History:

a. The extremely long time periods in the earth's history are divided into eons, eras and periods foreasier study.

b. Continental drift occurred by means of plate tectonic movements. The supercontinent Pangaeasplit into Laurasia in the north and Gondwana in the south, ultimately forming today's continents.

b. The Geological Time Scale:

• Before the Paleozoic era, life consisted of bacterial mounds called stromatolites, which oxygenated theatmosphere. The Cambrian explosion was a sudden increase in complex multicellular life early n thePaleozoic era. Organisms underwent further changes later.

• The Paleozoic era ended with a massive extinction, killing many species. The Mesozoic era started,when the dominant life forms were conifers and dinosaurs.

• 65 mya, the Mesozoic ended and the Caenozoic started, with mammals and �owering plants becomingdominant. Human evolution also occurred in the latter stages of this era.

a. Missing Links:

• Archaeopteryx is the link between dinosaurs and birds � it has teeth, feathers, a bony long tail andclaws on its forelimbs.

• The coelacanth is the link between amphibians and �sh. It has lobed �ns, primitive internal organsand nostrils that go right through into the mouth.

a. Fossils:

• Fossils are formed in sedimentary rocks only over long periods.• They can be casts, moulds, mineralized body parts like bones / shells, imprints, �lm fossils or even

dung samples.• Fossils are dated using carbon 14 dating (fairly recent organic samples only) or radiometric dating for

older fossils.• Fossil sites bring in many tourists and can be important for the economy.• Some ancient organisms are seen as �living fossils� � they were believed extinct, but are still present,

e.g. the coelacanth and tuatara.

a. Key events in the history of life:

• Stromatolite fossils were found in Barberton and fossils of soft-bodied animals in Namibia.• Ancient plant fossils like Glossopteris are present in central KZN and near Grahamstown.• The coelacanth was `rediscovered' by the East London museum in 1938.• Mammal-like reptile fossils were found in the Karoo � they have advanced palates and complex ear

bones, unlike true reptiles. They are called therapsids.• Dinosaur fossils can be seen in the Drakensberg and Maluti mountains. This are also has fossils of the

earliest small mammals.• Pre-human fossils like Australopithecus have been found in the Sterkfontein area and nearby places.

This is unique � nowhere else in the world do these fossils occur. The majority of these �nds weremade by paleontologists from Wits University.

48This content is available online at <http://cnx.org/content/m43085/1.1/>.

Available for free at Connexions <http://cnx.org/content/col11410/1.3>

275

4.2.8 4.2.8 - exercises49

4.2.8.1 Exercises

4.2.8.1.1 Question 1

1. In each of the following cases, write down the LETTER of the most correct alternative:

a. Which of the following fossils have been found in Namibia ?

1. Mammal-like reptiles2. Glossopteris leaves3. Soft-bodied animals4. Early mammals

b. What was the importance of stromatolites ?

1. They released oxygen into the air.2. They absorbed oxygen and prevented animals being poisoned.3. They are the closest ancestors of modern plants.4. They were algae trapped in sediment.

c. A problem in determining the accuracy of radiometric dating is that . . .

1. scientists are not sure that radioactive decay actually occurs.2. the decay rate of the minerals can change without warning.3. the rocks that contain the fossils can't be dated.4. only organic samples can be dated radiometrically.

d. Continental drift accounts for the fact that people have found . . .

1. fern-like fossils in Africa.2. the fossils of subtropical plants in Antarctica.3. tectonic plates under the continents.4. trilobite fossils in almost all continents.

e. Which statement below DOES NOT apply to the Paleozoic era?

1. Trilobites and algae were the dominant life forms.2. Coal swamps were formed in many areas.3. It started with the Cambrian explosion.4. Birds developed during this era. [10]

4.2.8.1.2 Question 2

Match the terms in the �rst column with the phrases / descriptions in the second column. Write down onlythe correct LETTER next to the question number: [5]

Questions Column 1 Column 2

2.1 Mesozoic era A ancestor of frogs and �sh

2.2 Coelacanth B are ancestral insect forms

2.3 Therapsids C are always solid fossils

2.4 Cast fossils D very super�cial fossil imprints

2.5 Trace/ �lm fossils E early mammals developed

F are always hollow fossils

G ancestral to birds and dinosaurs

H have advanced palates and ear bones

49This content is available online at <http://cnx.org/content/m43184/1.1/>.

Available for free at Connexions <http://cnx.org/content/col11410/1.3>

276 CHAPTER 4. DIVERSITY, CHANGE AND CONTINUITY

Table 4.5

4.2.8.1.3 Question 3

Give the correct biological TERM for each of the following. Write down only the question number and theanswer.

1. The era during which ferns and cycads were the dominant plants2. The ancient supercontinent3. A scientists who studies fossils4. The type of rock that may contain fossils5. An ancient �sh type found in very deep water at Sodwana Bay [5]

4.2.8.1.4 Question 4

Examine the two images below:

Figure 4.48

Available for free at Connexions <http://cnx.org/content/col11410/1.3>

277

Figure 4.49

1. Suggest THREE things that fossil footprints like those in picture A can tell scientists about the animalthat made them.(3)

2. Identify the fossil in picture B. (1)3. Explain brie�y how this fossil was formed.(8)4. What can we conclude about the area where this fossil is located millions of years ago? Give a reason

for your answer.(2)5. Why is it that fossils are not found in igneous rock?(2) [16]

4.2.8.1.5 Question 5

Examine the fossil and graph below and answer the questions:

Available for free at Connexions <http://cnx.org/content/col11410/1.3>

278 CHAPTER 4. DIVERSITY, CHANGE AND CONTINUITY

Figure 4.50

Figure 4.51

1. To which class of the Chordata does the fossil shown here belong? Choose from: Pisces, Amphibia,Reptilia, Aves or Mammalia.(2)

2. What does the term the �half life� of a radioactive element mean? (2)3. When the fossil was subjected to radiometric dating, it was found to contain 90% Uranium. If the half

life of Uranium 235 is 704 million years, how old is this fossil? Show all your working.(4)4. If another fossil was found and dated to 1408 million years, how much Uranium did it contain? Explain

how you obtained your answer. (5) [13]

Available for free at Connexions <http://cnx.org/content/col11410/1.3>

279

4.2.8.1.6 Question 6

Read the information below and answer the questions: Therapsid fossils from the late Permian have beenfound in South Africa. These animals had teeth di�erentiated into incisors, canines and molars, unlikereptilian teeth, which are all the same. Their legs were held more vertically, i.e. under the body, comparedto what is usual in reptiles. It seems that they survived into the Mesozoic, but died out when the dinosaursbecame to dominant life form.

1. Where in South Africa have therapsid fossils been found?(1)2. Tabulate TWO ways n which therapsids are di�erent from reptiles. (5) [6]

TOTAL 55

4.2.8.1.7 Suggested answers

4.2.8.1.7.1 Question 1

1. C2. A3. C4. B5. D[10]

4.2.8.1.7.2 Question 2

1. E2. A3. H4. C5. D [5]

4.2.8.1.7.3 Question 3

1. Mesozoic2. Pangaea3. Paleontologist4. Sedimentary5. Coelacanth[5]

4.2.8.1.7.4 Question 4

1. Mark 1st 3 only: Animal's species, approximate length of leg, stride length approximate body weight,speed it moved at(3)

2. part 2

a. Ammoniteb. Ammonite died, sank into mud. Mud was covered by more sand and excluded oxygen. Body

decayed, shell was slowly mineralized over time. Mud was compressed and became harder, turningto stone. Later uncovered by erosion. (max 8)

c. It was under the sea. Ammonites were marine creatures(2)

3. Igneous rock is volcanic - lava is too hot to preserve fossils OR heat from lava destroys remains ofanimals(2) [ 16]

Available for free at Connexions <http://cnx.org/content/col11410/1.3>

280 CHAPTER 4. DIVERSITY, CHANGE AND CONTINUITY

4.2.8.1.7.5 Question 5

1. Amphibia(2)2. The amount of time needed to reduce the amount of (parent) element to half of what it was (OR the

time needed for half of the element to decay / break down)(2)3. 0,2 x 704 million = 140,8 million years(4)4. 1408 million years ÷704 million years = 2 half lives, so the fossil had 25% Uranium 235.(5) [ 13]

4.2.8.1.7.6 Question 6

1. Karoo(1)2. Table comparing therapsids and reptiles : (5) [6]

Therapsid Reptile

Specialized teeth (or explain) Legs held under body All teeth look the sameLegs out to the sides

Table 4.6

TOTAL 55

Available for free at Connexions <http://cnx.org/content/col11410/1.3>

281

Solutions to Exercises in Chapter 4

Solution to Exercise 4.2.2.1 (p. 248)COMPARING THE SKELETONS OF DINOSAURS, Archaeopteryx & BIRDS:

SIMILARITIES:

1 Jaws have teeth

2 Hand / arm has claws

3 Long bony tail present

4 Presence of gastralia or dermal ribs (not attached to spine)

Archaeopteryx vs Dinosaur:

DIFFERENCES:

1 Long forelimbs, like wings Short forelimbs

2 Feathers present No feathers

3 Hand has three claws Hand has �ve claws

4 Furcula / wish bone present No furcula present

Archaeopteryx vs Modern bird

SIMILARITIES:

1 Feathers are present

2 Forelimbs are long and wing-like

3 Furcula / wish bone present (fused clavicles)

4 Bones of the lower forelimb are separate

Archaeopteryx vs Modern bird

DIFFERENCES:

1 Teeth in jaws No teeth in the beak

2 Claws on forelimbs Forelimbs without claws

3 Long bony tail Short tail bones / pygostral present

4 No breast bone Breast bone with a keel

SIMILARITIES:

1 Jaws have teeth

Available for free at Connexions <http://cnx.org/content/col11410/1.3>

282 CHAPTER 4. DIVERSITY, CHANGE AND CONTINUITY

2 Hand / arm has claws

3 Long bony tail present

4 Presence of gastralia or dermal ribs (not attached to spine)

Archaeopteryx vs Dinosaur:

DIFFERENCES:

1 Long forelimbs, like wings Short forelimbs

2 Feathers present No feathers

3 Hand has three claws Hand has �ve claws

4 Furcula / wish bone present No furcula present

Archaeopteryx vs Modern bird

SIMILARITIES:

1 Feathers are present

2 Forelimbs are long and wing-like

3 Furcula / wish bone present (fused clavicles)

4 Bones of the lower forelimb are separate

Archaeopteryx vs Modern bird

DIFFERENCES:

1 Teeth in jaws No teeth in the beak

2 Claws on forelimbs Forelimbs without claws

3 Long bony tail Short tail bones / pygostral present

4 No breast bone Breast bone with a keel

SIMILARITIES:

1 Jaws have teeth

2 Hand / arm has claws

3 Long bony tail present

4 Presence of gastralia or dermal ribs (not attached to spine)

Archaeopteryx vs Dinosaur:

DIFFERENCES:

Available for free at Connexions <http://cnx.org/content/col11410/1.3>

283

1 Long forelimbs, like wings Short forelimbs

2 Feathers present No feathers

3 Hand has three claws Hand has �ve claws

4 Furcula / wish bone present No furcula present

Archaeopteryx vs Modern bird

SIMILARITIES:

1 Feathers are present

2 Forelimbs are long and wing-like

3 Furcula / wish bone present (fused clavicles)

4 Bones of the lower forelimb are separate

Archaeopteryx vs Modern bird

DIFFERENCES:

1 Teeth in jaws No teeth in the beak

2 Claws on forelimbs Forelimbs without claws

3 Long bony tail Short tail bones / pygostral present

4 No breast bone Breast bone with a keel

Available for free at Connexions <http://cnx.org/content/col11410/1.3>

284 CHAPTER 4. DIVERSITY, CHANGE AND CONTINUITY

Archaeopteryx vs Dinosaur: SIMILARITIES:

1 Jaws have teeth

2 Hand / arm has claws

3 Long bony tail present

4 Presence of gastralia or dermal ribs (not attached to spine)

Archaeopteryx vs Dinosaur: DIFFERENCES:

1 Long forelimbs, like wings Short forelimbs

2 Feathers present No feathers

3 Hand has three claws Hand has �ve claws

4 Furcula / wish bone present No furcula present

Archaeopteryx vs Modern bird SIMILARITIES:

1 Feathers are present

2 Forelimbs are long and wing-like

3 Furcula / wish bone present (fused clavicles)

4 Bones of the lower forelimb are separate

Archaeopteryx vs Modern bird DIFFERENCES:

1 Teeth in jaws No teeth in the beak

2 Claws on forelimbs Forelimbs without claws

3 Long bony tail Short tail bones / pygostral present

4 No breast bone Breast bone with a keel

Table 4.7

Solution to Exercise 4.2.2.2 (p. 250)

a. Datab. Conclusionsc. Conclusions

Available for free at Connexions <http://cnx.org/content/col11410/1.3>

Chapter 5

Glossary1

Abiotic Describes the non-living elements of the ecosystemAbort To purposefully end a pregnancy before full termAbsorb To take something in. For example if a cell takes in water it absorbs water.Absorption (of nutrition) Taken into the cells of the bodyAccessory Something extra which is added to make the thing completeAccommodation Altering the shape of the lens of the eye to focus on close objectsAcetylcholine A chemical that carries a nerve impulse from one neuron to the next across the synapseAcid A chemical substance that releases hydrogen ions (H+) in water and has a pH less than 7Acquired A feature which occurs because of what has happened during the life of theorganism eg. an acquired characteristic could be a scar on the skinAcromegaly Larger hands, feet and bones caused by too much growth hormone as an adultActivation energy The energy needed for a reaction to take placeActive site Place on an enzyme molecule where a reaction happens; it �ts the substrateexactlyActive transport Movement of substances using respiration energy, needed for movement against a

concentration gradientAdaptation A change to suit the conditionsADH AntiDiuretic Hormone from the posterior pituitary; makes the collecting duct of the kidney more

permeable to water, reducing the volume of urineAdhesion When a substance sticks on to a di�erent kind of substance (compare Cohesion).ADP Adenosine Diphosphate, a chemical that can take up an extra phosphate when supplied with

chemical energyAdrenal gland An endocrine gland found near the kidney with an outer cortex (that makescortisone and aldosterone) and inner medulla (that makes adrenalin)Adrenalin The hormone from the adrenal medulla that prepares the body for stressAerobic Uses oxygenA�erent Carries something to a structure (compare this with "e�erent")Aldosterone A hormone that regulates water retention in the kidneys by controlling thedistribution of sodium ionsAlien Foreign to that place: in life sciences a species introduced by peopleAlimentary canal The tube that leads from the mouth in which food is broken down and absorbed

for the bodyAlkali An hydroxide chemical which dissolves in water to give hydroxyl ions (OH - ) and a pH greater

than 7Allergen Something that causes an allergic reaction in the body

1This content is available online at <http://cnx.org/content/m43254/1.1/>.

Available for free at Connexions <http://cnx.org/content/col11410/1.3>

285

286 CHAPTER 5. GLOSSARY

Allergy A more than normal sensitivity to a common substanceAlveolus Small, thin walled bag in the lungs containing air; it is where gaseous exchange happensAmino acid An organic molecule with an acid (-COOH) group and an amino (-NH 2 ) group. It is

the building block of proteinsAmino group The -NH 2 arrangement of atoms which is found in all amino acids and therefore all

proteinsAmniocentesis Procedure to sample the cells of a foetus by removing some �uid surrounding it in the

wombAmnion The innermost extra-embryonic membrane; it makes a �uid �lled sacAmniotic �uid The protective liquid around the embryo and foetus; it is enclosed by the amnionAmplify To increase the strength or loudnessAmpulla A bulge at the end of each semicircular canal where acceleration receptors are foundAnaemia A de�ciency disease caused by lack of iron which reduces the number of red blood cells,

a�ecting the �ow of oxygen to the tissuesAnaerobic Does not use oxygenAnalagous Doing the same job as the other structure; compare with `homologous'Androgen Male sex hormone produced by the testes; they develop and maintain the male sexual

characteristics eg. testosteroneAneuploidy A mutation in which the number of chromosomes changes eg.47 chromosomes in Down's

syndromeAnorexia A disease in which a person refuses to eat and cannot see that s/he is becoming illAnterior To the front of the organismAnther The box on top of a stamen where pollen is madeAntibody A protein in the blood which �ghts disease; made in response to the presence of foreign

substancesAntibiotic A chemical substance made by a micro-organism that stops the growth of other micro-

organismsAntigen A substance which is foreign to the body and causes antibodies to be madeAntiseptic A chemical that kills germs such as bacteria and fungiAorta The largest artery in the body, leading from the left ventricle of the heart to all the body except

the lungsAqueous humour The watery material �lling the space in front of the lens of the eyeArachnoid The middle of the three membranes covering the brain and spinal cordArtery A blood vessel which carries blood away from the heart towards capillariesAssimilation (in nutrition) Becoming a part of the bodyAsthma Disease causing breathing di�culty; it is often caused by an allergenAtherosclerosis A deposit of fatty substance leading to raised blood pressure and heart diseaseAtom The smallest part of an element which has the properties of that elementATP Adenosine Triphosphate, a chemical that holds energy for the cell (see ADP)Atrium The chamber of the heart which collects blood from veins and passes it to the ventricleAutonomic Self- governing and so acting without conscious thought ie. involuntaryAxon A long extension of a nerve cell carrying an impulse away from the cell bodyBacteria A very small single celled organism of the kingdom ProkaryotaBar graph Graph using separate columns that is used when the independent variable is not numerical

eg types of foodBenedict's reagent A blue solution with Copper salts that gives a red precipitate when boiled with

a reducing sugar.Beriberi The de�ciency disease caused by lack of vitamin B1 (thiamine). Symptoms include paralysis

of the digestive system and poor co-ordinationBicarbonate indicator A substance that contains bicarbonate and changes colour with di�erent

acidity (red <pH7, yellow >pH7)

Available for free at Connexions <http://cnx.org/content/col11410/1.3>

287

Bicuspid The heart valve which lies between the left atrium and the left ventricleBile An alkaline juice made by the liver that emulsi�es fats and neutralises acidity when it enters the

duodenum through the bile ductBinary �ssion A form of asexual reproduction in single-celled organisms when they split into two

equal cellsBiochemistry The study of the working of the chemicals of the bodyBiodiversity The whole variety of living things found in that de�ned environmentBiotic Describes the living elements of the ecosystem compare with `abiotic'birth control Any method of having a sexual relationship without causing a pregnancyBiuret test The test for Protein. Add dilute NaOH then a few drops of dilute (1%)CopperSulphate. A purple colour indicates proteinBivalent (in life sciences) A structure made from two homologous chromosomes joined together during

meiosisBladder Muscular bag that stores the urine which is made continuously in the kidneyBlastocyst The hollow ball of up to 64 cells formed from a zygote in mammal reproductionBlind spot The part of the retina with no receptors since neurons leave the eye at that pointBlood plasma The matrix of blood tissue, the liquid part of the bloodBlubber A thick layer of fat under the skin to separate the warm body of some mammals from the

cold outsideB-lymphocyte A type of white blood cell, made in the bone marrow, found in a lymph nodeBolus A small ball of food which is swallowed and moved down the gullet by peristalsisBone marrow A soft tissue in the centre of some bones that makes blood cellsBowmans capsule Cup shaped structure in the kidney into which the capillaries of the glomerulus �tBrain The swollen front end of the spinal cord, with a large mass of interconnected neurons from where

the body is controlledBreathing The movement of air from the atmosphere into the lung air spaces and then out again;

otherwise called ventilationBreathing centre Region of the medulla oblongata which controls the depth and rate of breathingBronchiole Thin branch of a bronchus that carries air to the alveoli; has no cartilage ringsBronchitis Disease caused by an in�ammation of the bronchi of the lungBronchus One of the main air pathways, strengthened with cartilage rings, that carries air into, and

through a lungBudding A type of arti�cial asexual reproduction when a bud is grafted onto a scionBulimia A disease in which a person purposely brings up the food eaten, so losing weightCaecum Pouched piece of intestine where herbivores digest cellulose, in humans theappendix.Calyx The collective term for all the sepals of one �owerCanine tooth The large, pointed "fang" toothCapillarity Movement of a liquid up narrow tubes due to surface tensionCapillary The smallest blood vessel with a thin, leaky wall so that liquid can pass outCapsule The outer protective "skin" around a kidney to hold the tissues in placeCarbohydrate A compound of Carbon, Hydrogen and Oxygen with twice as many Hydrogenatoms as Oxygen atoms eg C6H12 O6Carbon �xation Holding the carbon of carbon dioxide gas in the leaf in a new form so it cannot

escape; it becomes attached to ribulose biphosphateCarbon monoxide A poisonous, colourless gas CO: it stops haemoglobin from being able to carry

oxygenCarboxyl The arrangement of atoms -COOH which is found in all organic acidsCarcinogen Any substance that causes a cancer to developCarpel One unit of the female reproductive parts of a �owerCardiac Of the heart

Available for free at Connexions <http://cnx.org/content/col11410/1.3>

288 CHAPTER 5. GLOSSARY

Cartilage Supporting tissue which is softer and more �exible than bone; cells calledchondrocytesCasparian strip A layer of waterproof suberin on the walls of endodermal cells in a plant root that

stops water moving through cell walls, directing it into the steleCatalyst Substance that speeds up a chemical reaction but does not get used up itselfCerebellum Swollen part of the hind brain that controls muscle movement and balanceCerebro-spinal Of the brain and spinal cord; cerebro-spinal �uid �lls the spaces in these organsCerebrum Swollen part of the front brain that controls voluntary actions, memory, reasoning etcChemical bond The link of one atom to another in a molecule; the most common bonds are ionic

bonds and covalent bondsChemical energy The ability to do work which is held inside a molecule; for example. in a chemical

substance like ATPChemotherapy A medical treatment that uses poisonous chemicals to kill cancerous cellsChlorophyll A green substance that has the ability to convert radiant energy into chemical energyChloroplast The green organelle containing chlorophyll found in some plant cellsChlorosis A disease of plants when the leaves are yellow instead of green. It is caused by lack of light,

or iron or magnesium de�ciencyCholesterol A waxy substance found in all cells, it can cause disease if there is too much in the bloodChorion The outer extra-embryonic membrane; it protects the foetus and helps form the placentaChoroid The middle of the three layers of the eye; it is black to reduce re�ection and carries blood

vessels to feed the tissuesChromatin The substance that chromosomes are made of; in interphase it is very thin threads of DNA

together with �ve special proteins called histonesChromosome One of a �xed number of thread-like structures in a cell nucleus; it carries genes and is

seen during nuclear divisionChyme The liquid mixture of food and juices that moves down the gut from the stomachCilia Tiny hair-like structures on the surface of some cells; they move to push things alongCiliary body The swollen region of the choroid of the eye near the lens that holds the ciliary musclesCiliary muscle Muscles that control the shape of the lens of the eye, they contract foraccommodationCiliated epithelium A thin internal skin whose cells are covered in ciliaCochlea A coiled tube that is a part of the inner ear and contains the receptors for the sense of hearingCodominance When there is the full expression of both of two alleles because neither of them is

dominant to the otherCoenzyme A substance that helps an enzyme to work properlyCohesion When a substance sticks to itself.Collecting duct A tube that collects urine from the nephrons and feeds it into the kidney pelvisColon The main part of the large intestine where water and some mineral salts areabsorbed and the faeces form; rich in bacteria producing vitamins B and KComet A lump of frozen dusty material that orbits around the sunCompound A substance that it made from two or more di�erent elementsConcentration The strength of a substance in a mixture. It is a measure of the number ofmolecules present.Condensation The joining of two molecules by taking a water molecule from them (see Hydrolysis)Cone A photoreceptor cell which is sensitive to bright light and colourConjunctiva The transparent membrane covering the eyeball and attached to the eyelidsConnective tissue Any animal tissue that provides support, packing and insulation between the

organs of the body; usually the cells are found in a large amount of matrixConstipation Disease in which it is di�cult to release faecesConstrict To become narrower

Available for free at Connexions <http://cnx.org/content/col11410/1.3>

289

Continental drift A theory explaining how the major land masses of earth have moved over time dueto plate tectonics

Control The part of an experiment where the manipulated variable is controlled or is missing so theexperimenter can check that the variable is causing the e�ect

Convex With a surface that curves outwards towards the edgeConvoluted Very highly folded and tangled upCo-ordination Causing separate parts to work together as one unitCornea The transparent part of the sclera at the front of the eyeCoronary artery One of the arteries that surround the heart and feed its musclesCorpus callosum Bundle of nerve �bres making the connection between the right and left cerebral

hemispheresCorolla The collective term for all the petals of one �owerCortex The outer region of an organ such as the kidneyCotyledon The part of the embryo of a plant used to provide food; �owering plants have either one

(monocots) or two (dicots) in each seedCounter current A system where two liquids �ow close to each other in opposite directionsCranial nerves Peripheral nerves that come from the brain part of the central nervous systemCretinism A condition of having a poorly developed brain and body due to lack of thyroxinCristae Folds on the inner membrane of a mitochondrionCross pollination The transfer of pollen from the male part of one �ower to the female part of another

�ower of the same speciesCupula Jelly-like structure in the ampulla which moves when the head is movedCusp The place where two curved surfaces meetCuticle The waxy, waterproof outer layer of a plant organ such as the shiny coating on a leafCytoplasm All the living substance of a cell except for the nucleus; it includes the otherorganelles.Cytosol The liquid part of the cell which is the cytoplasm without its membraneous organellesDark phase (stage) The part of the process of photosynthesis that is independent of light; when

carbon dioxide is �xed and then reduced to make carbohydrateData Known facts which can be used for analysis and interpretationDeamination Removing the amino group (-NH2) from an amino acid to leave C, H and O onlyDe�ciency When there is not enough of something to do the job properlyDenature When a protein loses its shape due to changes in acidity or to high temperatures; then it

can no longer carry out its jobDendrite A shorter extension of a nerve cell; bringing the impulse to the cell bodyDental formula A statement of the number of each type of teeth in one half of the upper and lower

jaw of a mammal speciesDeoxyribose A �ve carbon sugar with one atom of oxygen less in its molecule than a normal sugar;

one of the building blocks of each DNA nucleotideDestarch To take starch out; leave a plant in the dark for 24 hours to move starch out of its leavesDiabetes A disease when insulin does not function properly causing blood sugar to rise when carbo-

hydrates are eaten. In type 1 diabetes no insulin is made, in type 2 diabetes the insulin loses its e�ect onthe cells.

Diaphragm A sheet of tendon, with muscles on it, that separates the thorax from the abdomenDiastole The relaxed part of the heart beatDicotyledon (Dicot) One of the large group of plants that has two cotyledons in each seed, �ower

parts in �ves and net veined leavesDi�usion The spread of a substance caused by the natural random movement of its moleculesDigestion Breaking down complex foods into simple, soluble nutrientsDilate To get wider

Available for free at Connexions <http://cnx.org/content/col11410/1.3>

290 CHAPTER 5. GLOSSARY

Disaccharide A sugar with molecules of two sugar units joined; it can split into two parts which arestill both sugars (compare with monosaccharide)

Dissolve To make something into a solution (become mixed with a liquid)Distal Far (or distant) from the main part of the bodyDominant In genetics a characteristic that appears in the phenotype whenever it is present on at least

one homologous chromosome compare with `recessive'Dorsal On the upper side of the organismDouble fertilisation The special fertilisation process in which two male nuclei are needed; it is unique

to �owering plantsDuodenum The �rst part of the small intestine, it is about 30 cm long. The bile and pancreatic juice

run into it.Dura mater The strong outer membrane of the three that cover the brain and spinal cordDynamic Of movement; dynamic balance is the force of acceleration on the bodyEctothermic Getting body heat from the surroundings (compare with endothermic)E�ector A cell or organ (usually muscle or gland) that an animal responds withE�erent Carries something away from a structure; for example e�erent arterioles carry blood away

from Bowmans CapsuleEgestion Removal from the body of the waste products that could not be digestedElectron A tiny, negatively charged particle that is a part of all atomsElement A substance made from only one of the 92 di�erent sorts of atom that exist naturally; it

cannot be split into simpler substancesEmulsion A colloidal mixture of two liquids that do not dissolve, one of which is spread through the

otherEnamel The hard resistant substance on the outer surface of a toothEndocrine Of the system of ductless glands that release hormones directly into the bloodEndolymph The �uid that �lls the canals made of membrane found in the inner earEndoplasmic reticulum A complex system of membranes throughout the cytoplasm; they act as

channels for transporting materialsEndosmosis Osmosis of water into a cell; compare with Exosmosis.Endosperm The food supply of a seed that is not inside a cotyledonEndothermic Getting body heat from the working of the body; that is, the body heat is from inside

the bodyEnergy The ability to do workEnterokinase An enzyme of the duodenum that changes inactive trypsinogen into the active enzyme

trypsinEnvironment The surroundings of an organism or cellEnzyme A protein that activates and speeds up chemical reactions in an organismEnzyme activator Substance that must be added to an enzyme before it is able to workEpiglottis Flap of tissue which closes the trachea during swallowingEquator An imaginary line round the middle of a structure; the line of the middle of the cell during

cell divisionEquilibrium To be in balance and therefore apparently not changingErosion The loss of soill by wind or water actionErythrocyte Red blood cellEster link The special bond that joins the fatty acid parts of a fat molecule to the glycerol partEthanol Ethyl alcohol (C2H5OH), the alcohol found in drinks such as beerEustachian tube A tube leading from the middle ear to the throat; it allows the air pressure to be

equalised on each side of the eardrumEutrophication When too much nitrate and phosphate enters a body of water, leading to a shortage

of oxygen and the death of many speciesEvaporation Being lost from the surface as a vapour or a gas.

Available for free at Connexions <http://cnx.org/content/col11410/1.3>

291

Excretion The removal from the body of the waste products of metabolismExhale To breathe outExocrine Of the system of glands with ducts that produce enzymes, etc but not hormones; compare

with endocrineExosmosis Osmosis of water out of a cellFat An ester of glycerol with three fatty acids; it makes molecules that are important in cell membranes,

energy foods etcFatty acid An organic acid (R-COOH) with many units of Carbon and Hydrogen making up the

radical "R"Fermentation Anaerobic respiration in plants, fungi etc to produce alcoholFertilisation In life sciences i) when the male gamete and the female gametes fuse to make a zygote;

ii) providing mineral salts to improve soil fertilityFibrinogen A soluble blood protein that can change into insoluble �brin to form a clot when a blood

vessel is cut and the blood exposed to the airFilament In �owers, the stalk part of a stamen, it holds the anther up for pollinationFiltrate The substances which have passed through a �lter and not been held backFlaccid Soft and �oppy.Focus To adjust so that a sharp image is madeFood The materials taken in to the gut so that nutrients can be extractedFood chain The pathway of food in an ecosystem from the producers to primary consumers, then

secondary consumers, etcFormalin A poisonous chemical that it is very good at killing bacteriaFovea The part of the retina with most cones so the image made here is the clearestFructose A hexose sugar that is common in fruit. It is an isomer of glucose C6H12O6Fruit The ripened ovary of a �owering plant, it may be �eshy or dry and assists with seed dispersalFuel A substance that contains energy that can be releasedGall Bladder A bag-like container on the underside of the liver to store bile until the duodenum needs

itGamete A sex cell with a haploid set of chromosomes; large less mobile ones are called female, smaller

more mobile ones are called maleGaseous exchange Swopping oxygen from the air with the carbon dioxide made by the body; compare

this with breathing and respirationGastric Of the stomachGene A short length of a chromosome that contain a unit of genetic information; often each gene

carries the code for making one proteinGenetic code The way the information about an organism is stored in DNA molecules; each amino

acid is represented by a triplet of N basesGenetics The study of inheritanceGenome The full set of genes that an individual has; also the range of genes in a speciesGenotype The set of alleles for a characteristic that an organism has inheritedGenus A group of di�erent species with many features in commonGerminal epithelium A layer of tissue in the sex organs of animals that makes new cells by meiosisGland A part of the body which makes a useful substance for the bodyGlobal warming The recent rise in average temperature all over the world caused by extragreenhouse gases in the airGlobulin A blood protein with a folded shape so that it forms a ball shaped moleculeGlomerulus A knot of very narrow capillaries that sits in Bowman's capsule of the kidneyGlucagon A hormone from the alpha cells of the Islets of Langerhans in the pancreas; it causes an

increase in the amount of glucose in the bloodGlucose A common, simple hexose sugar that is made by photosynthesis and is the main respiratory

substrate. Its formula is C 6 H 12 O 6

Available for free at Connexions <http://cnx.org/content/col11410/1.3>

292 CHAPTER 5. GLOSSARY

Glycerol An alcohol with three hydroxyl units (CH 2 OH CHOH CH 2 OH) that is part of themolecule of all fats

Glycogen A polysaccharide used as the store of carbohydrates in animal cellsGlycolysis The �rst stage of respiration when 1 molecule of glucose changes into 2 pyruvic acid

moleculesGonad A sex organ that produces gametesGraa�an follicle A structure in the ovary that is �lled with �uid; the female gamete develops inside

it until ovulationGracile Delicately formed; compare with `robust'Granulocyte A white blood cell with a large irregular nucleus and granules in the cytoplasmGranum The structure holding chlorophyll in a chloroplast, made of a pile of thylakoids.Grey matter Nervous tissue that is mostly cell bodies with few myelinated �bres (see white matter)Growth hormone A hormone (from the anterior pituitary gland in animals) which promotes an

increase in sizeGuard cell One of the two curved cells that surround the pore (or hole) of a stoma, allowing it to

open and close.Guttation When water drops are squeezed out of hydathodes on a leaf onto its surfaceHaemoglobin The red coloured conjugated protein that can carry oxygen; it is found in erythrocytesHaemophilia The genetic bleeding disease, caused by a person not being able to make clotting factor

so wounds do not healHealth The state of being well, with body systems working e�cientlyHelper cell The kind of T-lymphocyte that helps B-lymphocytes to produce antibodiesHeat exchanger A system where heat is moved from one place to anotherHepatic Of the liverHerbicide A chemical substance that can kill green plantsHeredity The process in which characteristics are passed on from parents to their o�springHermaphrodite When one organism has both male and female sex organs eg earthworms and most

�owering plantsHeterotrophic Uses complex organic foods to get its nutrients (compare with autotrophic)Hexose A sugar which has 6 carbon atoms in its moleculeHistamine A chemical given out by mast cells during the allergic response; it causes dilation of blood

vesselsHistone A special protein that attaches to DNA molecules and is used to package the DNA; DNA plus

histone makes chromatinHistogram Graph using columns that is used when the independent variable data is grouped according

to classesHomeostasis Maintaining a constant internal environmentHomeothermic Having a constant body temperature which is maintained by homeostasisHominid The group of primates that use two legs for walking; only Homo sapiens still lives, the other

species are extinctHomologous Structures that look similar to each other or have the same originHormone A chemical messenger substance made in ductless glands and carried in the blood to act

somewhere elseHumid The air contains a lot of water vapour.Hydathode A gland on the surface of a leaf that forms water drops for guttationHydrogen bond A very weak chemical bond between the negatively charged part of one molecule and

the positively charged part of another moleculeHydrolysis When a large molecule is split into two smaller parts by adding a water molecule to it

(compare with Condensation)Hydrostatic The pressure caused by a liquid such as water when at restHypertonic Having a lower solute potential than the reference solution

Available for free at Connexions <http://cnx.org/content/col11410/1.3>

293

Hypophysis Another name for the pituitary gland in the middle of the head, below the brainHypothalamus The �oor of the thalamus (front brain) which links the nerve and hormone systems

for better body co-ordinationHypothesis Statement that is a possible explanation which can be tested by experiment to �nd out if

it is true or notHypotonic Having a higher solute potential than the reference solution (compare withHypertonic)Ileum The longest part of the small intestine. It is about 4 m long in humans. Mostabsorption happens in itImpermeable Substances are not able to pass through itImplantatio n The process in which a new embryo settles onto the wall of the uterusImpotent In males, unable to have sexual relations because the penis will not erectIncomplete dominance The blending of the expression of two alleles because neither of the pair is

dominant to the otherImpulse In life sciences, the message that passes along a nerveIncisor The front chisel shaped teethIndicator A substance which changes colour to show that another substance is present eg. litmus goes

red if acid presentIndigenous Native to the place and not introduced there by peopleInfectious A disease caused by a living organism that can pass from one person to anotherIn�ammation A redness or swelling of tissue in response to an injuryIngestion Taking in food at the mouthInhale To breathe inInhibit To slow down or reduce the e�ectInorganic Substance that is not organic, it usually does not have large molecules and usually does not

have carbon.Insoluble Does not dissolveInsulation A substance to prevent the movement of something (for example of heat)Insulin A hormone from the beta cells of the Islets of Langerhans in the pancreas; it causes a reduction

in the amount of glucose in the bloodIntercostal muscle Muscle between the ribs; the external intercostals raise the ribs to breathe inIntra-speci�c A process involving individuals of the same species; intraspedi�c competition is when

individuals of one species are trying to get the same resourcesInvasive Of an organism that easily spreads to other placesInvoluntary Not controlled by the will; it is automaticIodine solution A mixture of Iodine in Potassium Iodide solution. It causes starch to go blue blackIonised An atom or molecule that has become electrically charged because it has lost or gained electrons

so is no longer electrically neutralIris The circular coloured membrane in front of the lens of the eyeIsomers Substances which are di�erent, but have the same chemical formula eg glucose, fructose and

galactose are isomers with the formula C6H12 O6Isotonic Having the same solute potential as a reference solutionKaryotype A description of the number, size and shape of the chromosomes of a speciesKidney The organ for excretion and osmoregulation containing nephrons and blood vesselsKiller cell A type of T-lymphocyte that destroys cancerous or virus infected cellsKilojoule (kJ) A measure of the amount of energy, for example heat energy. It is one thousand joules.

1kJ will raise the temperature of 1 litre of water by 1 ◦CKrebs' cycle The second stage of aerobic respiration when pyruvic acid changes to CO2 and hydrogenKwashiorkor A de�ciency disease caused by lack of protein in the diet of young children;symptoms include slow growth, enlarged belly and reddened hairLabyrinth A complicated network of passages eg. the passages of the inner ear

Available for free at Connexions <http://cnx.org/content/col11410/1.3>

294 CHAPTER 5. GLOSSARY

Lacteal A tiny lymph vessel in the centre of a villus on the lining of the small intestineLactic acid An organic acid (CH3 CHOH COOH) which is made in muscles during anaerobic respi-

ration, causing an oxygen debtLarva The sexually immature form of an animal that hatches from an egg and does not resemble the

parentLarynx The voice box; the part of the air passage above the trachea and below the pharynxLatent heat Energy that is taken up by a substance when it changes from a liquid to a gasLens The curved transparent structure which can bend light to focus it at a pointLeucocyte Any white blood cellLight phase (stage) The part of the process of photosynthesis that needs light; during the light stage

water is split and ATP made, with oxygen released as a wasteLignin The chemical substance of wood. It lines cell walls to makes them strong andwaterproof.Lime water A chemical that turns milky in the presence of CO2 so is used to �nd out if it is presentLine graph Graph using points joined by lines that is used when both variables are numerical and

continuously variableLipase An enzyme of the small intestine which digests lipids into glycerol and separate fatty acids.Liver The largest organ in the body, found just under the diaphragm. It acts as thechemical factory of the bodyLoop of Henle A hairpin shaped part of the nephron that is mostly in the kidney medulla; involved

in controlling water loss in the urineLung The organ whose job is to exchange waste carbon dioxide from the body for oxygen from the airLymph A colourless liquid that comes from tissue �uid; it does not go directly back into the blood

but collects in lymph ducts and returns to the blood laterLymph node A small swelling in a lymph vessel which helps protect the body from diseaseLymph vessel A vessel that carries lymph (formed from the tissue �uid) instead of bloodLymphocyte A white blood cell with a large round nucleus; it makes antibodiesMacroevolution Slow changes in a species with time, making it su�ciently di�erent to causeextinction, or the production of a di�erent speciesMacronutrient An element that is needed in quite large amounts by a living organism eg. Calcium

and phosphorusMacrophage A type of very large white blood cell that ingests foreign particles to destroy them; they

are found all over the body, not just in the bloodMacroscopic Can be seen with the naked eyeMacula Sensory structure in the sacculus and utriculus of the inner ear; it contains otoliths on hairs

which are sensitive to gravityMale gamete The sex cell produced by a male organism; it is very small and motileMalnutrition A disease caused by the diet not being balancedMammal The class of vertebrate animals with fur which feed the newly born from milk glandsMarasmus A de�ciency disease caused by lack of food in young children; it causes thin limbs and

wasted body, lack of energy and poor growthMast cell A cell found in connective tissue; it is similar to a white blood cell and involved in the

immune response to parasitesMastication Chewing, using the teeth to physically digest foodMatrix The substance that �lls the space; for example the liquid part in the middle of a mitochondrionMedian canal The central canal of the cochlea which is �lled with endolymphMedulla The inner region of an organ such as the kidneyMedulla oblongata The region at the bottom of the hindbrain where it joins onto the spinal cord; it

contains re�ex centresMeiosis The double nuclear division that reduces the chromosomes number to the haploid conditionMeninges The three membranes that cover the brain and spinal cord

Available for free at Connexions <http://cnx.org/content/col11410/1.3>

295

Menstrual cycle A monthly cycle of change to the womb and ovaries of a woman; it controlled by thefemale hormones called oestrogens

Mesophyll The tissue that makes up the middle of a leaf. The lower part is usually spongy cells andthe upper part is palisade cells

Metabolic rate The speed at which the body is workingMetabolism The chemical reactions of lifeMeteorite Any natural lump of material that falls to earth from spaceMethane A colourless gas (CH 4 ) released by decaying organic matter that is a powerful greenhouse

gasMicroevolution Slow changes in the characteristics of a species with time that are not enough to form

a new speciesMicronutrient An element needed in very small amounts by a living organism eg. iodineMicropyle A small hole at the end of the ovule of �owering plants that lets the male gametes in; it

remains as a hole in the testa of the seed to allow water inMicroscopic Cannot be seen with the naked eye but needs a system of lenses to see itMicrovillus A very tiny �nger-like projection on a cell membrane that can be imaged with an electron

microscopeMilligram (mg) One thousandth of a gram. One gram is the standard unit of mass.Mitochondrion An organelle with two layers of membrane that holds the enzymes for aerobic respi-

rationMitosis The nuclear division that is a part of somatic cell division; it produces two identical daughter

nucleiMonocotyledon The group of plants with one cotyledon in each seed; the �ower parts are in threes

and the leaves have parallel veins (compare with `dicotyledon')Molar The back, broad crushing teeth; they are not found in the milk dentitionMolecule The smallest part of a chemical substance still having the properties of the substance;

compare with �atom�Monocyte A very large white blood cell which is able to absorb and digest disease organismsMonomer The building block of a larger molecule (a polymer); the polymer is made by joining many

of them togetherMonosaccharide A sugar that has molecules made of one unit, it cannot be split into a smaller

molecule that is still a sugar; compare with `disaccharide'Motor nerve Bundle of neurons oncerned with causing a movement such as a muscle movementMucin A slippery protein substance that is called mucus when mixed with waterMutation Any spontaneous change in the DNA of a cell; it may happen to a gene or to one ormore

chromosomesMyelin sheath A fatty protective layer round an axon; it stops impulses moving to other axons nearby

and speeds up the impulseMyxoedema Swelling or pu�ness of the body due to water being held in the tissuesNAD Nicotinamide Adenine Dinulceotide; a coenzyme that carries hydrogen atoms during respirationNADP NAD phosphate; a coenzyme that carries hydrogen atoms during photosynthesisNegative feedback When the product of a process a�ects the production in the opposite direction;

so a rise in the product causes a fall in the product, keeping it constantNephron A kidney tubule, the working unit of a kidneyNeuron A nerve cellNissl granule Small pieces of RNA that can be seen in nerve cell bodies using a good microscopeNode of Ranvier A gap in the myelin sheath, allowing the nerve impulse to travel more quicklyNon-disjunction This is when the two chromosomes in a bivalent do not separate during the �rst

division of meiosis; as a result some nuclei have extra chromosomesNutrient A chemical substance that the body needs to keep healthy; usually carbohydrate, protein,

lipid, vitamin or mineral

Available for free at Connexions <http://cnx.org/content/col11410/1.3>

296 CHAPTER 5. GLOSSARY

Nutrition When food is taken in from the environment and then used in the body (it means the sameas feeding)

Obesity A disease caused by eating too much energy food that becomes stored as fatty tissueOesophagus The gullet. A tubular part of the alimentary canal which passes from the throat through

the thorax to the stomachOptic nerve The nerve that carries sensory neurons from the eye to the brainOptimum The most favourable conditionsOrganic A compound with large molecules that contains carbon and usually hydrogen and oxygenOsmoregulation Controlling the water content of an organismOsmosis The di�usion of water through a di�erentially permeable membrane to dilute a more concen-

trated solution.Ossicle One of the three small bones of the middle ear. The Malleus (hammer), Incus (anvil) or Stapes

(stirrup) bonesOtolith Chalky grains which are a part of the macula, allowing for a sense of gravityOvary Any organ where the female gametes of an organism are madeOvule The structure in �owering plants that contains the female gamete; it becomes the seed after

fertilisationOxidation In chemistry, the adding of oxygen or removal of hydrogen OR the taking away of one or

more electrons from a moleculeOxidative Carries out oxidation, or the removal of hydrogensOxygen debt The need to take in extra oxygen to remove lactic acid formed anaerobically, which

otherwise cannot leave the bodyOzone A colourless gas (O3); it is poisonous but protects the earth from dangerousultraviolet radiationPacemaker A region in the right atrium of the heart with nerves controlling the heart beatPancreas A gland of the digestive system, with a duct that opens into the duodenum; it is both

endocrine and exocrinePandemic A disease that has become very common and very widespread eg. the AIDS pandemicPant To breathe quickly across a wet surface (eg the tongue) so as to lose heatPathogen An organism that causes diseasePelvis The inner part of the kidney where the urine collects before running o� down the ureterPepsin An enzyme of the stomach that digests proteins into smaller units such as dipeptides; it is

made as inactive pepsinogen, then activated in acid conditionsPeptide bond The special bond that joins two amino acids together.Perilymph The �uid that is round the outside of the canals made of membrane in the inner earPeripheral Near the edge; the nerves that go from the Central Nervous System towards the edge of

the body are called the peripheral nervous systemPeristalsis A squeezing motion caused by waves of alternate contraction of circular and longitudinal

musclesPetal One of the second ring of modi�ed leaves in a �ower; they are often brightly coloured to attract

pollinatorspH How acid or alkali something is: pH 7 is neutral, below pH7 is acid, above pH7 is alkalinePhagocytosis To surround, absorb and digest a small object such as a bacteriumPharynx The tube from the back of the mouth to the top of the oesophagus it carries both air and

food into the body. The eustachian tubes enter itPhenotype The actual characteristic that an organism shows; compare with `genotype'Phosphate A salt containing the phosphate ion (PO4)Phospholipid A molecule similar to a lipid molecule but with a phosphate group in place of one fatty

acid; used as an important part of cell membranesPhosphorylation A process of adding phosphatePhotolysis The splitting of water using light energy that happens at the start of photosynthesis

Available for free at Connexions <http://cnx.org/content/col11410/1.3>

297

Photoreceptor A cell which responds to the presence of light by causing the start of a nerve impulsePhotosynthesis Making food from carbon dioxide and water in the presence of chlorophyll and light.Pia mater The easily damaged inner membrane of the three meninges that cover the brain and spinal

cordPigment A coloured substancePitch A quality of sound depending on its wavelength, it is heard as high or lowPituitary gland The master gland in the middle of the head, attached to and just below the brainPlasma The liquid part of the blood; the matrix of the connective tissue called bloodPlasmolysed Describes a plant cell when the cytoplasm has shrunk away from the cell wall due to

exosmosis; the cell is then �accidPleural membrane Sheet of tissue surrounding the lungs; there are two of them, with lubricating

�uid between them to reduce friction during breathing movementsPlumule The part of the embryo in a seed that develops into the shoot of the plantPneumonia The lung disease when they �ll with liquid, making breathing di�cultPodocyte A special star shaped epithelial cell that stands o� the membrane on small legs; podocytes

allow liquids to �ow past them very easilyPoikilothermic Having a body temperature which changes with the environmentPollen grain A small structure made in the anther of seed plants that carries the male gamete to the

female parts of a �owerPollution Damaging the environment by adding man-made thingsPolyploidy Having more than two sets of chromosomes in each nucleusPolymer A large molecule that has been made by adding a many similar blocks (monomers) togetherPolysaccharide A polymer of many monosaccharide units joined to make an insoluble molecule which

is not sweet eg starch, glycogenPolyunsaturated A fatty acid with more than one C=C (see unsaturated below)Pons A band of nerve �bres that joins the medulla oblongata to the mid brainPore A small hole through somethingPortal vein A vein with capillaries at both ends (most veins join to a larger vein at one end on their

way to the heart)Potometer An instrument for measuring the rate at which water is going into a cut stemPrecursor Something that comes before the real thing eg. trypsinogen is a substance that is made

into the useful trypsinPregnant With an embryo or foetus developing in the uterusPremolar The broad crushing teeth which are also found in the milk dentitionPressure The force applied per unit area, measured in kiloPascals. Air pressure is about 100kPaPressure Potential Hydrostatic pressure in a closed system that increases the free energy of water

moleculesPrimate An order of mammals with a large brain, colour vision, nails instead of claws and forward

looking eyesProduct What you get as the result of a reaction happeningProprioceptor (or Proprioreceptor) Stretch receptor that informs the body of the muscle toneProtein A polymer of at least 70 or more amino acid molecules joined. Used by the body in enzymes,

cell membranes etc.Proximal Close toProvirus The form of a retrovirus when it contains DNA instead of the normal RNAPulmonary Of the lungs eg. the pulmonary artery is an artery to the lungsPupil The hole in the centre of the iris which lets light in to the back of the eyePure breeding When two parents will produce identical o�spring to themselves for the characteristic

being considered; they are homozygous for the characterRadiant energy The ability to do work when it is in the form of waves that can pass through a

vacuum eg light

Available for free at Connexions <http://cnx.org/content/col11410/1.3>

298 CHAPTER 5. GLOSSARY

Radicle The part of the embryo in a seed that develops into the root of the plantRadioactive Of a substance that gives o� harmful rays made of highly energetic particlesRadiotherapy A medical treatment using high energy radiation to kill active cancerous cellsRandom It happens without purpose or conscious choiceReacts To be chemically added or taken from another molecule so that it changesReceptacle The end of the �ower stalk where the �ower parts are attachedReceptor A cell or organ that collects a stimulus and turns it into a nerve impulseRecessive Any characteristic that appears in the phenotype only when it is present on both homologous

chromosomesReducing sugar A sugar acts as a reducing agent by taking oxygen or adding hydrogen to another

substance; monsaccharides and disaccharides except sucroseReduction In chemistry, the removal of oxygen or addition of hydrogen OR the adding of one or more

electrons to a moleculeRe�ex action A rapid, automatic behaviour that is protective in function and does not need conscious

thoughtRe�ex arc Pathway of neurons that causes a re�ex action to happenRefraction When a ray of light is bent at an angle as it passes out of one substance and intoanotherRenal Of the kidneyResin Preservative liquid from the bark and wood of some trees; it hardens in the airRespiration The complex chemical reactions which release energy from glucose and other molecules

and store it as ATPResponse (in life science) The activity that happens when a sense is exposed to a stimulus, causing a

change in behaviourRetina The light sensitive layer at the back of the eye; the inner of the three layers making up the wall

of the eyeRetrovirus An RNA virus that can make DNA complementary to its RNA using the enzyme called

reverse transcriptase ; HI Virus is of this typeRobust Large, strong and tough; compare with ' gracile'Rod A photoreceptor cell sensitive to movement and low light intensity, but not to colourRoot hair An extension to an epidermal cell near the root tip to give a larger surface area for absorbing

water from the soilRoot pressure A pressure due to activity in the root tissues that pushes water up the xylem of a stemSaliva An enzyme rich juice from the salivary glands in the mouth. It starts starch digestion, lubricates

the food, etc.Saturated Describes a fatty acid that cannot hold any more hydrogen atomsSchwann sheath The layer of cells that makes the myelin sheath round the outside of an axonSclera The tough outer layer of the eye; muscles are attached to it to move the eyeScrotum The bag of skin that holds the testes, keeping them cooler than the bodyScurvy The de�ciency disease caused by lack of vitamin C. Symptoms are bleeding, loose teeth and

weak connective tissueSecrete To release a useful substance that has been made by a cell or gland (compare with `excrete')Secretin A hormone from the wall of the duodenum. It causes the liver to produce bile and the

pancreas to release alkaline juiceSeed A reproductive structure holding the embryo and food stores; a feature of the higher plantsSelection To choose some things but not others; the environment chooses some suitable characteristics

to pass on to the next generation in natural selectionSelf pollination The transfer of pollen from the male part of one plant to the female part of the same

�owerSemicircular canal One of three tubes in the ear that is sensitive to dynamic balance, the direction

of accelerationSemilunar Half moon shaped; used to describe the valves where arteries leave the heart

Available for free at Connexions <http://cnx.org/content/col11410/1.3>

299

Seminiferous tubule Thin tubes in the testes that are lined with germinal epithelium to make sperm;androgen hormones are made in the gaps between them

Semipermeable It is permeable to substances with small molecules but impermeable to others withlarge molecules; same as di�erentially permeable

Sense Ability to feel or have the body aware of somethingSensory nerve Carries nerve impulses from the receptors to the Central Nervous SystemSepal One unit of the calyx, the outer layer whorl of a �ower that protects the �ower while in budSet point The level which has been �xed; eg. for body temperature it is �xed at 36,9 ◦ CSkeleton The system of bones that support the body, make red blood cells, etcSmall intestine The long narrow region of the gut where digestion is completed and absorption

happensSoda Lime A mixture of sodium hydroxide and calcium hydroxide that is able to absorb carbon dioxide

from the airSoluble Able to dissolveSolute The substance which is dissolved.Solute Potential A measure of the ability of water molecules to move by osmosis due to their free

energy compared to pure water; it is reduced by dissolved substancesSolution A mixture of a liquid such as water (the solvent) with another substance (the solute) which

becomes liquidSpeciation The formation of a new speciesSperm duct The vas deferens, a tube that carries sperm from the testes to the urethraSphincter A ring of muscle that can close o� a tube in the body when its muscles contractSpinal cord Trunk of nervous tissue that runs up through the spine to the brainSpinal nerve Peripheral nerves that come from the spinal cord region of the central nervous systemSpore A single cell from a parent organism that is able to grow into a new organism if given suitable

conditionsSquamous With �at cells like paving stones; a squamous epithelium is a very thin skinStamen The male organ of a �owering plant; made of a �lament and an antherStarch A polysaccharide for the storage of carbohydrate, common in plant cells; it is made of many

glucose units joined togetherStatic Not moving or changing; static balance is the force of gravity on the bodySterilisation i) having a medical operation to make it impossible to have children ii) killing all germs

on an objectSternum The breast bone that the upper ribs are �xed to at the frontStigma The receptive part of the female organ of a �ower at the top of the style; it is specialised to

receive pollenStimulate To speed up or increase the action of something; compare with `inhibit'Stimulus Something which causes an e�ect to start (eg. in a receptor)Stoma A tiny hole (or pore) in the surface of a plant, especially common underneath the leavesStroma The enzyme-rich liquid that �lls most of the space around the grana in a chloroplast; it is the

matrix of the chloroplastStyle The stalk that holds the stigma of a �ower in a suitable place to receive pollenSubstrate Substance that is going to react with the help of an enzymeSucrose Common sugar (cane sugar); a disaccharide made by the condensation of glucose and fructose

(C 12 H 22 O 11 )Suction Having reduced pressure, which makes the external atmospheric pressure cause a �uid to moveSugar Any simple carbohydrate that tastes sweet and can dissolve in waterSurface area The size of the surface (measured, for example, in square centimetres)Suspensory ligament A ligament that hold the lens of the eye and is attached to the ciliary bodySweat The watery liquid poured on to the skin from sweat glands to help cool itSynapse The tiny gap between two neurons because they never quite touch

Available for free at Connexions <http://cnx.org/content/col11410/1.3>

300 CHAPTER 5. GLOSSARY

Synthesis Making something by building it from simpler partsSystemic Of the body's organ systems (except the lungs) see pulmonarySystole The contraction part of the heart beatTaste bud A group of chemoreceptor cells on the tongue which are specialised for tasteTendon A strong, �brous tissue joining a voluntary muscle to a bone; when the muscle contracts it

causes the bone to moveTension A pulling or stretching forceTesta The outer coat of a seed that protects the embryo insideThalamus The part of the front brain that collects and interprets information from the sensesThermoregulation The control of body temperature; therefore a part of homeostasisThoracic Of the chest cavityThrombocyte A blood platelet; a small piece of cytoplasm which can cause blood to clotThrough gut A gut that is like a tube with a front entrance (mouth) and a back exit (anus)Thylakoid A small disc in the chloroplast made of two membranes; a pile of them makes up one

granumThymus An organ in the upper chest that matures T-lymphocytes in young peopleThyroxin The hormone from the thyroid gland that speeds up metabolic rateT-lymphocyte A white blood cell from a lymph node; it is made in the bone marrow then passes

through the thymus glandTissue �uid The liquid surrounding cells, it leaks out of capillaries from the blood plasmaTone The state of readiness of muscle cells for workTopsoil The soil nearest the surface which contains most organic matter and nutrientsTrachea The windpipe; an air passage supported by `C' shaped cartilage that leads from the throat,

through the nec,k to the lungsTranslucent Light can pass through, although you cannot see objects through itTranspiration The evaporation of water from the leaves of a plantTricuspid The heart valve which lies between the right atrium and the right ventricleTriplet code When information is stored as sets of three units eg three nitrogenous bases in RNA are

the code for one amino acidTriploid (3n) Having three sets of chromosomes in each nucleus; found in the endosperm of �owering

plantsTSH Thyroid Stimulating Hormone from the pituitary gland that stimulates the thyroid glandTumour A swelling caused by overactive cell division in a speci�c area of the body; it is caused by

cells becoming cancerous and dividing uncontrollablyTurgid Swollen and fullTympanic canal The tube that leads through the cochlea from the oval window to the vestibular

canalTympanic membrane The eardrum; it separates the outer ear from the middle earUltrasound Sound waves with a very high frequency which means they cannot be heard; they re�ect

o� tissue so can be used to make a picture of internal structuresUnsaturated A fatty acid with a C=C (two double bonded carbons) which means it is not completely

full of hydrogen atomsUrea A compound that is the break down product of amino acids; it can be removed from the body

in the urineUreter The tube running from the kidney to the bladderUrethra The tube running from the bladder to the outsideUrine The liquid which is mostly water with urea, pigments etc which is made in the kidneyVacuole A �uid �lled cavity inside a cell; in plant cells they may be very largeValve A structure that allows �uid to only pass one wayVariable Something that can change during an investigation.Variegated Of a leaf having parts that are white and parts that are green.

Available for free at Connexions <http://cnx.org/content/col11410/1.3>

301

Vasoconstriction Narrowing of the small blood vessels of the skinVasodilation Widening of the small blood vessels of the skinVein A blood vessel that carries blood from the capillaries towards the heartVena cava The largest veins in the body; they lead blood into the right atrium of the heartVentilate To move air across or through somethingVentral Of the lower side of the bodyVentricle Liquid �lled space in an organ eg. the liquid �lled spaces in the brainVesicles Small `bubbles' of liquid in a cellVestibular canal The tube that leads through the cochlea from its end to the round windowVillus A small �nger of tissue with blood vessels and lymph vessels, used to absorb food from the

intestineVision The sense of sight - being sensitive to light so that a picture if formedVitamin An organic compound required in the diet in small amounts for good health, they mostly act

as co-enzymesVitreous humour The glassy material �lling the back of the eye behind the lensVolume The total space taken up by somethingVoluntary Controlled by the will (or consciousness)Water Potential The tendency to take in water by osmosis; it is the sum of the solute potential and

pressure potentialWhite matter Nervous tissue that is rich in myelinated �bres giving it a shiny white appearanceXerophyte A plant which is well suited to living in very dry conditionsXylem A plant tissue that contains large cells with lignin, used to carry water from the rootsZygote The diploid cell made when a male and female gamete join; a fertilised egg cell

Available for free at Connexions <http://cnx.org/content/col11410/1.3>

302 GLOSSARY

Glossary

D De�nition:

= Muscle is a contractile 2 tissue 3

type of animals

H Human locomotion

= the ability you have to move from

one place to another ( walking fromyour house to a friend's)

L Locomotion

= Movement or the ability to move fromone place to another.

2http://en.wikipedia.org/wiki/Muscle_contraction3http://en.wikipedia.org/wiki/Tissue_%28biology%29

Available for free at Connexions <http://cnx.org/content/col11410/1.3>

INDEX 303

Index of Keywords and Terms

Keywords are listed by the section with that keyword (page numbers are in parentheses). Keywordsdo not necessarily appear in the text of the page. They are merely associated with that section. Ex.apples, � 1.1 (1) Terms are referenced by the page they appear on. Ex. apples, 1

B biology, � 3.1.5(209), � 4.2.4(253), � 4.2.8(274)

C cell, � 1.3.1(1)cycle, � 1.3.1(1)

D De�nition:, 98dicotyledon, � 2.1.1(53)

E Ecosystems, � 3.1.4(199)Environmental studies, � 3.1.3(182)Exercises, � 2.2.9(107)

F Fossils, � 4.2.5(254)

G Gr 10, � 3.1.3(182), � 3.1.4(199), � 4.2.3(252)grade 10, � 3.1.5(209), � 4.2.4(253),� 4.2.8(274)

H Human locomotion, 94

L life sciences, � 2.1.1(53), � 2.2.8(107),� 2.2.9(107), � 2.3.4(153), � 3.1.3(182),

� 3.1.4(199), � 3.1.5(209), � 3.1.8(221),� 4.1.3(231), � 4.2.3(252), � 4.2.4(253),� 4.2.5(254), � 4.2.7(274), � 4.2.8(274)life sciences, biosphere, grade 10, biology,� 3.1.1(155)Life sciences, Cancer, Gr 10, � 1.3.2(20)Life Sciences, Gr 10, South africa, � 1.4.4(45),� 1.4.6(51), � 2.1.4(75)Life's History, � 4.2.3(252)Locomotion, 94

M mass extinctions, � 4.2.4(253)mitosis, � 1.3.1(1)

P plants, � 2.1.1(53)

S science, � 3.1.5(209)South Africa, � 3.1.3(182), � 3.1.4(199),� 4.2.3(252)

U Unit summary, � 2.3.4(153), � 3.1.8(221),� 4.1.3(231), � 4.2.7(274)

Available for free at Connexions <http://cnx.org/content/col11410/1.3>

304 ATTRIBUTIONS

Attributions

Collection: Siyavula: Life Sciences Grade 10Edited by: Siyavula, Megan BeckettURL: http://cnx.org/content/col11410/1.3/License: http://creativecommons.org/licenses/by/3.0/

Module: "1.3.1 The Cell Cycle and Mitosis"By: Daniel WilliamsonURL: http://cnx.org/content/m43135/1.1/Pages: 1-20Copyright: Daniel WilliamsonLicense: http://creativecommons.org/licenses/by/3.0/

Module: "1.3.2 Cancer"By: Megan BeckettURL: http://cnx.org/content/m43059/1.1/Pages: 20-24Copyright: Megan BeckettLicense: http://creativecommons.org/licenses/by/3.0/

Module: "1.3.3 Summary"By: Jesuit Virtual Learning AcademyURL: http://cnx.org/content/m43130/1.1/Page: 25Copyright: Jesuit Virtual Learning AcademyLicense: http://creativecommons.org/licenses/by/3.0/

Module: "1.4.1 What is a tissue?"By: Megan BeckettURL: http://cnx.org/content/m43172/1.1/Page: 25Copyright: Megan BeckettLicense: http://creativecommons.org/licenses/by/3.0/

Module: "1.4.2 Plant Tissues"By: Jesuit Virtual Learning AcademyURL: http://cnx.org/content/m43140/1.1/Pages: 25-31Copyright: Jesuit Virtual Learning AcademyLicense: http://creativecommons.org/licenses/by/3.0/

Module: "1.4.3 Animal Tissues"By: Denver GreeneURL: http://cnx.org/content/m43137/1.1/Pages: 31-45Copyright: Denver GreeneLicense: http://creativecommons.org/licenses/by/3.0/

Available for free at Connexions <http://cnx.org/content/col11410/1.3>

ATTRIBUTIONS 305

Module: "1.4.4 Applications of indigenous knowledge and biotechnology"By: Megan BeckettURL: http://cnx.org/content/m43060/1.1/Pages: 45-48Copyright: Megan BeckettLicense: http://creativecommons.org/licenses/by/3.0/

Module: "1.4.5 Organs"By: Dr Zdzislaw (Gustav) Meglicki, JrURL: http://cnx.org/content/m43162/1.1/Pages: 48-50Copyright: Dr Zdzislaw (Gustav) Meglicki, JrLicense: http://creativecommons.org/licenses/by/3.0/

Module: "1.4.6 Summary"By: Megan BeckettURL: http://cnx.org/content/m43064/1.1/Page: 51Copyright: Megan BeckettLicense: http://creativecommons.org/licenses/by/3.0/

Module: "2.1.1 - Anatomy of dicotyledenous plants"By: SiyavulaURL: http://cnx.org/content/m43142/1.1/Pages: 53-69Copyright: SiyavulaLicense: http://creativecommons.org/licenses/by/3.0/

Module: "2.1.2 Transpiration"By: Zamekile SondzabaURL: http://cnx.org/content/m43163/1.1/Pages: 69-73Copyright: Zamekile SondzabaLicense: http://creativecommons.org/licenses/by/3.0/

Module: "Movement of Manufactured Food"Used here as: "2.1.3 - Movement of manufactured food"By: Zamekile SondzabaURL: http://cnx.org/content/m43091/1.1/Pages: 73-74Copyright: Zamekile SondzabaLicense: http://creativecommons.org/licenses/by/3.0/

Module: "2.1.4 Summary"By: Megan BeckettURL: http://cnx.org/content/m43074/1.1/Page: 75Copyright: Megan BeckettLicense: http://creativecommons.org/licenses/by/3.0/

Available for free at Connexions <http://cnx.org/content/col11410/1.3>

306 ATTRIBUTIONS

Module: "2.2.1 Skeletons"By: Denver GreeneURL: http://cnx.org/content/m43098/1.1/Pages: 75-83Copyright: Denver GreeneLicense: http://creativecommons.org/licenses/by/3.0/

Module: "2.2.2 Human Skeleton"By: Daniel WilliamsonURL: http://cnx.org/content/m43146/1.1/Pages: 83-90Copyright: Daniel WilliamsonLicense: http://creativecommons.org/licenses/by/3.0/

Module: "2.2.3 Associated Tissues"By: Tom CaswellURL: http://cnx.org/content/m43144/1.1/Pages: 90-93Copyright: Tom CaswellLicense: http://creativecommons.org/licenses/by/3.0/

Module: "2.2.4 Joints"By: Daniel WilliamsonURL: http://cnx.org/content/m43161/1.1/Pages: 93-94Copyright: Daniel WilliamsonLicense: http://creativecommons.org/licenses/by/3.0/

Module: "2.2.5 Human Locomotion"By: Daniel WilliamsonURL: http://cnx.org/content/m43153/1.1/Pages: 94-98Copyright: Daniel WilliamsonLicense: http://creativecommons.org/licenses/by/3.0/

Module: "2.2.6 Muscles"By: Denver GreeneURL: http://cnx.org/content/m43159/1.1/Pages: 98-105Copyright: Denver GreeneLicense: http://creativecommons.org/licenses/by/3.0/

Module: "2.2.7 Diseases"By: Daniel WilliamsonURL: http://cnx.org/content/m43104/1.1/Pages: 105-106Copyright: Daniel WilliamsonLicense: http://creativecommons.org/licenses/by/3.0/

Module: "2.2.8 Summary"By: Megan BeckettURL: http://cnx.org/content/m43078/1.1/Page: 107Copyright: Megan BeckettLicense: http://creativecommons.org/licenses/by/3.0/

Available for free at Connexions <http://cnx.org/content/col11410/1.3>

ATTRIBUTIONS 307

Module: "2.2.9 Exercises"By: Megan BeckettURL: http://cnx.org/content/m43169/1.1/Pages: 107-113Copyright: Megan BeckettLicense: http://creativecommons.org/licenses/by/3.0/

Module: "2.3.1 Blood circulatory system"By: Daniel WilliamsonURL: http://cnx.org/content/m43150/1.1/Pages: 114-146Copyright: Daniel WilliamsonLicense: http://creativecommons.org/licenses/by/3.0/

Module: "2.3.2 Lymph circulatory system"By: Daniel WilliamsonURL: http://cnx.org/content/m43110/1.1/Pages: 146-148Copyright: Daniel WilliamsonLicense: http://creativecommons.org/licenses/by/3.0/

Module: "2.3.3 Diseases of the heart and circulatory system"By: Daniel WilliamsonURL: http://cnx.org/content/m43156/1.1/Pages: 148-152Copyright: Daniel WilliamsonLicense: http://creativecommons.org/licenses/by/3.0/

Module: "2.3.4 Summary"By: Megan BeckettURL: http://cnx.org/content/m43080/1.1/Page: 153Copyright: Megan BeckettLicense: http://creativecommons.org/licenses/by/3.0/

Module: "3.1.1 - Biosphere"By: Colleen HenningURL: http://cnx.org/content/m43182/1.1/Pages: 155-161Copyright: Colleen HenningLicense: http://creativecommons.org/licenses/by/3.0/

Module: "3.1.2 Biomes"By: Daniel WilliamsonURL: http://cnx.org/content/m43240/1.1/Pages: 161-182Copyright: Daniel WilliamsonLicense: http://creativecommons.org/licenses/by/3.0/

Module: "3.1.3 Environment"By: Megan BeckettURL: http://cnx.org/content/m43180/1.1/Pages: 182-199Copyright: Megan BeckettLicense: http://creativecommons.org/licenses/by/3.0/

Available for free at Connexions <http://cnx.org/content/col11410/1.3>

308 ATTRIBUTIONS

Module: "3.1.4 Ecosystems"By: Megan BeckettURL: http://cnx.org/content/m43207/1.1/Pages: 199-209Copyright: Megan BeckettLicense: http://creativecommons.org/licenses/by/3.0/

Module: "3.1.5 - Energy �ow"By: SiyavulaURL: http://cnx.org/content/m43176/1.1/Pages: 209-212Copyright: SiyavulaLicense: http://creativecommons.org/licenses/by/3.0/

Module: "3.1.6 Nutrient Cycles"By: Denver GreeneURL: http://cnx.org/content/m43195/1.1/Pages: 212-219Copyright: Denver GreeneLicense: http://creativecommons.org/licenses/by/3.0/

Module: "3.1.7 Ecotourism"By: Daniel WilliamsonURL: http://cnx.org/content/m43191/1.1/Pages: 219-220Copyright: Daniel WilliamsonLicense: http://creativecommons.org/licenses/by/3.0/

Module: "3.1.8 Summary"By: Megan BeckettURL: http://cnx.org/content/m43081/1.1/Page: 221Copyright: Megan BeckettLicense: http://creativecommons.org/licenses/by/3.0/

Module: "4.1.1 Classi�cation Schemes"By: Daniel WilliamsonURL: http://cnx.org/content/m43187/1.1/Pages: 223-226Copyright: Daniel WilliamsonLicense: http://creativecommons.org/licenses/by/3.0/

Module: "4.1.2 Five kingdom system"By: Daniel WilliamsonURL: http://cnx.org/content/m43221/1.1/Pages: 226-230Copyright: Daniel WilliamsonLicense: http://creativecommons.org/licenses/by/3.0/

Module: "4.1.3 Summary"By: Megan BeckettURL: http://cnx.org/content/m43088/1.1/Page: 231Copyright: Megan BeckettLicense: http://creativecommons.org/licenses/by/3.0/

Available for free at Connexions <http://cnx.org/content/col11410/1.3>

ATTRIBUTIONS 309

Module: "4.2.1 Life's History"By: Denver GreeneURL: http://cnx.org/content/m43241/1.1/Pages: 231-235Copyright: Denver GreeneLicense: http://creativecommons.org/licenses/by/3.0/

Module: "4.2.2 Geological timescale"By: Daniel WilliamsonURL: http://cnx.org/content/m43214/1.2/Pages: 235-252Copyright: Daniel WilliamsonLicense: http://creativecommons.org/licenses/by/3.0/

Module: "4.2.3 Cambrian Explosion"By: Megan BeckettURL: http://cnx.org/content/m43205/1.1/Pages: 252-253Copyright: Megan BeckettLicense: http://creativecommons.org/licenses/by/3.0/

Module: "4.2.4 Mass extinctions"By: SiyavulaURL: http://cnx.org/content/m43235/1.1/Pages: 253-254Copyright: SiyavulaLicense: http://creativecommons.org/licenses/by/3.0/

Module: "4.2.5 Fossils"By: Megan BeckettURL: http://cnx.org/content/m43253/1.1/Pages: 254-261Copyright: Megan BeckettLicense: http://creativecommons.org/licenses/by/3.0/

Module: "4.2.6 Key Events in Life's History"By: Denver GreeneURL: http://cnx.org/content/m43229/1.1/Pages: 261-273Copyright: Denver GreeneLicense: http://creativecommons.org/licenses/by/3.0/

Module: "4.2.7 Summary"By: Megan BeckettURL: http://cnx.org/content/m43085/1.1/Page: 274Copyright: Megan BeckettLicense: http://creativecommons.org/licenses/by/3.0/

Module: "4.2.8 - exercises"By: SiyavulaURL: http://cnx.org/content/m43184/1.1/Pages: 274-280Copyright: SiyavulaLicense: http://creativecommons.org/licenses/by/3.0/

Available for free at Connexions <http://cnx.org/content/col11410/1.3>

310 ATTRIBUTIONS

Module: "Glossary"By: Daniel WilliamsonURL: http://cnx.org/content/m43254/1.1/Pages: 285-301Copyright: Daniel WilliamsonLicense: http://creativecommons.org/licenses/by/3.0/

Available for free at Connexions <http://cnx.org/content/col11410/1.3>

Siyavula: Life Sciences Grade 10The Gr10 life science textbook

About ConnexionsSince 1999, Connexions has been pioneering a global system where anyone can create course materials andmake them fully accessible and easily reusable free of charge. We are a Web-based authoring, teaching andlearning environment open to anyone interested in education, including students, teachers, professors andlifelong learners. We connect ideas and facilitate educational communities.

Connexions's modular, interactive courses are in use worldwide by universities, community colleges, K-12schools, distance learners, and lifelong learners. Connexions materials are in many languages, includingEnglish, Spanish, Chinese, Japanese, Italian, Vietnamese, French, Portuguese, and Thai. Connexions is partof an exciting new information distribution system that allows for Print on Demand Books. Connexionshas partnered with innovative on-demand publisher QOOP to accelerate the delivery of printed coursematerials and textbooks into classrooms worldwide at lower prices than traditional academic publishers.